ORTHOPEDIC MCQS ONLINE 20 OB TRAUMA 2A

ORTHOPEDIC MCQS ONLINE OB 20 TRAUMA  2A

OrthoCash 2020

 

  1. A 25 year-old-male presents with the injury seen in Figure A. Which of the following would be a contraindication to closed management with a functional brace?

     

     

     

     

    1. Radial nerve injury

    2. 1 cm shortening

    3. 20 degree varus deformity

    4. Brachial plexus injury

    5. Comminuted fracture pattern Corrent answer: 4

    Closed treatment of humeral shaft fractures with functional bracing is indicated in the vast majority of isolated injuries. An ipsilateral brachial plexus injury, however, is a contraindication to nonoperative management in a functional brace.

     

    Indications for operative management of humeral shaft fractures are limited given the high rates of union and ability of adjacent joints to compensate for deformity. Intact muscular tone is necessary to effect bony apposition in closed treatment with a functional brace. The absence of neurologic and muscle

    function in patients with a flail extremity leads to increased rates of nonunion and malunion.

     

    Rutgers and Ring conducted a retrospective review of patients managed with functional bracing of humeral shaft fractures at a single institution. The authors found a 90% overall union rate, with maintenance of shoulder and elbow motion. They caution though, that 29% of their proximal third fractures went on to nonunion.

     

    Figure A demonstrates an AP radiograph of a comminuted humeral shaft fracture with varus alignment.

     

    Incorrect Answers:

    Answer 1: Radial nerve injury is not an indication for operative management as the vast majority of radial nerve injuries recover with conservative management

    Answer 2: 1cm of shortening is an acceptable deformity with closed management

    Answer 3: 20 degree varus deformity is not an indication for operative management

    Answer 5: Fracture comminution is not a contraindication to functional bracing

     

     

     

    OrthoCash 2020

     

  2. What is the most proximal level of amputation that a child can undergo and still maintain a normal walking speed without significantly increasing their energy cost?

    1. Girdlestone hip resection

    2. Above-knee amputation

    3. Through-knee amputation

    4. Below-knee amputation

    5. Chopart amputation

     

    Corrent answer: 3

     

    A through-knee amputation, or knee disarticulation, is the most proximal level of amputation a child can undergo and still maintain a self-selected walking speed without significantly increasing their energy expenditure.

     

    Jeans et al. conducted a study of 73 children. They found that children with Above-knee amputations (AKA) and hip disarticulation amputations had significantly reduced walking speed and increased oxygen consumption. They also determined that children with a Syme's amputation, transtibial

    amputation, or knee disarticulation ambulated with approximately the same speed and oxygen consumption as did normal children in the same age group.

     

    Herbert et al. performed a study of 10 children with below knee amputations (BKA's)compared to 14 children without amputations. They found that that children with BKAs did not choose speeds different from their peers without amputations, regardless of residual limb length of the BKA.

    Illustration V is a video showing a through-knee amputation. Incorrect Answers:

    Answer 1: Girdlestone hip resection removes the entire proximal femur and

    commonly is done through a lateral approach.

    Answer 2: Above-knee amputation is performed through the femur and attention is needed for adductor myodesis to prevent abduction deformity. Answer 4: Below-knee amputation is performed commonly 12-15cm inferior to the knee joint line.

    Answer 5: Chopart amputation is performed through the transverse tarsal joints and leaves the talus and calcaneus intact.

     

     

     

    OrthoCash 2020

     

  3. A 43-year-old male sustains the injury shown in Figure A. He has an overlying 3 centimeter wound with exposed bone. Which of the following antibiotics is indicated for initial prophylaxis?

     

     

     

    1. Ciprofloxacin

    2. Vancomycin

    3. Penicillin

    4. Gentamycin

    5. Cefazolin

     

    Corrent answer: 5

     

    The scenario described above is of a type II open fracture, and current initial recommendations for these injuries include tetanus prophylaxis/update and a first generation cephalosporin (if no allergies).

     

    Hauser et al review the literature on antibiotics and open fractures. They note that use of first-generation cephalosporins, along with appropriate fracture care, minimize risk of infection. They also state that current treatment is often rooted in old (>30 years) low-level data.

     

    Saveli et al report that although methicillin-resistant Staphylococcus aureus (MRSA) is increasingly common, no evidence exists to recommend for use of MRSA prophylaxis. They recommend selecting antibiotics against MRSA for these open fractures only if significant prevalence of MRSA carriers is seen or other risk factors are present.

    Zalavras et al review open fracture treatment protocols and state that although controversy exists regarding optimal treatment of open fractures, immediate intravenous antibiotic administration should be done in order to minimize infection. They also report that the goals of treatment of these injuries are the prevention of infection, union of the fracture, and restoration of function.

    Figure A shows an isolated tibial shaft fracture. Incorrect Answers:

    Answer 1: Fluoroquinolones can be used in these injuries as second-line agents

    if allergies are present.

    Answer 2: Vancomycin is not currently recommended as a first-line agent for prophylaxis.

    Answer 3: Penicillin can be added in contaminated type III injuries. Answer 4: Gentamycin can be added in type III injuries.

     

     

     

    OrthoCash 2020

     

  4. Which of the following patients is most appropriately treated with a total elbow arthroplasty?

    1. 42-year-old laborer with an open T-type supracondylar distal humerus fracture

    2. 90-year-old male with a comminuted transolecranon fracture-dislocation of the elbow

    3. 66-year old female with a coronal shear fracture of the distal humerus

    4. 50-year-old male with a nonunion of a supracondylar humerus fracture

    5. 86-year-old female with a comminuted bicolumnar distal humerus fracture Corrent answer: 5

    Total elbow arthroplasty (TEA) has a limited, but well-described indication for treatment of distal humerus fractures. Due to the postoperative patient limitations inherent to the current prostheses, the current recommendation for use of TEA for distal humerus fractures is only for elderly patients with osteoporotic bone and comminution not amenable to stable fixation.

     

    McKee et al. peformed a randomized controlled trial of 42 patients comparing ORIF to TEA. Patients who had the TEA had better Mayo elbow scores from 3 months until 2 years postoperatively, but DASH scores were only better in the TEA group for the 1st 6 months (equal thereafter). Reoperation rates were not statistically different, but note was made of a 25% intraoperative conversion rate from ORIF to TEA after randomization. They conclude that this elderly

    population appears to adjust well to elbow limitations regardless of treatment.

     

    Illustration A shows a intraarticular fracture of the distal humerus in a patient with rheumatoid arthritis, severe osteoporosis, and preexisting arthritic changes who was treated with a total elbow replacement.

     

    Incorrect Answers:

    Answer 1: Dual plate fixation, with placement of a separate strong plate on each column and orientation of the plates either at 90° or 180° to each other, is indicated for all adult fractures involving both columns of the distal part of the humerus.

    Answer 2: This fracture pattern requires fixation of the ulna and possible reconstruction/repair of ligamentous injury.

    Answer 3: Displaced coronal shear fractures of the distal humeral articular surface require operative fixation, most typically via a lateral approach.

    Answer 4: Nonunion of a supracondylar humerus fracture typically requires revision with plate/screw constructs, unless elderly.

     

     

     

     

     

     

    OrthoCash 2020

     

  5. A 43-year-old male suffers a knee injury and undergoes the operation seen in Figures A and B. At his one-year follow-up appointment, the patient notes pain in the peri-patellar region that is aggravated by palpation and kneeling. Range-of-motion is from -5 degrees to 130 degrees. A merchant view is performed which shows no significant degenerative changes of the patellofemoral joint. Which of the following treatments would most likely alleviate his pain?

     

     

     

     

     

     

    1. Symptomatic treatment of his patellofemoral arthritis

    2. Manipulation under anesthesia

    3. Operative treatment of his non-union

    4. Knee intrarticular corticosteroid injection

    5. Removal of symptomatic hardware Corrent answer: 5

    Symptomatic hardware following surgical fixation of patella fractures is the most common complication, with ~18% of patients electing to undergo hardware removal.

     

    Patella fractures are treated in a variety of ways. Non-displaced fractures with an intact extensor mechanism are typically treated non-operatively, displaced transverse fractures are typically treated with a tension-band construct, and severely comminuted fractures of the superior and inferior pole may be treated with partial patellectomy. When using the tension-band construct, either K-wires or cannulated screws may be used. Studies have shown decreased rates of symptomatic hardware when cannulated screws are used.

     

    Melvin et al. reviewed patella fractures in adults. They cite the most common complication of operative fixation is symptomatic hardware, with literature reporting variable rates from 0-60%. This is increased in open fractures likely secondary to added insult to the soft tissues at the time of injury.

     

    Eggink et al. looked at 60 patients who underwent tension-band wiring for patella fractures. Group 1 had their k-wires bent proximally and distally, while group 2 only had their k-wires bent proximally (see article for pictures). All failures (9) occurred in the group that had the k-wires bent proximally only.

     

    Figures A and B show the post-operative films of a cannulated-screw tension band construct with a check-rein wire inserted in the tibial tubercle. Illustration A shows an example of a K-wire tension band construct used to treat a comminuted patella fracture.

     

    Incorrect Answers:

    Answer 1: Patellofemoral arthritis is a known complication following surgical fixation of patella fractures, but is less common than symptomatic implants. Answer 2: Knee stiffness is a common complication following patella fractures, however the clinical symptoms in this patient are consistent with prominent hardware

    Answer 3: Nonunion following patella fractures is a rare complication for closed injuries.

    Answer 4: Intrarticular corticosteroid injection would not be indicated in this clinical scenario.

     

     

     

     

     

    OrthoCash 2020

     

  6. A 22-year-old male sustains the injury seen in Figures A and B as the result of a motor vehicle collision. He subsequently undergoes the procedure shown in Figures C and D with a 12 millimeter nail. When would full weight-bearing be allowed after surgery?

     

     

     

     

     

     

     

     

     

     

     

    1. Immediately

    2. 4-6 weeks

    3. 8 weeks

    4. 12 weeks

    5. After consolidation is seen Corrent answer: 1

    Comminuted femoral shaft fractures treated with statically locked intramedullary nails of appropriate diameter can be treated with immediate weight-bearing, with little risk of nail/screw breakage or deformity. Immediate range of motion and weight-bearing can be extremely beneficial to short-term patient outcomes, especially in polytrauma patients.

     

    Brumback et al.(1988) reviewed 133 dynamically locked femoral nails and report that 10.5% lost fixation and/or reduction postoperatively. They recommend reviewing high-quality radiographs to determine fracture characteristics, and note that dynamic fixation only be considered for transverse fracture patterns.

     

    Brumback et al.(1988) performed a prospective series of 97 patients with statically locked femoral nails, and they report that 98% of these went on to successful union without additional procedures, and the 2% with nonunions were successfully treated with later conversion to dynamic interlocking. They also found that no implant failure or deformity occurred with early walking or weight-bearing.

     

    Brumback et al.(1999) reviewed the biomechanics of immediate weightbearing after statically locked intramedullary nails are used in a segmental femur model, and they found that it would be safe. They then looked at immediate weight-bearing of these fractures after statically locked nail insertion in a series of 35 patients, and found no loss of reduction or implant failure with immediate weight-bearing.

     

    Figures A and B show a comminuted, segmental femoral shaft fracture, while Figures C and D show the immediate postoperative radiographs of this patient after intramedullary nailing.

     

    Incorrect Answers:

    Answers 2-5: Delay in weight-bearing is not required for this injury pattern, if treated with an appropriate diameter statically locked intramedullary nail.

     

     

     

    OrthoCash 2020

     

  7. Alternating single-leg-stance radiographs are most helpful for evaluation of which of the following diagnoses?

    1. Leg length discrepancy

    2. Pelvic ring instability

    3. Femoroacetabular impingement

    4. Hip abductor weakness

    5. Lumbosacral instability

     

    Corrent answer: 2

     

    Alternating single-leg-stance radiographs are used for the diagnosis of chronic or subtle pelvic instability.

     

    Pelvic instability is a rare etiology of lumbar and low-back discomfort; patients report subjective instability and mechanical symptoms. Static radiographs (AP pelvis, inlet pelvis, outlet pelvis) are often not adequate for diagnosis of this condition.

     

    Garras et al. performed a study of healthy volunteers and reported on the normal range of physiologic motion with single leg stance radiographs. They found that multiparous women exhibited the most symphyseal motion with alternating single leg stance weightbearing AP pelvic radiographs, and up to 5mm of symphyseal translation was seen in healthy, asymptomatic patients.

     

    Siegel et al. reviewed 38 patients with pelvic instability and pain. They found that single leg stance radiographs were more indicative of instability than standard AP pelvis and inlet/outlet radiographs. They found that up to 5 cm of sympyhseal translation can be present with these injuries.

     

    Illustration A shows a single leg stance (left leg) AP pelvis radiograph with cephalad displacement of the left hemipelvis. Illustration B shows a single leg stance (right leg) AP pelvis radiograph, with cephalad displacement of the right hemipelvis.

     

    Incorrect Answers:

    Answer 1,3,4,5: Standing alternating single-leg-stance radiographs are not used for diagnosis or evaluation of these disorders.

     

     

     

     

     

     

     

     

    OrthoCash 2020

     

  8. A 32-year-old male sustains the injury shown in Figure A through D as the result of a high-speed motor vehicle collision. This particular injury is best treated with which of the following single approaches?

     

     

     

     

     

     

     

     

     

     

     

    1. Ilioinguinal

    2. Hardinge

    3. Extended iliofemoral

    4. Watson-Jones

    5. Kocher-Langenbeck

     

    Corrent answer: 3

    The radiograph and CT images shown in A-D show an acute both column acetabular fracture with segmental posterior column comminution. For difficult fractures with anterior displacement in which access to the entire anterior column is required, the ilioinguinal or Stoppa approach is ideal. These approaches allow access to the anterior column as far as the symphysis and includes the quadrilateral plate.

     

    Most both-column fractures can also be managed through these approaches, but only if the posterior fragment is large and in one piece. In this case, the posterior column is in several pieces and requires either two approaches or an extended approach, such as the iliofemoral.

     

    The original description of the ilioinguinal approach makes intraarticular visualization of the hip impossible. If visualization of the joint is required, a T extension of the incision just medial to the anterior-superior iliac spine can be made. Most surgeons accept that the joint is reduced when the fracture lines inside the pelvis are reduced, and thus this extension is very rarely used.

     

    The extended iliofemoral approach gives excellent visualization of the outer table of the ilium, the superior dome, and the posterior column. The anterior column can be visualized to the iliopectineal eminence. The exposure is similar to that provided by the triradiate approach with the additional benefit of access to the bone above the sciatic notch. The approach can be extended to provide exposure to the iliac fossa; however, this is very rarely necessary and should be avoided. Extending the approach to the inside of the pelvis greatly increases the risk of devascularizing segments of the acetabulum.

     

     

     

    OrthoCash 2020

     

  9. A 65-year-old female sustains a fall onto her outstretched right hand. The injury is closed and she is neurovascularly intact. There is no median nerve paresthesias. Radiographs are shown in Figures A and B. What is the next best step in management of this patient?

     

     

     

     

     

     

    1. Admit for acute carpal tunnel syndrome monitoring

    2. Admit for acute open reduction/internal fixation

    3. Place into removable soft splint and follow-up in clinic

    4. Place into rigid splint and follow-up in clinic

    5. Place into rigid splint and schedule for outpatient open reduction/internal fixation

    Corrent answer: 4

     

    Based on the presentation and radiographic findings, the next best step is to place the patient into a rigid splint (ie. sugar tong) and have the patient followup in clinic. It is important to closely follow this patient, to assess the stability of the reduced fracture.

     

    Distal radius fractures can be successfully managed when the articular surface has been appropriately reduced and when anatomic relationships have been restored. Indications for closed reduction and rigid immobilization include extra-articular fractures, fractures with less than 5 mm of radial shortening,

    and fractures with dorsal angulation of less than 5 degrees. Based on the AAOS Clinical Practice Guidelines, this fracture does not meet criteria for operative intervention.

     

    Lichtman et al. discuss the AAOS Clinical Practice Guidelines for managing distal radius fractures. There were only five moderate strength recommendations, which included: 1) surgical fixation for fractures that had a post-reduction radial shortening > 3 mm, dorsal tilt > 10 degrees and intraarticular step off > 2 mm, 2) rigid immobilization for non operative treatment,

    3) use of a true lateral to assess the DRUJ, 4) beginning early range of motion of the wrist after stable fixation and 5) use vitamin C to help mitigate intractable pain.

     

    LaFontaine et al. reviewed greater than 100 cases of consecutively treated distal radius fractures. Acute management involved reduction of the fracture and placement in to a plaster cast. Fractures that were more likely to displace were found to have 1) dorsal angulation of > 20 degrees, 2) comminution of the dorsal cortex, 3) intra-articular radiocarpal fractures, 4) an associated ulnar fracture or 5) age greater than 60 years of age. They recommend that patients with 3 or more of these factors should have closer radiologic follow up to guide definitive treatment.

     

    Figures A and B demonstrate a non-displaced, extra-articular distal radius fracture. Because the fracture is appropriately aligned, the patient can be placed into a rigid splint.

     

    Incorrect Answers

    Answer 1: This patient does not have evidence of acute median nerve compression.

    Answers 2, 5: Because the reduction is within the limits set forth by the AAOS guidelines, operative intervention is not warranted, even in the outpatient setting.

    Answer 3: The guidelines indicate that rigid mobilization is warranted when treating distal fractures non-operatively.

     

     

     

    OrthoCash 2020

     

  10. A 26-year-old male sustains a fall from a ladder onto his outstretched right hand. He is evaluated in the emergency room and is found to have a closed injury to his elbow without evidence of neurovascular compromise. Plain radiographs are obtained and are shown in Figures A and B. During surgery a sequential approach is used to treat each element of this injury. Which part of the procedure is felt to add the most to rotatory stability?

     

     

     

     

     

     

    1. Radial head replacement

    2. Radial head ORIF

    3. Capsular plication

    4. Lateral collateral ligament complex repair or reconstruction

    5. Medial collateral ligament complex reconstruction Corrent answer: 4

    The essential lesion that results in the most instability in a terrible triad injury of the elbow is rupture of the lateral collateral ligament. Repair of this lesion

    results in the greatest increase in elbow rotatory stability.

     

    The key components of a terrible triad injury are a radial head fracture, coronoid fracture, dislocation of the ulnohumeral joint and disruption of the lateral collateral ligament complex. While restoration of the bony anatomy is important for static stability, the key primary stabilizer that needs to be addressed is the lateral collateral ligament complex. In acute injuries LCL repair may be possible. In chronic injury, LCL reconstruction would need to be considered.

     

    Forthman et al. reviewed 34 patients with an elbow dislocation, 22 of 34 of which were terrible triad injuries. Open reduction internal fixation or radial head replacement (as appropriate) along with LCL repair was completed; the MCL was not surgically addressed. Seventeen of 22 had good or excellent results, indicating that MCL repair is not necessary.

     

    Pugh et al. discuss their surgical protocol for addressing terrible triad injuries with 28/36 of their patients obtaining good or excellent results. Their inside out protocol is described as follows: 1) coronoid fracture ORIF (capsular repair), 2) radial head fracture ORIF or replacement 3) LCL complex repair (isometric point is center of capitellum), 4) reevaluation of stability; MCL repair or hinged fixator application

     

    Jensen et.al in cadaveric studies have demonstrated that radial head replacement alone decreases varus laxity and external rotatory laxity to 14.6 &

    14.8 degrees respectively. Isolated LCL repair neutralized varus laxity, suggesting that repair of this structure was more important thatn radial head replacement for gross stability of the elbow.

     

    Figures A & B reveal a terrible triad injury with a posterolateral elbow dislocation, comminution of the radial head and injury to the coronoid process. Illustrations A & B demonstrate the post operative images for this particular patient who underwent open reduction, radial head replacement, and LCL primary repair. A video is attached that provides an overview of the terrible triad pathology.

     

    Incorrect Answers:

    Answers 1, 2: Radial head fractures need to be addressed during management of terrible triad injuries, but critical to restoration of stability is repair of the LCL complex.

    Answer 3: Capsular plication may help with the operative management, but is not considered an essential step in restoring stability

    Answer 5: The majority of these injury patterns may be addressed without repair or reconstruction of the medial collateral ligament complex

     

     

     

     

     

     

     

    OrthoCash 2020

     

  11. A 36-year-old woman presents with a grade 3 open midshaft femoral shaft fracture as the result of a high-speed motor vehicle collision. Concomitant injuries include a high-grade splenic laceration requiring splenectomy as well as a subdural hematoma that requires monitoring and maintenance of cerebral perfusion pressure. After irrigation and debridement of the open fracture, which of the following is the most appropriate management of the femoral shaft fracture at this time?

    1. Placement of antibiotic beads, wound closure and immobilization

    2. Reamed antegrade intramedullary nailing

    3. Unreamed antegrade intramedullary nailing

    4. Wound closure and Hare traction splint placement

    5. Placement of an external fixator Corrent answer: 5

    The clinical scenario is consistent with a femoral shaft fracture in a patient that is not stable from a neurosurgical perspective. Therefore, the most appropriate treatment at this time is placement of an external fixator.

     

    When evaluating polytrauma patients with long bone fractures, timing of surgery must be approached considering all clinical conditions. One factor most likely to adversely affect long term outcome in polytrauma patients with severe brain injury is intra-operative hypotension; therefore, whenever a patient has a subdural hematoma that requires close observation, definitive surgery of long bone fractures should be delayed.

     

    Flierl et al. review the immunopathophysiology of traumatic brain injury and the role of the orthopaedic surgeon in avoiding a "second hit" injury to the brain by appropriately timing the fixation of femoral shaft fractures. They recommend a multidisciplinary approach, taking individual patient-specific factors into consideration and in general, DCO principles for severe head-injured patients (GCS 3-13) and "early total care" principles for patients with mild head injury (GCS 14-15).

     

    Illustration A is a visual representation of the treatment algorithm recommended in the article.

     

    Incorrect Answers:

    Answer choice 1 is incorrect because it does not appropriately address the fracture and there is no indication for bead placement.

    Answer choices 2 and 3 are incorrect as this patient is not stable for prolonged surgery.

    Answer 4 is incorrect as this patient is already under general anesthesia and external fixation is a better option than traction for stabilization of the fracture.

     

     

     

     

     

     

    OrthoCash 2020

     

  12. A 23-year-old healthy male was involved in a motor vehicle collision and sustained the injury seen in Figure A. Physical examination after ORIF of the plateau fracture revealed a Grade 3 Lachman, varus laxity at both 0 and 30 degrees of knee flexion, and 15 degrees of external rotation asymmetry at 30 degrees of knee flexion. Which of the following structures (indicated with asterisk*) must be surgically repaired to restore stability to the knee?

     

     

     

     

     

     

     

     

     

     

     

     

     

     

     

    1. Figure B

    2. Figure C

    3. Figure D

    4. Figure E

    5. Figure F

     

    Corrent answer: 2

     

    This patient sustained a high-energy injury to the left knee, including a tibial plateau fracture as well as both anterior cruciate ligament (ACL) and posterolateral corner (PLC) injuries as indicated on the physical examination findings. In addition to ORIF of the plateau fracture, the surgical plan should include ACL reconstruction as well as posterolateral corner (PLC) reconstruction, specifically with lateral meniscal repair and allograft reconstruction of the lateral collateral ligament (LCL,asterisk in Figure C) and popliteofibular ligaments.

     

    The PLC consists of static (LCL, popliteus tendon, popliteofibular ligament, lateral capsule) and dynamic (biceps femoris, popliteus muscle, IT band, lateral head of the gastrocnemius) structures. Failure to identify a PLC injury associated with an ACL injury often leads to failure of ACL repair.

     

    Stannard et al. reported on the clinical outcomes of 22 patients with PLC injuries (7 isolated) who underwent modified 2-tailed reconstruction of the popliteofibular ligament and LCL utilizing transtibial and transfibular bone tunnels. At an average 29.5 months post-operatively, the authors noted excellent results with restoration of range of motion and stability in both the isolated and multiligamentous injured groups.

     

    Stannard et al. reported on a separate cohort of 56 patients with PLC injuries either undergoing direct repair or modified 2-tailed reconstruction. The authors noted significantly inferior results in the repair group (37% failures) compared to the reconstruction group (9%) failures, and concluded that reconstruction is the procedure of choice for the majority of patients who sustain high-energy PLC injuries.

     

    Levy et al. reported on 28 patients with multiligament knee injuries undergoing either direct PLC repair with staged cruciate ligament reconstruction or delayed single-stage multiligament reconstruction. The authors noted a significantly higher rate of failure in the repair/staged group compared to the delayed reconstruction group, and deemed reconstruction to be a more reliable option than repair alone in the multiligamentously injured knee.

     

    Figure A includes AP and lateral radiographs of the left tibia demonstrating a

    comminuted lateral tibia plateau fracture. Figure C demonstrates a gross dissection of the posterolateral corner (left knee), with the asterisk on the lateral collateral ligament (LCL)

     

    Incorrect answers:

    Answer 1: Figure B demonstrates a gross dissection of the posterolateral corner (left knee), with the asterisk on the IT band

    Answer 3: Figure D demonstrates a gross dissection of the posterolateral corner (left knee), with the asterisk on the biceps femoris

    Answer 4: Figure E demonstrates a gross dissection of the posterolateral corner (left knee), with the asterisk on the peroneal nerve

    Answer 5: Figure F demonstrates a gross dissection of the posterolateral corner (left knee), with the asterisk on the lateral head of the gastrocnemius

     

     

     

    OrthoCash 2020

     

  13. A 25-year-old, training for a marathon, presents with persistent heel pain over the past several weeks. He has difficulty with ambulation and has an antalgic gait. A squeeze test of the heel is positive. A lateral foot radiograph is shown Figure A. Of the options listed below, what is the most appropriate next step in management?

     

     

     

     

    1. EMG/NCV study

    2. Heel pad cortisone injection

    3. Physical therapy with Graston techniques to plantar fascia

    4. MRI of the foot

    5. Non-weight bearing cast for 4-6 months Corrent answer: 4

    Based on the clinical findings and imaging shown, one should be suspicious for a calcaneal stress fracture. This can be confirmed by obtaining an MRI.

    Calcaneal stress fractures are often associated with increases in training intensity. They may be seen in patients with the female athletic triad. An MRI is used to help delineate the diagnosis when it is not clear from the history and physical exam.

     

    Neufeld et al. review the diagnosis and management of plantar fasciitis. They note that there are many causes of inferior heel pain, including nerve compression, FHL tendinitis and calcaneal stress fractures. The latter commonly presents with diffuse swelling and pain with medial to lateral compression of the heel.

     

    Figure A shows a lateral radiograph of the foot with no obvious osseous abnormality of the calcaneus.

     

    Illustration A shows a T1 weighted sagittal reconstruction of an MRI that is demonstrative of a dark line; this is consistent with a stress fracture.

     

    Incorrect Answers

    Answer 1: EMG would be indicated in evaluation for tarsal tunnel syndrome Answer 2: Heel pad injections are not indicated in the treatment of calcaneal stress fractures

    Answer 3: Observation alone is not indicated

    Answer 5: A cast can be used for immobilization initially based on history and exam and discussion with patient or after the diagnosis is confirmed with a MRI. However, in this scenario 4-6 months of nonweightbearing is likely excessive.

     

     

     

     

     

    OrthoCash 2020

     

  14. A 28 year-old-male presents with the injury pattern seen in Figure A. Which of the following is a risk factor for wound complications following operative treatment?

     

     

     

     

    1. Open injury

    2. Workers' Compensation involvement

    3. Adjunct use of allograft

    4. Contralateral calcaneus fracture

    5. Male sex

     

    Corrent answer: 1

     

    According to the referenced study by Folk et al, the risk of early wound complications is highest in open injuries, diabetics, and smokers.

     

    No significant differences were seen in complication rates in terms of: age, sex, other pre-existing medical conditions, social history, mechanism of injury, time from injury to surgical stabilization, the type of incision used, use of preoperative antibiotics, or type of wound closure.

    Notably, 25% of the patients had some sort of early wound complication, and 21% of the patients required surgical treatment due to their wound complication.

     

    Their conclusion: "Smoking, diabetes, and open fractures all increase the risk of wound complication after surgical stabilization of calcaneus fractures.

    Cumulative risk factors increase the likelihood of wound complications."

     

     

     

    OrthoCash 2020

     

  15. A 86-year-old man slips on the ice and falls sustaining the injury shown in Figure A. He has Type 2 diabetes mellitus, atrial fibrillation, coronary artery disease, end-stage renal disease on dialysis and chronic obstructive lung disease. All of the following variables are associated with increased mortality at one year after injury EXCEPT?

     

     

     

     

    1. Intertrochanteric fracture

    2. Two or more pre-existing medical conditions

    3. Age of eighty-five years or more

    4. Male gender

    5. Operative fixation within 48 hours

    Corrent answer: 5

     

    Operative fixation within 48 hours is not associated with increased mortality. However, operative delay of 3 or more days results in increased mortality.

     

    Medical optimization and surgery for hip fractures in elderly patients should be performed as soon as possible following admission to hospital. Surgical intervention in elderly hip fracture patients reduces morbidity and mortality.

    Postoperatively, weightbearing as tolerated decreases the risk for poor outcomes and decreases complications e.g. muscle atrophy, pressure sores, pneumonia, urinary retention.

     

    Zuckerman et al. reviewed mortality associated with operative delay in older patients with hip fractures. They found that operative delay of 3 or more calendar days results in greater 1-year mortality. Other predictors of mortality include age >85yrs, male sex, presence of 2 or more pre-existing medical conditions, ASA grade III or IV, and inter-trochanteric fracture.

     

    Switzer et al. reviewed perioperative considerations in the geriatric patient. They showed that hip fracture repair after 2 days results in decrease in independent living, pressure sores and longer hospital stay. They found with early surgery, pain, length of hospital stay and 1-month mortality was reduced.

    Figure A shows unstable intertrochanteric fracture. Incorrect Answers:

    Answers 1, 2, 3: These conditions are all associated with increased 1-year

    mortality.

    Answer 4: One-year mortality after hip fracture is greater in men (25-30%) than in women (20%).

     

     

     

    OrthoCash 2020

     

  16. Which of the following processes relies on an exopolysaccharide glycocalyx?

    1. Osteoclast differentiation

    2. Biofilm creation

    3. Metastatic bone disease

    4. Endochondral bone formation

    5. Intramembranous bone formation Corrent answer: 2

    Exopolysaccharide glycocalyx allows bacteria to adhere to orthopaedic implants and elude antimicrobial therapies through the creation of biofilms.

     

    Biofilms are defined as a structured community of bacterial cells enclosed in a self-produced polymeric matrix and adherent to an inert or living surface.

    Biofilm production usually occurs within 4 weeks, and is extraordinarily hard to eradicate with antibiotic therapy alone. In the setting of a chronic infection of an orthopaedic implant (>4 weeks), explantation of the implant followed by antimicrobial therapy is the most reliable method of curing the infection.

     

    Nguyen et al. reviewed 21 patients who underwent revision arthroplasty with negative intraoperative cultures. After standard cultures were obtained (all negative), the implants underwent an ultrasound protocol to theoretically disrupt the biofilm. 1 of the 21 implants grew coag-negative Staph after the bath.

     

    Fux et al. reviewed biofilms with respect to orthopaedic and non-orthopaedic conditions. They discussed how aspirations are often falsely negative possibly because the microorganisms persist only within a biofilm on the synovia but not in planktonic form.

    Illustration A and the video provided show the process of biofilm formation. Incorrect Answers:

    Answer 1: Osteoclast differentiation relies on the RANK-RANKL signaling

    pathway.

    Answer 3: Osteolytic bone lesions are also caused by the RANK-RANKL pathway.

    Answer 4: Endochondral bone formation occurs when osteoblasts lay down bone on a cartilaginous framework. Exopolysaccharide glycocalyx does not play a role in this.

    Answer 5: Intramembranous bone formation does not involve exopolysaccharide glycocalyx.

     

     

     

     

     

    OrthoCash 2020

     

  17. A ballistics expert examines the effects of bullets on tissues. He defines a "penetrating missile" as one that delivers an entrance wound but no exit wound, and a "perforating missile" as one that possesses both entrance and exit wounds. He also defines bullet "yaw" as the tumble of a bullet or its tendency to turn sideways in flight. A diagram of bullet yaw is seen in Figure A. Which of the following scenarios leads to the greatest transfer of kinetic energy to tissues?

     

     

     

     

    1. Penetrating missile with mass "2m", velocity "v", yaw of 90 degrees at the point of impact

    2. Perforating missile with mass "m", velocity "2v", yaw of 0 degrees at the point of impact

    3. Penetrating missile with mass "m", velocity "2v", yaw of 90 degrees at the point of impact

    4. Perforating missile with mass "m", velocity "2v", yaw of 90 degrees at the point of impact

    5. Penetrating missile with mass "2m", velocity "v", yaw of 0 degrees at the point of impact

    Corrent answer: 3

     

    A penetrating (but not perforating) missile with highest velocity (2v) and largest yaw (90 degrees, or sideways travel) leads to greatest transfer of

    kinetic energy.

     

    The kinetic energy (E) of a bullet is proportional to its mass (m), and velocity

    (v) squared. A bullet of mass m traveling at 2v will have greater E than one of mass 2m traveling at v. Maximum energy transfer is achieved with yaw of 90 degrees (sideways). Yaw is decreased with longer distances of bullet travel, allowing a bullet to strike its target nose-on. Penetrating (non-exiting) missiles deliver all their contained kinetic energy, while perforating (exiting) missiles transfer significantly less energy to tissues

     

    Bartlett et al. reviewed ballistics and gunshot injuries. They state that energy transfer depends on 6 factors including: (1) amount of kinetic energy at impact, (2) stability and entrance profile (yaw), (3) caliber, construction and configuration of the bullet, (4) distance and path traveled within the body (penetrating vs perforating), (5) biological characteristics of tissues impacted, and (6) mechanism of tissue disruption (stretching, tearing, crushing).

     

    Figure A illustrates the concept of bullet yaw. Illustration A shows blocks of gelatin perforated by similar caliber missiles at different velocities (A, 1,000fps; B, 2,800 fps), with arrows indicating missile tracks.

     

    Incorrect Answers:

    Answers 1, 2, 4, 5: These scenarios do not lead to the greatest transfer of kinetic energy to tissues.

     

     

     

     

     

     

    OrthoCash 2020

  18. A 35-year-old man is thrown from his vehicle and sustains a left proximal femoral shaft fracture and right distal femoral shaft fracture. The surgeon elects to treat both fractures with reamed intramedullary nailing. Which of the following is true regarding the risk of malrotation?

    1. The left femur (proximal fracture) is at increased risk of internal malrotation and the right femur (distal fracture) is at increased risk of external malrotation.

    2. The left femur (proximal fracture) is at increased risk of external malrotation and the right femur (distal fracture) is at increased risk of internal malrotation.

    3. Malrotation does not depend on fracture location, but whether the nail is placed antegrade or retrograde.

    4. Both femora are at increased risk of internal malrotation.

    5. Malrotation does not depend on fracture location, but whether the nail uses a piriformis entry point or a trochanteric entry point.

    Corrent answer: 1

     

    In proximal femoral fractures, the distal fragment (femoral shaft) will be relatively internally rotated. In distal femoral fractures, the distal fragment will be relatively externally rotated.

     

    The direction of femoral malrotation depends on the pull of attached muscles. In PROXIMAL fractures, the proximal fragment is externally rotated by the iliopsoas, short external rotators and abductors. This leads to relative internal rotation of the distal fragment (femoral shaft), leading to INTERNAL malrotation. In DISTAL fractures, the proximal fragment (femoral shaft) is pulled medially by the adductors, while the distal fragment is pulled into external rotation by the lateral gastrocnemius and plantaris, leading to EXTERNAL malrotation.

     

    Lindsey et al. reviewed malrotation following femoral shaft nailing and found that malrotation was present in up to 27.6% of all femoral shaft fractures managed this way. Risk was highest with pure transverse fractures (OTA 32-A3), and Winquist III and IV fractures (OTA 32-C). Using a fracture table increases risk of internal malrotation, and supine positioning with a bump (without fracture table) increases risk of external malrotation.

     

    Ricci et al. reviewed nailing of femoral shaft fractures. They recommend the following to obtain correct rotation: (1) using alignment of the anterior superior iliac spine, patella and second toe, (2) fluoroscopic evaluation of cortical widths, key fragments or femoral anteversion, (3) checking both legs for symmetry before leaving the operating room. They also state that "The direction of femoral malrotation is based on which attached muscles are

    involved. For example, proximal femur fractures tend toward net internal rotation of the femoral shaft secondary to the pull of the iliopsoas muscle, short external rotators, and glutei on the proximal femur. The relative external rotation of the proximal femur results in internal rotation of the distal segment. Conversely, external malrotation can occur in distal femoral fractures secondary to the pull of the adductor muscles on the proximal fragment and the pull of the plantaris and lateral gastrocnemius muscles on the distal fragment."

     

    Illustration A shows how muscle attachments affect fracture deformity and predispose to malrotation.

     

    Incorrect Answers

    Answers 2, 4: Proximal fractures tend to develop internal malrotation, while distal fractures develop external malrotation.

    Answer 3: There is no known difference in malrotation between retrograde vs antegrade nail placement.

    Answer 5: There is no known difference in malrotation between piriformis vs trochanteric nail starting points.

     

     

     

     

     

    OrthoCash 2020

     

  19. A 30-year-old man is brought to your level 1 trauma center with a closed left diaphyseal humerus fracture, a closed left midshaft femur fracture, right sided rib fractures, and multiple facial fractures following a motorcycle accident. He is neurovascularly intact in his left arm and leg. Figure A shows a radiograph of his left humerus. What would be the most appropriate definitive treatment?

     

     

     

     

    1. Non-operative management of the humerus and plating of the femur

    2. Plating of the humerus and intramedullary nailing of the femur

    3. Non-operative management of the humerus and intramedullary nailing of the femur

    4. Plating of both the humerus and femur

    5. Intramedullary nailing of the humerus and plating of the femur Corrent answer: 2

    The clinical scenario involves a polytrauma patient with ipsilateral humerus and femur fractures. The humerus should be plated to facilitate early weight bearing, allowing for mobilization with crutches.

     

    Humeral shaft fractures account for 3-5% of all fractures and follow a bimodal distribution. Most humerus fractures can be treated non-operatively with a coaptation splint, followed by functional bracing. However, a strong relative indication for surgical management is a polytrauma patient. Plating of humerus fractures has high union rates and facilitates early weight bearing, which is necessary for rehabilitation with a concomitant lower extremity injury.

    Bell et al. retrospectively reviewed the outcomes of polytrauma patients treated with plate fixation for humeral shaft fractures. All but one of the fractures united, and patients had excellent function following surgery, allowing early weight-bearing through the injured extremity.

     

    Heineman et al. recently updated their systematic review of randomized controlled trials comparing plating with intramedullary nailing for humeral shaft fractures. They conclude that current literature supports a reduction in complication rates when plating humeral shaft fractures compared to intramedullary nailing.

     

    Tingstad et al. performed a retrospective study evaluating immediate weightbearing with plated humeral shaft fractures. They demonstrated that ORIF of humeral shaft fractures followed by early weight-bearing was safe and efficacious.

     

    Figure A is an AP x-ray of a left humeral shaft fracture. Illustration A shows the diaphyseal humerus fracture from Figure A following ORIF with a plate.

     

    Incorrect Answers:

    Answers 1 and 3: Non-operative management of the humerus would delay rehabilitation of this patient.

    Answers 4 and 5: Midshaft femur fractures should be treated with intramedullary nailing to facilitate early weight-bearing.

     

     

     

     

     

     

    OrthoCash 2020

  20. Risk of postoperative fixation failure for a complete sacral fracture has been associated with what variable?

    1. Anterior pelvic ring fixation method

    2. Vertical nature of sacral fracture

    3. Iliosacral screw length

    4. Number of iliosacral screws

    5. Age > 50

     

    Corrent answer: 2

     

    Illustration A, a coronal CT image, shows a vertical sacral fracture, which creates a vertically unstable pelvic ring. Percutaneous iliosacral screw fixation is a useful technique in the management of vertically unstable pelvic fractures, but a vertical sacral fracture should make the surgeon more wary of fixation failure and loss of reduction.

     

    According to the referenced study by Griffin et al, fixation failure of iliosacral screws was significantly associated with vertical sacral fractures and not with any of the other answers listed above. All cases of hardware failure occured within the first 3 weeks; a lesser relationship between hardware failure and sacroiliac joint injury was noted.

     

     

     

     

     

     

    OrthoCash 2020

     

  21. Which muscle shown in Figure A-E derives its innervation from the posterior cord of the brachial plexus?

     

     

     

     

     

     

     

     

     

    1. Figure A

    2. Figure B

    3. Figure C

    4. Figure D

    5. Figure E

     

    Corrent answer: 4

     

    Figure D shows the subscapularis. It is innervated by the upper and lower subscapular nerves derived from the posterior cord of the brachial plexus.

     

    While the subscapularis is innervated by the upper and lower subscapular nerves, it may also be innervated by a middle subscapular nerve. The upper subscapular nerve consistently originates from the posterior cord (C5-C6) and usually innervates the bulk of the muscle, but the lower subscapular nerve often shows variation. The most common variation of the lower subscapular nerve is as a branch of the axillary nerve, yet this has not been shown to place the nerve at greater risk during surgery.

     

    Lyons et al. review subscapularis tears, including diagnosis and management. While they recommend repair of acute tears, they state that repair of chronic tears is often impossible and may require tendon or muscle transfers.

     

    Kasper et al. dissected 20 cadavers to detail the innervation of the subscapularis muscle. They found a highly variable pattern of innervation, with 25% of cadavers having a lower subscapular nerve arising from the axillary nerve.

    Illustration A shows a picture of the brachial plexus. Incorrect Answers:

    Answer 1: Image shows the biceps brachii, innervated by the

    musculocutaneous nerve derived from the lateral cord.

    Answer 2: Image shows the pectoralis major, innervated by the medial and

    lateral pectoral nerves derived from the medial and lateral cords, respectively. Answer 3: Image shows the rhomboid major, innervated by the dorsal scapular nerve derived from the C5 nerve root.

    Answer 5: Image shows the supraspinatus, innervated by the suprascapular nerve derived from the upper trunk.

     

     

     

     

     

     

    OrthoCash 2020

     

  22. A 19-year-old male sustains the injury shown in Figure A while skiing. Injury to what structure should be evaluated intraoperatively during fixation of the fibula?

     

     

     

    1. Deltoid ligament

    2. Syndesmosis

    3. Proximal fibula

    4. Calcaneofibular ligament

    5. Posterior tibial tendon Corrent answer: 2

    According to the referenced study by Jenkinson et al, up to 37% of operatively treated ankle fractures can have undetected syndesmotic instability when examined intraoperatively. This is important due to the negative effects of a displaced mortise and the abnormal loading forces seen on the talus with even a 2mm lateral shift. Also, fibular fractures >4.5cm proximal to the mortise are more likely to be associated with syndesmotic instability, especially when deltoid ligament tears are present. When fixing the syndesmosis, Tornetta et al's referenced study has shown that the syndesmotic compression has no negative effects on ankle range of motion.

     

     

     

    OrthoCash 2020

     

  23. An 19-year-old male presents to the emergency room following an motor vehicle accident as an unrestrained driver. Examination reveals unilateral jugular vein engorgement. Chest and special view

    radiographs are seen in Figures A and B respectively. Following CT scan of the chest, the next step in management is

     

     

     

     

     

     

     

    1. Nonsurgical management and follow-up CT scan in 6 weeks

    2. Closed reduction in the emergency room under sedation

    3. Closed reduction in the operating room under general anesthesia with thoracic surgery on standby, followed by immobilization for 4 weeks

    4. Closed reduction in the operating room under general anesthesia with thoracic surgery on standby, followed by compression plating

    5. Open reduction in the operating room under general anesthesia, followed by transarticular pinning with K-wires

    Corrent answer: 3

     

    This patient has a right posterior sternoclavicular (SC) dislocation. Management involves closed reduction and bracing. Closed reduction should be performed with a thoracic surgeon available in the event of mediastinal involvement.

    The SC joint can dislocate anteriorly or posteriorly. Posterior dislocations are first treated with closed reduction. If closed reduction fails, open reduction is indicated. Early complications of posterior SC dislocation include pneumothorax, laceration/erosion/occlusion of great vessels, esophageal rupture and brachial plexus compression. Late complications include tracheoesophageal fistula, stridor and dysphagia.

     

    Groh et al. reviewed traumatic SC injuries. Reduction maneuvers in posterior SC dislocation include: (1) traction on the arm and slowly bringing it into extension, (2) traction with the arm in adduction and posterior pressure applied to the shoulder, and (3) pulling anteriorly on a towel clip encircling the medial clavicle. Chronic instability after posterior SC dislocations can be managed with figure-of-8 semitendinosus graft or medial clavicle resection and reattachment of the clavicle to the first rib with dacron tape.

     

    Glass et al. performed a systematic review on SC dislocations. They found mediastinal compression occurred 30% of the time with posterior dislocations.

     

    Figures A and B are radiographs demonstrating asymmetry of the SC joints, characteristic of a right posterior SC dislocation (Figure B is not a serendipity view). Illustration A demonstrates how in POSTERIOR dislocation, the clavicle appears INFERIOR, and in ANTERIOR dislocation, the clavicle appears SUPERIOR on a serendipity view radiograph respectively. Illustration B shows the imaging technique for a serendipity view radiograph. Illustration C is a reconstructed CT image of the patient showing left posterior SC dislocation.

     

    Incorrect Answers:

    Answer 1: Nonsurgical management is not advised because of potential complications of posterior dislocations. Unstable dislocations will remain dislocated at 6 weeks if untreated.

    Answer 2: Closed reduction in the emergency room is ill-advised because of potential mediastinal involvement. An operating room and thoracic surgeon should be available during the reduction maneuver.

    Answer 4: Compression plating across the sternoclavicular joint is not recommended.

    Answer 5: Pinning with K-wires is contraindicated because of potential catastrophic consequences of wire migration.

     

     

     

     

     

     

     

     

     

     

    OrthoCash 2020

     

  24. Figure A shows an isolated left ankle injury in an active 48-year-old recreational hockey player. Past medical history includes insulin dependent diabetes mellitus for 35 years. On physical examination the patient is unable to feel a 5.07 gm monofilament on the plantar aspect of his foot. His pedal pulses are palpable. Of the following options, what would be the recommended treatment?

     

     

     

     

    1. Closed reduction and casting for 6 weeks

    2. Closed reduction and casting for 12 weeks

    3. Open reduction and internal fixation with restricted weight bearing for 2 weeks

    4. Open reduction and internal fixation with restricted weight bearing for 6

      weeks

    5. Open reduction and internal fixation with restricted weight bearing for 12 weeks

    Corrent answer: 5

     

    Figure A shows an unstable, ankle fracture-dislocation in a otherwise healthy 48-year-old diabetic patient. The most appropriate management would be open reduction and internal fixation with an extended period of restricted weight-bearing.

     

    Surgical treatment of unstable ankle fractures in diabetic patients is associated with a high complication rates. Diabetic patients are inherently poor healers due to the alterations in their microvascular system. Over-fixation of the fracture and extended immobilization has been shown to reduce wound and bone healing complications associated with diabetes. Surgical techniques typically call for multiple syndesmotic screws, stronger plates (vs 1/3 tubular plates) and prolonged periods of immobilization.

     

    Jani et al. retrospectively examined a cohort of 15 patients with diabetes mellitus who sustained unstable ankle fractures. The combination of transarticular fixation (Retrograde transcalcaneal-talar-tibial fixation using large Steinmann pins or screws) and prolonged (>12 weeks), protected weightbearing provided 13 of 15 patients with a stable ankle for weight bearing.

     

    Wukich et al. compared the complication rates of ankle fracture fixation in 46 patients with complicated diabetes and 59 patients with uncomplicated diabetes. They found that patients with complicated diabetes had 3.4 times increased risk of a non-infectious complications (eg. malunion, nonunion or Charcot arthropathy) and 5 times higher likelihood of needing revision surgery/arthrodesis.

    Figure A shows AP and lat radiographs of SER4 ankle fracture-dislocation. Incorrect Answers:

    Answer 1,2: Non-operative treatment would be appropriate in stable ankle

    fractures. Again, these need to be treated with an extended period of immobilization.

    Answer 3,4: Internal fixation would be warranted in this patient, however the duration of immobilization should more than double the typical period of immobilization.

     

     

    OrthoCash 2020

     

  25. Fixed-angle implants are often used for fixation of distal femur fractures. Three commonly used implants (Implants A, B and C) are shown in Figures A, B and C respectively. Which of the following statements is true reagarding these implants?

     

     

     

     

     

     

     

     

    1. Implant B is better able to control fractures with a small distal segment than Implants A and C.

    2. Implant C is better able to control coronal plane fractures than Implants A and B.

    3. During insertion, Implant C results in removal of a larger amount of bone, compared with Implants A and B.

    4. Implant A demonstrates less subsidence and greater load to failure compared with Implant C.

    5. Implant A demonstrates lower fixation strength in torsional loading compared with Implant C

    Corrent answer: 2

     

    Implant C (locking compression plate, LCP) affords better control of coronal plane fractures than Implant A (95-degree angled blade plate, ABP) and Implant B (dynamic condylar screw, DCS).

     

    The LCP allows for better control of coronal plane and multi-fragmented fractures because the multiple locking screws at the distal end secure the plate at multiple points and allow capture of fracture fragments in different planes.

    Newer polyaxial locking plates have even greater versatility in screw positioning.

     

    Vallier et al. reviewed their experience with the ABP and LCP in distal femur fixation. They note that complications and secondary procedures (treatment of infection, nonunion, malunion, prominent implant removal) were more frequent in LCP than ABP patients.

    Gwathmey et al. reviewed the fixation of distal femoral fractures. They state that the LCP is biomechanically superior to the ABP in cyclic loading and ultimate strength. However, the LCP has less fixation strength in torsional loading.

     

    Figure A shows a 95-degree angled blade plate. Figure B shows a dynamic condylar screw. Figure C shows a locking compression plate. Illustration A shows a coronal plane fracture (Hoffa fracture, OTA 33-B3).

     

    Incorrect Answers:

    Answer 1: Implant C (LCP) is better able to control fractures with a small distal segment than Implants A (ABP) and B (DCS).

    Answer 3: Insertion of the lag screw component of Implant B (DCS) requires removal of a greater amount of bone than Implant A (ABP) and Implant C (LCP).

    Answer 4: Implant A (ABP) has greater subsidence and lower load to failure compared with Implant C (LCP).

    Answer 5: Implant A (ABP) demonstrates greater fixation strength in torsional loading compared with Implant C (LCP).

     

     

     

     

     

     

    OrthoCash 2020

     

  26. A 34-year-old man is involved in a motor vehicle accident and sustains an open tibia fracture and is treated with intramedullary nailing. For the next 4 years, he continues to have pain and persistent discharge from a sinus over his shin. He ambulates with crutches and refrains from putting weight on the extremity. The clinical appearance and radiographs are seen in Figures A and B. Wound culture reveals methicillin-resistant Staphylococcus aureus (MRSA). What is the next step in treatment?

     

     

     

    1. Retention of tibial nail, lifelong intravenous antibiotic suppression

    2. Debridement and lavage, exchange nailing using a larger diameter nail, intravenous antibiotics for 6 weeks.

    3. Debridement and lavage, excision of sinus tract, implant removal, intravenous antibiotics for 6 weeks.

    4. Debridement and lavage, addition of ring fixator, intravenous antibiotics for 6 weeks.

    5. Debridement and lavage, excision of sinus tract, exchange nailing using antibiotic impregnated-cement nail, intravenous antibiotics for 6 weeks.

    Corrent answer: 5

     

    The patient has chronic osteomyelitis and an infected nonunion complicating previous IM nailing of an open tibia fracture. Successful treatment requires debridement, removal of the existing tibial nail, placement of an antibiotic-impregnated rod and IV antibiotics. Sinus tract excision and biopsy is important to exclude malignant transformation (Marjolin's ulcer).

     

    Intramedullary infection is a recognized complication of IM nailing, especially in the setting of an open fracture. When the fracture fails to unite prior to deep infection treatment options include: nail removal and antibiotic exchange nailing, nail removal, intramedullary debridement and uniplanar external

    fixation, or nail removal and resection of the infected segment with circular frame application and bone transport.

     

    Paley et al. first described the treatment of intramedullary infection with antibiotic-impregnated cement nails in 6 femora, 2 tibiae and 1 humerus. There was no recurrence of infection. The antibiotic-impregnated cement nail fills the canal dead space while locally eluting high concentrations of antibiotics (for up to 36 wk), and is easy to remove.

     

    Qiang et al. described antibiotic-cement rod placement in 19 patients (5 femora, 14 tibiae). There was no recurrence of infection. 11 cases went on to union, 6 cases achieved partial union, 1 case had nonunion and 1 went on to amputation.

     

    Riel et al. described the method of creating a PMMA-coated nail. They advocate this method because it provides limited axial and bending stability (but no rotational stability).

     

    McGrory et al. described 53 patients with malignancy complicating chronic osteomyelitis. 50 patients had squamous cell carcinoma. Most had mixed infections, predominantly Staph and Strep.

     

    Figure A shows a poor soft tissue envelope with a draining sinus consistent with chronic osteomyelitis. Figure B is an AP radiograph showing fluffy callus formation, lack of bony bridging and interlocking screw back out. Combined with the clinical picture this would be consistent with deep infection.

    Illustration A shows the steps of making an antibiotic-impregnated cement rod using a 3 mm guidewire, chest tube and cement gun. Illustration B is a lateral radiograph of a cement rod in the tibia.

     

    Incorrect Answers:

    Answer 1: Tibial nail removal is paramount as the implant is likely seeded and infection will not be eradicated as long as foreign material is present.

    Answer 2: While exchange nailing provides mechanical support for the nonunion, immediate nailing risks reinfection. Sinus tract biopsy is important to exclude malignant transformation.

    Answer 3: The tibial nail is stabilizing an infected nonunion. Following its removal, some other form of stabilization must be substituted. Sinus tract biopsy is important to exclude malignant transformation.

    Answer 4: The addition of an external fixator does not detract from the fact that the foreign body (nail) is left behind.

     

     

     

     

     

     

     

     

    OrthoCash 2020

     

  27. A 45-year-old construction worker sustains a fall and presents with an isolated injury to his upper extremity. Radiographs of the affected wrist are shown in Figure A. After soft tissue swelling subsides, open reduction and internal fixation of the distal radius is performed. Following fixation, a "shuck" test is performed and shows persistent instability of the distal radioulnar joint. Incompetence of which of the following anatomic structures is the most likely etiology of this finding?

     

     

     

    1. Radioulnar ligaments of the TFCC

    2. Ulnar collateral ligament

    3. Fracture fixation

    4. Ulnolunate ligament of the TFCC

    5. Ulnotriquetral ligament of the TFCC Corrent answer: 1

    The patient has sustained a distal radius fracture and concomitant ulnar styloid fracture. The shuck test is performed after fixation of the distal radius to assess the status of the DRUJ, namely the radioulnar ligaments.

     

    Injuries to the DRUJ often occur with distal radius fractures. The presence of an ulnar styloid fracture may signify injury to the DRUJ. After the distal radius has been fixed, the shuck test is performed. This test is completed with the elbow at 90 degrees of flexion, the forearm in neutral rotation, followed by pronation and supination. The examiner attempts to translate the ulnar in the sagittal plane. Excessive sagittal plane ulnar translation signifies DRUJ injury.

     

    Kim et al. review the effect of ulnar styloid nonunion on functional outcome after distal radius ORIF. Of the 91 patients treated with distal radius ORIF, 22% were found to have a nonunion of the ulnar styloid. There was no

    difference in wrist functional outcomes, ulnar sided wrist pain, or DRUJ stability.

     

    Sammer et al. reviewed 144 patients undergoing ORIF of the distal radius. The DRUJ was stable in all patients after internal fixation. An ulnar styloid fracture was found in 88 patients. Functional outcome scores were not affected by the presence of an ulnar styloid fracture. Additionally, the size of the fracture, extent of displacement, or healing status did not affect the outcome.

     

    Figure A shows a PA radiograph of the wrist demonstrating a comminuted distal radius fracture with a concomitant ulnar styloid fracture. Illustration A shows an example of the shuck test used to test the DRUJ.

     

    Incorrect Answers:

    Answer 2: The ulnar collateral ligament is involved with injuries to the thumb. Answer 3: The shuck maneuver is not utilized to assess stability of the fracture.

    Answers 4, 5: The ulnolunate and ulnotriquetral ligament origins are part of the TFCC, but are not the key stabilizers of the DRUJ.

     

     

     

     

     

    OrthoCash 2020

     

  28. Which of the following deformities is most common after the amputation shown in Figure A?

     

     

     

     

    1. Pes cavus

    2. Pes planus

    3. Hindfoot valgus

    4. Equinovarus

    5. Calcaneovalgus

     

    Corrent answer: 4

     

    The most common deformity after a midfoot amputation as shown in Figure A is an equinuovarus deformity due to the pull of the Achilles and plantarflexors in face of loss of the common extensors and distal insertion of the tibialis anterior.

     

    Ng et al. review foot and ankle amputations, and review the issues inherent with each amputation level, including prosthesis fitting and use. They also mention that careful repair of all released or transected tendons is needed to maintain a plantigrade foot.

     

    Early reviews the importance of soft tissue balancing with midfoot amputations. They note that the attachment of the resected tendons into the more proximal retained bones is critical for success in restoration of foot position and ambulation capabilities.

    Figure A shows a midfoot amputation as the result of trauma. Illustration A shows the lateral view of the amputation, with an obvious equinus deformity.

     

    Incorrect Answers:

    Answer 1, 2, 3, 5: Loss of the common extensor tendons of the toes would not lead to the listed deformities.

     

     

     

     

     

     

    OrthoCash 2020

     

  29. A 68-year-old patient undergoes total knee arthroplasty for end-stage degenerative joint disease. Two years later, she trips and falls at home and sustains a fracture seen in Figures A and B. Before her fall, she was a community ambulator and had no knee pain. The component is determined to be stable and the surgeon decides to treat this fracture with closed reduction and retrograde intramedullary fixation with a supracondylar nail. Which of the following statements is true?

     

     

     

    1. The starting point tends to be more posterior than usual, resulting in hyperextension at the fracture site.

    2. An arthrotomy is not necessary

    3. A high-speed carbide burr is usually necessary to enlarge the box for nail entry.

    4. The backup plan should include devices that allow multiple points of fixation in the distal segment, such as dynamic condylar screw and fixed angle blade plate.

    5. The backup plan should include devices that resist varus collapse, such as condylar buttress plates.

    Corrent answer: 1

     

    The patient has a cruciate-retaining (CR) prosthesis. The starting point for nail entry is more posterior than normal because of the femoral component. This leads to hyperextension at the fracture site.

     

    Periprosthetic femur fractures above total knee implants occur in 2% of patients. It is important to note: (1) pre-injury function, to determine if the prosthesis was loose, (2) the type of implant (CR vs posterior stabilized, PS) as a PS implant with a closed box would make retrograde intramedullary nailing more difficult (the surgeon has to consider the size of the box vs size of the

    nail, and if the box is smaller than the nail, must be prepared to enlarge the box with a metal-cutting burr, which has inherent problems of introducing wear debris into the joint), (3) pre-fracture radiographs help determine the position of the implants (flexion-extension, varus-valgus). These fractures can be treated with non-locking condylar buttress plates (not recommended today), fixed angle devices and intramedullary nailing.

     

    McLaren et al. describe 7 osteopenic patients (mean age, 61yrs, range 47-84yrs) treated with retrograde supracondylar nailing. They suggest not reaming, and placing 2-3 screws in the distal fragment. This may require leaving the nail protruding by 1cm. They then suggest removing the protruding segment with a burr at the end of the procedure.

     

    Haidukewych et al. debate plating vs nailing in a 80yr old osteopenic patient. It may be difficult to introduce retrograde intramedullary nails through the same incision if dense scar tissue is present. On the other hand, most plates require extensive dissection and do not respect the soft tissues and fracture biology, except for LISS plates and nails.

     

    Figures A and B show a displaced Lewis and Rorabeck type II periprosthetic fracture. Illustration A shows the technique of retrograde supracondylar nailing. With the knee flexed, the fracture is reduced and the entry point is in the intercondylar notch. Illustration B shows a comparison between PS and CR implants. Note the "box" in the PS implant. This is absent in the CR implant.

    Illustration C shows the Lewis and Rorabeck classification.

     

    Incorrect Answers:

    Answer 2: A formal arthrotomy is necessary to protect the polyethylene liner Answer 3: A CR implant does not possess a box, unlike posterior-stabilized (PS) implants. PS femoral components may require box enlargement with a burr if retrograde nailing is planned.

    Answer 4: The backup plan should include plates which allow multiple points of fixation in the distal segment. These include locking periarticular plates and polyaxial locking plates, and non-locking condylar buttress plates. The DCS and ABP only have 1 (at most 2) point of fixation in the distal segment.

    Answer 5: The backup plan should include devices that resist varus collapse (especially in cases with medial comminution), such as angle-stable devices (ABP, DCS and locking plates). Non-locking condylar buttress plates will not resist varus collapse.

     

     

     

     

     

     

     

     

     

     

     

    OrthoCash 2020

     

  30. A 34-year-old female is involved in a high-speed motor vehicle collision and sustains a traumatic proximal forearm amputation. She successfully undergoes debridement and closure, and six weeks later, is fitted with her temporary prosthesis. In order to optimize her

    outcomes upon returning to work as a secretary, which of the following is recommended?

    1. Obtaining formal functional capacity testing

    2. Waiting for final prosthesis fitting prior to full release

    3. Minimize use of her prosthetic while at work

    4. Allowing return to work when full elbow range of motion is seen

    5. Offer outpatient psychological counseling Corrent answer: 5

    Upper extremity trauma has serious, acute psychological effects that can linger long after the physical injury. These effects may negatively affect patient-reported outcomes, and may also be associated with worsening pain complaints. Coping and stress management techniques can be reviewed with formal psychological counseling, and should be offered to all patients who have underwent an amputation.

     

    Richards et al surveyed 34 patients who had emergency upper extremity surgery and found high levels of psychological distress in patients, including 29% with high levels of both depression and post-traumatic stress disorder (PTSD). They also found that disability was strongly related to pain, depression, and PTSD symptoms.

     

    Mallette et al assessed the attitudes of hand surgery patients and hand surgeons regarding psychologic influences on illness and compared their attitudes with those of the general population. They found that surgeons underestimated the openness of patients to discuss psychological issues and that patients believed in the strong effect of psychologic factors on healing and pain.

    Illustration A shows a myoelectric prosthesis in a military veteran. Incorrect Answers:

    Answer 1: Formal function capacity testing is not typically necessary unless

    Workers' Compensation is involved or formal disability proceedings occur. Answer 2: Final prosthetic fitting is not necessary for full release.

    Answer 3: Prosthetic use will vary according to needs and patient factors such as pain.

    Answer 4: Return to work does not have to wait for full return of elbow range of motion.

     

     

     

     

     

    OrthoCash 2020

     

  31. A 40-year-old male patient sustains a bimalleolar ankle fracture and undergoes open reduction and internal fixation. Four months later, he returns for follow-up with mild ankle discomfort, and a radiograph is shown in Figure A. What is the most appropriate next step in treatment?

     

     

     

    1. Syndesmosis sagittal plane reduction and fixation

    2. Syndesmosis coronal plane reduction and fixation

    3. Osteotomy and revision of the fibula and syndesmosis

    4. Retrieval of osteochondral fragment

    5. Revision plating of the fibula and syndesmosis reduction and fixation.

     

    Corrent answer: 3

     

    This patient has undergone ORIF of the lateral malleolus with shortening of the lateral malleolus and lateral tibiotalar tilt. Revision surgery would entail bone grafting and re-plating of the fibula.

     

    Malunion of the fibula component of ankle fractures lead to tibiotalar instability and post-traumatic ankle arthritis. The distal fragment is usually shortened and externally rotated. The osteotomy can restore length and correct rotation.

    Markers for potential instability include: (1) asymmetry of the medial-lateral clear spaces, (2) talar tilt >2mm, (3) talar subluxation, (4) abnormal talocrural angle (normal, 75-86deg).

     

    Chu et al. opined that reconstruction for distal fibula malunions should include:

    (1) osteotomy, (2) +/- syndesmotic fixation and (3) autologous bone graft. They recommend: (1) low oblique osteotomy for fractures below the

    syndesmosis, (2) transverse osteotomy above the syndesmosis for high fractures (PER4) and low fractures with tibiofibular instability, (3) inspection of the tibiofibular joint through an anterolateral window to ensure anatomic reduction.

     

    Weber et al. described a method of corrective lengthening osteotomy of the fibula in 23 cases. They described 3 criteria for assessing normal fibular length. Seventeen patients had good-excellent results, and 6 had fair-poor results (1 of these 6 needed ankle fusion).

     

    Figure A is an AP radiograph of a distal fibula fracture fixed in a shortened position with lateral talar tilt and degenerative changes at the anterolateral tibiotalar joint. Illustration A is an anteroposterior radiograph after fibular osteotomy and correction with medial distal tibial autograft to correct talar tilt and restore anatomic fibular length. Illustration B shows the normal talocrural angle. Illustration C shows the Weber-Simpson method of fibula lengthening used in Illustration A.

     

    Incorrect Answers:

    Answer 1, 2, 5: There is no syndesmotic disruption.

    Answer 4: There is no evidence of a loose osteochondral fragment.

     

     

     

     

     

     

     

     

    OrthoCash 2020

  32. A 38-year-old concert violinist presents after falling onto a pronated, outstretched hand this morning. She complains of lateral elbow pain. Examination reveals lateral elbow tenderness, and an 80 degree arc of flexion-extension and 60 degree arc of prono-supination, with extremes of motion limited by pain. There is no bony block to motion. Radiographs of her injury are seen in Figures A through D. The most appropriate treatment plan that would allow her to return to her occupation would be

     

     

     

     

     

     

     

     

     

     

     

    1. Sling immobilization for 2 days, followed by active mobilization.

    2. Long-arm cast immobilization for 1 week, followed by active mobilization.

    3. Long-arm cast immobilization for 1 week, followed by passive mobilization.

    4. Long-arm cast immobilization for 2 weeks

    5. Open reduction and internal fixation Corrent answer: 1

    This patient has a Mason Type I radial head fracture (minimally displaced, no mechanical block, intra-articular displacement <2mm). Non-operative treatment is recommended. Sling immobilization for 2 days followed by active mobilization is recommended.

     

    Radial head fractures occur after axial loading/fall onto a pronated, outstretched hand as the most force is transmitted from the wrist to the radial head in this position. For Type II and III fractures, open reduction and internal fixation is indicated. For Type III fractures with more than 3 fragments, radial head replacement is advocated. Radial head excision in the acute setting is generally not recommended to prevent late proximal radial migration and ulnocarpal impingement, as an easily missed Essex-Lopresti injury is possible; any patient with a painful DRUJ or mid forearm in the face of a radial head fracture should not undergo excision.

     

    Paschos et al. compared (1) immediate active mobilization vs (2) sling immobilization for 2 days, followed by active mobilization vs (3) immobilization in a cast for 7 days followed by active mobilization. Early mobilization (Groups 1 and 2) had better ROM and less pain at 4 wks. Group 2 had better pain relief than Group 1 in the first 3 days, and the best functional scores at 12wks. They recommend early mobilization after a delay of 48 hours.

     

    Tejwani et al. reviewed current management of radial head and neck fractures. Most fractures can be managed nonoperatively with early motion if there is no instability or block to elbow motion. Complex fractures require ORIF or arthroplasty (fragment >1/3 of the radial head, ORIF not possible).

     

    Figures A through D are radiographs showing an undisplaced simple (AO/OTA 21-B2.1) radial head fracture.

     

    Incorrect Answers:

    Answers 2, 3, 4: Immobilization for 1 week or more leads to poorer functional outcome and pain scores. Early mobilization is important at the elbow joint.

    Answer 5: ORIF is only indicated if there is a displaced fragment or mechanical block to motion.

     

     

     

    OrthoCash 2020

     

  33. What is the primary advantage of two incisions compared to one for open reduction internal fixation of a both bones forearm fracture?

    1. lower risk of synostosis

    2. lower risk of wound complications

    3. lower rate of radial neuritis

    4. less pronator teres denervation

    5. lower malunion rate Corrent answer: 1

    Post-osteosynthetic synostosis is a known complication in both bone forearm fractures. The risk is increased in fractures of the proximal 1/3 of the ulna and radius. Other risk factors include severity of injury, head trauma, polytrauma. Vince et al found synostosis was often associated with bone fragments or hardware in the interosseous space. Bauer et al found 1/65 cases treated utilizing the two-incision approach developed synostosis, while 5/12 cases in which the fractures were stabilized using a single incision developed synostoses. They recommended a two incision approach to both bones ORIF.

     

     

     

    OrthoCash 2020

     

  34. A 25-year-old female presents to the emergency room for the fourth time in the last week. She has vague complaints of extremity pain. Physical examination by a male ER resident has been limited each visit because she is terrified of the pain that the clinician may cause. On physical examination, she is withdrawn and frightened.

    Regions of ecchymosis are noted throughout chest and abdomen. She has requested multiple radiographs, MRI and CT scans. Today's imaging (radiographs, MRI, CT scan) has been unrevealing. What is the most likely diagnosis?

    1. Malingering

    2. Complex regional pain syndrome

    3. Anxiety disorder

    4. Intimate partner violence

    5. Fibromyalgia

     

    Corrent answer: 4

     

    Based on the history and clinical presentation, the most likely diagnosis is intimate partner violence.

     

    Domestic violence or intimate partner violence can be in the form of mental or physical abuse, neglect or abandonment. Close to 25% of women will experience domestic violence. Risk factors include young age (19-29 years of age), females, pregnancy and lower socioeconomic status. Affected patients will have repeated visits to the emergency room, find reasons to stay in a treatment facility for an extended period of time and constantly seek approval

    of their partner.

     

    Shields et al. reviewed factors influence outcome in treatment of patients affected by domestic violence. They found that positive outcomes were associated with interdisciplinary approaches to management. This included better history assessment, providing written documentation regarding intervention and better access to information on community resources.

     

    Illustration A is a chart documenting the frequency of female domestic violence throughout the world as of 2012.

     

    Incorrect Answers

    Answers 1, 2, 3, 5: These conditions are not consistent with this patient’s history.

     

     

     

     

     

     

    OrthoCash 2020

     

  35. Which of the following injuries is most likely associated with the fracture seen in Figure A?

     

     

     

    1. Medial meniscal tear

    2. Lateral meniscal tear

    3. Lateral collateral ligament rupture

    4. Medial collateral ligament rupture

    5. Posterior cruciate ligament rupture Corrent answer: 2

    Lateral meniscal tears are most commonly associated with Schatzker II tibial plateau fractures (split/depressed).

     

    Soft tissue pathology is common in tibial plateau fractures. In general, fractures that are largely displaced and/or a result of high energy trauma are more likely to have associated soft tissue pathology. A majority of meniscal injuries that occur in the setting of tibial plateau fractures are meniscocapsular detachments. This has important implications for healing (more reliable healing in the vascular zone). Additionally, the meniscus usually remains in close contact with the femoral condyle, while the tibial plateau widens around it. It is generally agreed upon that meniscal tears should be repaired, if possible, at the time of internal fixation to decrease the likelihood of postraumatic arthritis.

     

    Gardner et al. review 62 patients with Schatzker II tibial plateau fractures that had an MRI preoperatively. For displaced fractures, the incidence of lateral meniscal tears was 83%, while the incidence of lateral collateral and posterior cruciate ligament injuries was 30%.

    Ringus et al. attempted to determine if the degree of lateral tibial plateau fracture depression on computed tomography (CT) images predicted the presence of lateral meniscus tears. Fractures with > 9mm depression had an eight-fold increase in lateral meniscal tears, and those younger than 48 years-old had a four-fold increase in lateral meniscal tears.

     

    Illustration A shows an MRI of a Schatzker II tibial plateau fracture with a lateral meniscal detachment and a medial meniscal tear. Illustration B shows the Schatzker Classification, I-VI.

     

    Incorrect Answers:

    Answers 1, 3, 4, 5: While these soft tissue injuries may occur, lateral meniscal tears are more common for this particular fracture pattern.

     

     

     

     

     

     

     

     

    OrthoCash 2020

     

  36. A 26-year-old male sustains a traction injury to his left arm after a motorcycle crash with resulting weakness in this left upper extremity. An electromyography (EMG) done shows normal cervical paraspinal muscle activity. Which of the following statements is true regarding this injury?

    1. The injury has likely resulted in the avulsion of several nerve roots

    2. Physical exam would likely reveal drooping of his left eyelid and anhidrosis

    3. Intact paraspinal musculature on EMG is suggestive of a post-ganglionic lesion

    4. Immediate surgical intervention with neurotization would eliminate weakness and restore function

    5. The patient would show a normal histamine test Corrent answer: 3

    Normal cervical paraspinal muscle activity on EMG is characteristic of a postganglionic injury.

     

    Determining whether a brachial plexus injury is pre- or post-ganglionic has important treatment and prognostic implications. Findings that suggest a preganglionic lesion include Horner syndrome (ptosis, miosis, anhidrosis), a medially winged scapula, loss of paraspinal musculature activity on EMG, and a normal histamine test. These injuries tend to have a worse prognosis than post-ganglionic lesions, which show an abnormal histamine test and intact cervical paraspinal activity on EMG.

     

    Moran et al. review brachial plexus injuries. They recommend a baseline EMG for non-operative injuries at 3-4 weeks time after Wallerian degeneration has occurred.

     

    Shin et al. also review brachial plexus injuries. While an MRI can visualize much of the brachial plexus and may be able to demonstrate neuromas, a CT myelogram still remains the primary mode of radiographic evaluation for nerve root avulsion in the acute setting.

     

    Illustration A shows the difference between a pre- and post-ganglionic lesion. Illustration B shows the brachial plexus for reference.

     

    Incorrect Answers:

    Answer 1: This is the definition of a pre-ganglionic lesion.

    Answer 2: Horner syndrome is typically found in patients with pre-ganglionic lesions.

    Answer 4: Neurotization is usually only considered for pre-ganglionic lesions. Answer 5: Post-ganglionic injuries show an abnormal histamine test.

     

     

     

     

     

     

     

     

     

     

    OrthoCash 2020

     

  37. A 25-year-old male pedestrian sustained a Type II open tibia fracture after being struck by a car at 10:00PM. He was transported to a Level I trauma hospital where he was given intravenous antibiotics

    and tetanus at 10:45PM. He underwent irrigation and debridement of the wound with 9L of saline solution and was treated with reamed intramedullary nail fixation at 11:45PM. A vacuum assisted dressing was placed over a 5x3cm skin deficit. What part of his overall treatment has shown to reduce the risk of infection THE MOST at the site of injury?

    1. Early tetanus administration

    2. Early intravenous antibiotic administration

    3. Reamed intramedullary nail fixation

    4. Irrigation and debridement of the open fracture with 9L of solution

    5. Vacuum assisted dressings over skin deficit Corrent answer: 2

    The most important factor shown to reduce the risk of infection at the site of an open fracture is early intravenous antibiotic administration.

     

    Infection risk after Gustilo Type II open fractures ranges from 10-20% in large studies. Antibiotic treatment initiated within 3 hours from the time of injury has shown to significantly reduce the rate of infection. Antibiotic coverage for Type II open fractures should cover gram positive bacteria. Soil-contaminated wounds should include anaerobic coverage. The dose of antibiotic given must be within a therapeutic range and titrated to the patient's weight (e.g. Ancef 2 g IV for >70 kg). Duration of antibiotic therapy has been suggested to be between 1 and 3 days, although there is no agreement on a firm end point.

     

    Pollak et al. reviewed a large cohort of open fractures treated at Level I trauma centers. They demonstrated a significant decrease in infection rate with either early direct admission (<2 hours) or transfer (<11 hours) for ONLY type III open tibia fractures. They did not not discuss timing of antibiotic treatment because this was not prospectively collected. Although they did not collect data on antibiotic treatment, the authors theorize that early transfer potentially resulted in earlier administration of antibiotics.

     

    Patzakis et al. examined a series of 1104 open fractures to determine the factors contributing to infection. They showed the most important factor in reducing the infection rate was the early administration of antibiotics.

    Illustration A is table showing the Gustilo classification of open fractures. Incorrect Answers

    Answers 1,4: These responses have been shown to reduce the risk of infection

    in open fractures, however, the most important factor has been shown to be

    early antibiotic therapy.

    Answer 3: Reamed intramedullary nailing has not been shown to decrease infection risk in open tibia fractures, however, skeletal stability will prevent ongoing soft tissue damage

    Answer 5: Vacuum-assisted wound dressings (or negative pressure wound therapy) are controversial as to whether they provide any protective effect against infection.

     

     

     

     

     

     

    OrthoCash 2020

     

  38. A 65-year-old female sustained the injury seen in Figure A after a slip and fall getting out of the shower. She is an avid golfer and walks the course on most days. Her past medical history includes borderline hypertension and migraine headaches. Which treatment option has shown to have the lowest re-operation rate and best clinical outcomes scores in this patient population?

     

     

     

     

    1. Closed reduction with cannulated screw fixation

    2. Open reduction with cannulated screw fixation

    3. Closed reduction and short intramedullary nail fixation

    4. Hemiarthroplasty

    5. Total hip arthroplasty

    Corrent answer: 5

     

    Figure A shows a displaced right femoral neck fracture in an active, healthy elderly patient. Treatment of her hip fracture with total hip arthroplasty (THA) has shown to have the lowest re-operation rates and best functional outcome scores when compared to internal fixation devices and hemiarthroplasty.

     

    Large studies have shown the incidence of femoral head AVN to be approximately 30-45% with displaced femoral neck fractures (Garden III-IV). For this reason, treatment of these injuries in elderly patients have supported arthroplasty over ORIF. Treatment of patients with THA vs. hemiarthroplasty have also been investigated. Studies have shown that THA has lower reoperation rates and improved functional outcome scores in younger, active elderly patients compared to hemiarthroplasty.

     

    Avery et al. prospectively followed a cohort of 81 patients treated with THA vs. hemiarthroplasty in high functioning elderly patients with displaced femoral neck fractures. They showed a lower mortality rate (p = 0.013) and trend towards superior function in patients treated with THA. Advantages with THA vs hemiarthroplasty must be traded off against a slightly higher risk of dislocations.

     

    Hedbeck et al. performed a randomized controlled trial involving 120 elderly patients with acutely displaced femoral neck fractures that were treated with either bipolar hemiarthroplasty or THA. They showed Harris hip scores and EQ-5D scores in favour of THA. They suggested treatment with THA in elderly, lucid patients with displaced femoral neck fractures.

     

    Figure A is a AP pelvic radiograph. The most obvious finding is a displaced femoral neck fracture.

     

    Incorrect Answers:

    Answers 1,2,3: Arthroplasty of any type has been shown to have the least amount of complications and greatest functional outcome scores compared to internal fixation devices. Indications for treatment of femoral neck fractures with internal fixation include: (1) stable or unstable fractures in young or physiologically young patients (2) stable fragility fractures (Garden I and II) in low demand elderly patients.

    Answer 4: Hemiarthroplasty is most appropriate for displaced femoral neck fractures (Garden III or IV) in low-functional demand elderly patients.

     

     

     

    OrthoCash 2020

  39. A 30-year-old man is the front seat passenger in a motor vehicle accident. He presents with deformity in his knee seen in Figures A and

    B. Radiographs are seen in Figures C and D. Examination reveals weak foot pulses. After unsuccessful attempts at closed reduction, it is noted that the pulses are no longer palpable and the foot is cool. What is the next step in treatment?

     

     

     

     

     

     

     

     

     

     

     

    1. Open reduction through an anteromedial approach, spanning external fixation. If pulses do not return, perform popliteal artery exploration.

    2. Closed reduction in the operating room using a femoral distractor. If pulses do not return, perform on-table angiogram.

    3. Manual in-line skeletal traction using a calcaneal pin in the emergency room, provisional long-leg splinting. If pulses do not return, perform computed tomography angiography in the radiology suite.

    4. Manual in-line skeletal traction using a proximal tibial pin in the emergency room, provisional long-leg splinting. If pulses do not return, perform standard angiography in the angiography suite.

    5. Open reduction through a posterior approach, spanning external fixation. If pulses do not return, perform popliteal artery exploration.

    Corrent answer: 1

     

    This patient has a posterolateral knee dislocation with a avascular limb. Urgent surgical intervention is warranted. The medial femoral condyle (MFC) has button-holed through the medial capsuloligamentous structures, leaving skin and medial subcutaneous tissues entrapped between the MFC and the joint cavity producing a ‘pucker sign’. An anteromedial approach is necessary.

    Stabilization is then best achieved with an external fixator. Persistent ischemia (absence of pulses after reduction) is an indication for popliteal artery exploration.

     

    Posterolateral dislocations are caused by a posterior-directed and rotational force, and are often irreducible. Vascular injury arises because of proximal tethering (fibrous adductor hiatus tunnel) and distal tethering (fibrous soleus hiatus tunnel) at the popliteal fossa.

     

    Rihn et al. outlined the treatment algorithm for acutely dislocated knees. If pulses return after reduction, radiographs and evaluation of ABI are indicated. If ABI<0.9, CT angiography or formal angiography is indicated. If ABI >0.9, a period of in-hospital observation is indicated. If pulses remain absent and the limb remains ischemic following reduction, emergent surgical exploration and revascularization in the operating room is necessary. The spanning external fixator supplies enough rigidity to maintain reduction and allows access for serial neurovascular examinations.

     

    Patterson et al. examined knee dislocations with vascular injury in the Lower Extremity Assessment Project (LEAP) study. Of the 18 patients in this group, all required popliteal arterial repair. Overall, 14 patients were treated with limb salvage and 4 patients were treated with an amputation. Patients with salvaged limbs had moderate to high level of disability 2 years after injury.

    Figures A and B show the clinical appearance of posterolateral knee dislocation with a ‘pucker sign’. Figures C and D are radiographs showing posterolateral knee dislocation. These radiographs classically show 1 view of the tibia, but another view of the femur. Thus, the AP XR shows an AP of the tibia, but an oblique of the femur. Similarly, the lateral XR shows a lateral of the tibia, and an oblique of the femur. This is because XR technologist determines the AP/lateral projection based on the position of the foot (which follows the tibia).

     

    Incorrect Answers:

    Answer 2: Closed reduction is contraindicated because of the risk of skin necrosis, and is also unlikely to be successful because of button-holing. While a femoral distractor is a useful tool to aid reduction, an external fixator is necessary to hold the reduction post-operatively.

    Answers 3 and 4: While the limb may be splinted to aid transfer to the operating room, the next step must involve open reduction of the dislocated limb, which can only take place in the operating room. Skeletal traction is unlikely to be successful because of button-holing.

    Answer 5: An anterior approach to the knee is necessary to free the entrapped structures.

     

     

     

    OrthoCash 2020

     

  40. An ankle-brachial index is most commonly indicated after sustaining which of the following fracture patterns, seen in Figures A-E?

     

     

     

     

     

     

     

     

     

     

     

     

     

     

    1. Figure A

    2. Figure B

    3. Figure C

    4. Figure D

    5. Figure E

     

    Corrent answer: 3

     

    Figure C shows a Schatzker IV tibial plateau fracture, or medial fracture-dislocation of the knee. Of the fracture patterns shown, Schatzker IV tibial plateau fractures have the highest incidence of vascular injury and most often require measurement of an ankle-brachial index (ABI) to rule-out associated vascular injury.

     

    Schatzker IV tibial plateau fractures (fracture of the medial plateau) are rare and are most commonly associated with high-energy trauma after a varus/axial load. At the time of initial injury, the fracture pattern produces a temporary dislocation of the knee, placing tension on the peroneal nerve and popliteal artery. Because of the likelihood of associated popliteal artery injury, ankle-brachial indices, frequent neurovascular checks, and arteriography are commonly performed following injury.

     

    Berkson et al. review high-energy tibial plateau fractures. They state that Schatzker IV fractures are usually the result of high-energy trauma, and have a high incidence of popliteal artery and peroneal nerve injury. In contrast, Schatzker V and VI are more commonly associated with compartment syndrome.

     

    Gardner et al. review 103 tibial plateau fractures. 77% of fractures had an associated rupture of either cruciate or collateral ligaments. 86% of Schatzker IV fractures had an associated medial meniscus tear.

     

    Illustration A is a worksheet for calculating the ankle brachial index. An ABI less than 0.90 has been shown to have a sensitivity exceeding 87% and a specificity exceeding 97% for identifying lower-extremity arterial injury.

     

    Incorrect Answers:

    Answer 1: Figure A shows a non-displaced Schatzker I tibial plateau fracture. These are more commonly associated with lateral meniscal pathology.

    Answer 2: Figure B shows a Schatzker V tibial plateau fracture. While it occurs through a high-energy mechanism, vascular compromise is less common than Schatzker IV fractures.

    Answer 4: Figure D shows a distal 1/3 tibial shaft fracture. These should be monitored carefully for compartment syndrome.

    Answer 5: Figure E shows an ankle fracture-dislocation. After successful reduction, vascular status usually returns to normal and further diagnostic imaging is not needed unless the foot is avascular.

     

     

     

     

     

    OrthoCash 2020

     

  41. A 34-old-male was involved in a high speed MVC. He sustained an injury to his right leg as seen in Figures A and B. He was treated initially with external fixation for 11 days before his soft-tissues would permit definitive open internal fixation. After removing the external fixator and plating the fibula, what would be next step in the operative plan for reduction and fixation of this injury?

     

     

     

     

     

    1. Application of an anterolateral pre-contoured plate with distal locking screws to the tibia

    2. Anatomical reduction and stabilization of the tibial articular surface

    3. Application of a medial pre-contoured plate with distal non-locking screws to the tibia

    4. Anatomical reduction and stabilization of the tibial metaphyseal segment

    5. Proximal screw insertion with non-locking screws to distract the metaphyseal fracture comminution

    Corrent answer: 2

     

    Figures A and B show an AO/OTA Type C Pilon fracture with metaphyseal comminution and intra-articular involvement of the tibia. There is an associated fibula fracture. The next step in the operative treatment of this injury, after removal of external fixation, would be anatomical reduction and stabilization of the articular surface.

     

    The first step in the treatment of pilon fractures involves anatomical reduction and stabilization of the articular surface. This can be accomplished with pointed reduction clamps, K-wires, lag screws, or any combination of these. Plate fixation and reduction of the metaphyseal comminution should occur after the joint surface has been re-established. Simple fibular fractures can be plated before fixation of the tibia. Comminuted fibular fractures are usually better reconstructed after the tibia has been repaired, so that the tibia and talus can be used as a guide for positioning of the lateral malleolus.

     

    Sirkin et al. reviewed the protocol for treatment of complex pilon fractures. They showed that the severity of soft-tissue injury will dictate the timing of fixation and choice of implant. To avoid wound healing problems, it is generally accepted that two or more stages of repair should be used.

    Figure A, B and C show a high energy fracture to the distal tibia. Incorrect Answers:

    Answer 1, 3: Plate preparation and insertion should be considered after

    reduction and stabilization of the articular fracture fragments. The use of medial and anterolateral locking plates are appropriate for fixation of this fracture. Longer implants improve load distribution and stability.

    Answer 4: The metaphyseal segment of this fracture does not require anatomical reduction. Basic fixation principles for metaphyseal comminution would include, re-establishing tibia length, rotation and angulation at the fracture site. Reduction of the metaphysis should occur after reduction of the articular surface.

    Answer 5: The final step in metaphyseal reduction is usually achieved by

    securing the proximal end of the plate to the tibial shaft. A kick-stand screw can be placed in the most proximal hole to increase the working length of the plate. This can be placed percutaneously if desired.

     

     

     

    OrthoCash 2020

     

  42. Which of the following ankle fractures seen in Figures A-E most likely occurred as a result of abduction of the foot relative to the tibia?

     

     

     

     

     

     

     

     

    1. A

    2. B

    3. C

    4. D

    5. E

     

    Corrent answer: 2

     

    Figure B shows a lateral ankle dislocation with a high, transverse fibula fracture. This most likely occurred as a result of abduction of the foot relative to the tibia.

     

    The Lauge-Hansen classification system was designed to predict the mechanism and ligament injury patterns of ankle fractures on the basis of radiographs. It is broken down into four different categories: 1) supination-adduction, 2) supination-external rotation, 3) pronation-abduction, and 4) pronation-external rotation. The first word is used to describe the position of the foot, while the second word is used to describe the motion of the foot relative to the tibia.

     

    Lauge-Hansen's original article is cited, in which he attempts to determine the pathoanatomy of ankle fractures. Amputated ankles were fixed to a post and

    external stresses were applied. The ankles were then dissected and fracture patterns were analyzed.

     

    Illustrations A and B further demonstrates the Lauge-Hansen classification system for supination and pronation injuries, respectively. Note the sequence in which the fractures occur as the forces move through the ankle.

     

    Incorrect Answers:

    Answer 1: Figure A shows an isolated syndesmotic injury without fracture. This most commonly occurs as a result of an external rotation injury.

    Answer 3: Figure C shows a vertical medial malleolus fracture with a distal, transverse fibula fracture consistent with a supination-adduction injury.

    Answer 4: Figure D shows a medial subtalar dislocation, which occurs through forceful inversion of the foot.

    Answer 5: Figure E shows a transverse medial malleolus fracture with a high spiral fracture of the fibula consistent with a pronation-external rotation injury.

     

     

     

     

     

     

     

    OrthoCash 2020

     

  43. A 76-year-old male sustains a minimally displaced distal radius fracture and undergoes closed treatment with a cast. Four months post-injury, he presents to the office with an inability to extend his thumb. Which of the following injuries is the most likely cause of this finding?

    1. Extensor pollicis brevis rupture

    2. Posterior interosseous nerve palsy

    3. Adhesions within the first and third dorsal wrist compartments

    4. Dorsal wrist septic tenosynovitis

    5. Extensor pollicis longus rupture Corrent answer: 5

    In this scenario, rupture of the extensor pollicis longus (EPL) is the most likely cause of his acute inability to extend his thumb. This is seen in 3-5% of nondisplaced distal radius fractures treated nonoperatively.

     

    The mechanism of tendon rupture is currently unknown, although the pathogenesis involves tenosynovitis of the EPL. There is likely a contribution of an ischemic insult at the time of injury as well. Edema and tenderness are seen over the EPL in the region of the dorsal wrist in nearly every instance.

     

    Roth et al. noted an incidence of 5% EPL ruptures in nondisplaced distal radius fractures, and found that it occurred at a mean of almost 7 weeks post-injury. The cardinal signs of injury included an inability to extend the thumb and dorsoradial wrist edema/pain.

     

    Skoff reported a 3% incidence of EPL ruptures in a retrospective series of 200 distal radius patients. He recommends a 3rd dorsal wrist release in the pre-rupture setting, and extensor indicis proprius to EPL transfer after rupture occurs.

    Illustration A shows the dorsal wrist compartment anatomy. Incorrect Answers:

    1: EPB rupture is not commonly reported after these injuries.

    2: PIN palsy is not frequently noted after nondisplaced distal radius fractures and would typically occur in the immediate postinjury period.

    3: Adhesions would limit but not prevent active extension.

    4: Septic tenosynovitis would also likely limit but not prevent active motion.

     

     

     

     

     

    OrthoCash 2020

     

  44. A 23-year-old man undergoes intramedullary nailing for a comminuted right femur fracture. Three weeks after surgery, CT scans are performed to assess for rotational malalignment. In Figure A, the angular rotation of the right femoral neck is internal rotation of 13° while the angular rotation of the left femoral neck is external rotation of 13°. In Figure B, the angular rotation of the right and left femoral condyles is external rotation of 17° and 3°, respectively. At revision surgery, in order to correct the rotational malalignment, the right distal femur must be rotated which of the following?

     

     

     

     

     

     

    1. Internally by 20°

    2. Externally by 20°

    3. Internally by 14°

    4. Externally by 14°

    5. Internally by 40°

     

    Corrent answer: 5

     

    This patient has an external rotation deformity of 40° of the distal fragment of the right femur. Correction would entail rotating the distal fragment internally by 40°.

     

    The right femoral neck (RFN) is internally rotated (IR) by 13° to the horizontal (IR13). The left femoral neck (LFN) is externally rotated (ER) by 13° to the horizontal (ER13). The right distal fragment (RDF) is ER17. The left distal fragment (LDF) is ER3. Bringing both femoral necks to ER0/IR0 gives the absolute amount of rotation of the distal fragment to the horizontal. To do this, the RFN has to EXTERNALLY rotate 13° and the LFN has to INTERNALLY rotate 13°. Thus, RDF has a total of ER(13+17)=ER30, and LDF has IR(13-3)=IR10 to the horizontal. To correct the RDF from ER30 to IR10, internal rotation of 40° must occur.

    Malrotation is the most common cause of limb deformity after nailing. To avoid this, patients should be examined for rotation and limb length after insertion of static interlocks, before leaving the operating room. Correction is easier to perform prior to fracture union. Drill-hole cutout is possible if correction<20° if the previous distal locking site is to be used because of the proximity of the new interlock to the previous interlock. To avoid this, (1) use alternative locking holes or the dynamic locking slot, or (2) advance or retract the nail to avoid previous locking sites.

     

    Lindsey et al. reviewed rotational malalignment after femoral nailing. The incidence of rotational malalignment was 27.6%. Normal femoral neck anteversion (angle of the femoral neck relative to the transverse axis through the femoral condyles) is 11-13°. Some patients have up to 15° difference in version between limbs.

     

    Jaarsma et al. reviewed rotational malalignment after nailing of 76 femoral fractures. The incidence of rotational malalignment >=15° was 28%. Patients with an ER malalignment (n = 12) had more functional problems than patients with a IR malalignment (n = 9).

     

    Incorrect Answers:

    Answer 1: This answer would be correct if LFN was IR13, and LDF was IR3, giving absolute LDF rotation of ER10. Then to correct ER30 (right) to ER10 (left), internal rotation of 20° would be needed.

    Answers 2, 4: The RDF is more externally rotated. Correction must involve internal rotation.

    Answer 3: This answer would be correct if LFN was IR13, giving absolute LDF rotation of ER16. Then to correct ER30 (right) to ER16 (left), internal rotation of 14° must occur.

     

     

     

    OrthoCash 2020

     

  45. Figures A through E are paired diagrams depicting the anteroposterior and lateral profiles of the proximal tibia. Which of the following figures has arrows that correspond to the ideal entry point for intramedullary nailing of a proximal third diaphyseal tibial fracture?

     

     

     

     

     

     

     

     

     

     

     

     

     

     

    1. Figure A

    2. Figure B

    3. Figure C

    4. Figure D

    5. Figure E

     

    Corrent answer: 2

     

    The ideal entry point is just medial to the lateral tibial spine (AP view), and anterior to the anterior articular margin (lateral view).

     

    The ideal insertion point allows nail placement (1) without injury to the menisci or articular cartilage, and (2) allows the nail to travel down the center of the intramedullary canal of both fracture fragments, ending in the center of the ankle perpendicular to the joint. An excessively medial insertion point leads to valgus malalignment. Malposition in the sagittal plane may cause anterior knee pain from unrecognized intra-articular injury.

     

    Mcconnell et al. reviewed the anatomic safe zone for tibial portal placement. The safe zone measures 12.6-22.9 mm wide. With proximal tibial fractures, a

    superolateral portal minimizes stress during nail insertion and decreases angulation and rotation. The center point is slightly medial to the lateral tibial spine (on AP radiograph) and adjacent and anterior to the anterior articular margin (on lateral radiograph).

     

    Song et al. evaluated the insertion point by examining the intramedullary canal with CT. The mean length from the LTS to the canal axis center was medial 1.1 mm (medial 1.5 mm in males, medial 0.9 mm in females).

     

    Illustration A shows the safe zone on an AP radiograph. The black arrow represents the center of the safe zone. The white arrows indicate the medial-lateral boundaries of the safe zone. Illustration B is a lateral view of the proximal tibia. The black arrow shows the starting portal adjacent to the anterior articular margin.

     

    Incorrect Answers:

    Answers 1, 3, 4, 5: The ideal entry point is medial to the lateral tibial spine and anterior to the anterior articular margin.

     

     

     

     

     

     

     

     

    OrthoCash 2020

     

  46. A 44-year-old female sustains the injury shown in Figures A and B as the result of a motor vehicle collision. She undergoes immediate four compartment leg fasciotomy and placement of a spanning external fixator. A post-fixator CT scan image is shown in Figure C. After allowing her soft tissues to improve, the optimal definitive stabilization of this fracture is which of the following?

     

     

     

     

     

     

     

     

     

    1. Continued use of knee-spanning external fixator

    2. Conversion of external fixator to a simple hinged knee fixator

    3. Conversion to intramedullary nailing

    4. Open reduction and internal fixation with a lateral locked plate

    5. Open reduction and internal fixation with medial and lateral plates Corrent answer: 5

    Treatment of a comminuted bicondylar tibial plateau fracture such as the one shown in Figures A and B is best treated with dual plates (or more), as the posteromedial fragment(s) is usually a large section of the medial plateau and is poorly stabilized from a single lateral plate.

     

    Barei et al (2008) retrospectively reviewed 57 patients with bicondylar tibial plateau fractures, evaluating the frequency and morphologic characteristics of the posteromedial fragment in this injury pattern. They found that 74% of these injuries had a posteromedial fragment that may require alternate or supplementary fixation methods when managing this injury pattern.

     

    Barei et al (2006) reviewed 83 bicondylar tibial plateau fractures that were treated with medial and lateral plate fixation through two exposures. They noted that residual dysfunction is common. Accurate articular reduction was possible in about 55% of the patients and the reduction was associated with better outcomes within the confines of the injury severity.

     

    Figure A and B show a comminuted bicondylar tibial plateau fracture. Figure C

    is an axial CT cut showing the medial fragments (anterior and posterior).

     

    Incorrect Answers:

    Answer 1: Definitive use of the spanning external fixator would lead to significant knee stiffness.

    Answer 2: Conversion to a hinged knee fixator is not commonly recommended for this injury pattern.

    Answer 3: This comminuted tibial plateau fracture is not amenable to treatment with an intramedullary nail.

    Answer 4: Use of more than one plate to instrument this fracture is necessary.

     

     

     

    OrthoCash 2020

     

  47. A 25-year-old male presents with a Gustilo-Anderson IIIB tibia fracture after a motor cycle crash. Which of the following descriptions matches this classification?

    1. Distal third tibial shaft fracture with extensive soft tissue injury and a pale foot following adequate closed reduction

    2. Posterolateral ankle fracture-dislocation with 8cm laceration on medial ankle, amenable to primary closure

    3. Midshaft tibia fracture with 1cm anterior laceration requiring compartment releases intraoperatively

    4. Proximal third tibial shaft fracture with extensive soft tissue loss requiring gastrocnemius flap

    5. Closed pilon fracture that will require delayed surgical fixation after initial external fixation

    Corrent answer: 4

     

    A Gustilo-Anderson IIIB fracture is defined as a high-energy open fracture with extensive soft tissue damage and periosteal stripping that will require a free flap or rotational flap for coverage.

     

    The Gustilo-Anderson classification is the most commonly used classification system for open fractures. In addition to looking at the size of the wound, it looks at the presence of neurovascular injury, degree of contamination, and degree of periosteal stripping. While it is extremely useful for communication and descriptive purposes, it also serves to guide treatment and predict prognosis (risk for infection).

     

    Gustilo et al. retrospectively analyze 673 open fractures and prospectively analyze 352 patients treated for open fractures. 70.3% of the open wounds were culture positive, leading the authors to conclude that antibiotics were

    therapeutic, rather than prophylactic, in most cases. Additionally, they recommend delayed surgical fixation in the setting of extensive soft tissue damage.

     

    Gustilo et al. analyze factors that contribute to the increased incidence of complications associated with Type III open fractures. The four factors identified were 1) massive soft tissue damage, 2) wound contamination, 3) compromised vascularity, and 4) fracture instability.

    Illustration A shows the Gustilo-Anderson classification system. Incorrect Answers:

    Answer 1: This describes a Type IIIC fracture, as vascular repair will be

    required given the lack of distal circulation.

    Answer 2: This describes a Type IIIA fracture, as it is amenable to primary closure.

    Answer 3: The need for fasciotomies is not accounted for in the Gustilo-Anderson classification system, and there is no mention of flap coverage for this particular injury.

    Answer 5: This is a closed injury, thus is not classified according to the Gustilo-Anderson system.

     

     

     

     

     

     

    OrthoCash 2020

     

  48. A 35-year-old male with a pronation abduction ankle injury would have which of the following radiographs?

     

     

     

     

     

     

     

     

    1. Figure A

    2. Figure B

    3. Figure C

    4. Figure D

    5. Figure E

     

    Corrent answer: 4

     

    Figure D shows a pronation abduction ankle fracture according to the Lauge-Hansen classification. This injury pattern is associated with a comminuted fibula fracture above the level of the syndesmosis and frequently has a concominant syndesmotic injury.

     

    Lauge-Hansen's classic article describes in detail his proposed classification of ankle fractures based on both the position of the foot (supination or pronation) and an externally applied deforming force (adduction, abduction, external rotation). The Lauge-Hansen classification system is based on cadaveric experiments using manually applied forces and roentographs performed at each stage of injury.

     

    Edwards and DeLee review their results in managing diastasis of the tibiofibular joint without an associated fracture. The authors propose a classification system of this uncommon injury and and theorize that the injury results from a pronation abduction mechanism.

    Incorrect Answers:

    Answer 1: Figure A represents a supination adduction fracture based on the vertical medial malleolar fracture, medial dislocation on the talus, and low transverse fibula fracture.

    Answer 2: Figure B represents a pronation external rotation injury; note the high oblique fibula fracture and corresponding transverse medial malleolus fracture.

    Answer 3: Figure C represents a supination external rotation ankle injury based on the oblique fibula fracture at the level of syndesmosis and the associated transverse medial malleolar fracture.

    Answer 4: Figure E represents a pilon fracture based on the significant articular comminution signifiying an axial loading mechanism instead of a rotational injury.

     

     

     

    OrthoCash 2020

     

  49. A 51-year-old female sustained a comminuted radial head fracture with 4 fragments and an associated elbow dislocation. She was initially closed reduced and splinted with the elbow joint in a reduced position and presents to the orthopedists office 10 days later. In response to the patient's question of what treatment offers the best chance for a good outcome, the surgeon should recommend?

    1. Excision of the radial head

    2. ORIF of the radial head

    3. Continued splinting, no surgery

    4. Radial head arthroplasty

    5. Hinged external fixation Corrent answer: 4

    In the context of elbow instability, optimal treatment of a comminuted radial head fracture with greater than three fragments is with a radial head replacement.

     

    Radial head fractures are often seen in conjunction with injuries such as elbow dislocations and DRUJ instability. Although minimally displaced injuries can be treated non-operatively, an ORIF can be utilized when there is limited comminution of the radial head. Improved outcomes are demonstrated when radial head arthroplasty is used in the setting of greater than 3 fracture fragments.

     

    Ring et al. reviewed 56 patients who had been treated with an ORIF of the

    radial head at 48 months after injury. They found that patients had unsatisfactory outcomes for patients who had an ORIF of radial fractures with greater than 3 articular fragments. They recommend ORIF of radial head fractures with 3 or few fragments.

     

    Moro et al. reviewed functional outcomes of patients treated with radial head arthroplasty for un-reconstructible fractures. They found that patients treated with metal radial head implants had mild physical impairment. Short term follow-up indicates that radial head arthroplasty is a reasonable option for severely comminuted radial head fractures.

     

    Illustration A shows a 3D reconstruction of a radial head fracture with a coronal and sagittal split. The presence of 3 fragments portends good outcomes with an ORIF procedure.

     

    Incorrect Answers

    Answer 1: Resection is not indicated in this patient to improve her function and optimize outcome

    Answer 2: With greater than 3 fragments, better outcomes have been reported with radial head replacement.

    Answer 3: Non operative management will likely lead to loss of motion and radiocapitellar arthritis

    Answer 5: Hinged external fixation is not indicated in this patient

     

     

     

     

     

     

    OrthoCash 2020

  50. A 42-year-old female sustains the injury shown in Figure A as the result of a fall from a ladder. Which of the following is the most common complication after the procedure shown in Figure B?

     

     

     

     

     

     

     

    1. Knee arthrofibrosis

    2. Symptomatic implant

    3. Implant failure

    4. Patella alta

    5. Patella baja

     

    Corrent answer: 2

     

    Patella fractures have a high incidence of symptomatic implants due to their subcutaneous nature. Most series note a high incidence of removal of fixation constructs, with wire/tension band constructs typically having the highest incidence of later removal.

     

    Lebrun et al. reviewed a series of 241 operatively treated patella fractures and

    found that at over 6 years postoperatively, significant symptomatic complaints and functional deficits persisted based on validated outcome measures as well as objective physical evaluations. Removal of symptomatic fixation was required in 52% of the patients treated with osteosynthesis, whereas 38% of those with retained fixation self-reported implant-related pain at least some of the time.

     

    Melvin et al. review adult patellar fracture diagnosis and treatment. They also report a high incidence of symptomatic implant and need for removal.

     

    Figure A shows a displaced patella fracture, and Figure B shows the patella fracture after reduction and plating. Illustration A shows an example of patellar fracture fixation with a screw as well as K-wires and a tension band construct. Migration of a K-wire fragment is seen in the posterior knee.

     

    Incorrect Answers:

    1: Knee arthrofibrosis can occur but is generally transient and improves with therapy.

    3: Implant failure can most commonly occur with wire-based constructs, but is less common than symptomatic implants.

    4 and 5: Patella alta and baja does not typically occur as it can with quadricep or patellar tendon injuries.

     

     

     

     

     

     

    OrthoCash 2020

  51. A 17-year-old football player is tackled with an opposing player's helmet hitting him hard in the abdomen. He is knocked backwards and suffers a diaphyseal femur fracture. He denies any loss of consciousness. Vital signs reveal a heart rate of 118, mean arterial pressure (MAP) of 68, and a respiration rate of 32 per minute. A FAST ultrasound study shows trace free fluid in the perisplenic space. A CBC taken prior to bolus IV fluids reveals a hematocrit of 48%, and a blood gas shows a lactate level of 1.8 and a base excess of -2.0. Which of the follow statements regarding the patient's hemodynamic status is correct?

    1. A well-placed and well-calibrated arterial line would be the most helpful clinical tool for determining when this patient is out of shock

    2. The hematocrit well within normal limits means the patient is not in hemodynamic shock

    3. A combination of heart rate greater than 120 and MAP less than 65 equates to poor tissue perfusion levels

    4. Normal lactate levels and base excess are markers of adequate tissue perfusion

    5. His orthopaedic injury alone cannot explain his vital sign derangements and an exploratory laparotomy is indicated

    Corrent answer: 4

     

    Normal lactate levels or base excess indicate adequate tissue perfusion.

     

    Hypovolemic shock leads to poor tissue perfusion due to inadequate flow or oxygenation. If a patient is in compensated shock (i.e. normal vital signs), there may be ongoing inadequate perfusion of some end-organs. Elevated lactate or a base deficit are markers of poor end-organ perfusion, thus when normalized indicate appropriate end-organ perfusion even if vital sign derangements persist.

     

    Rossaint et al. wrote a comprehensive review article in 2006 in which they discuss principles of fluid management, coagulopathy, hypothermia and tissue oxygenation in hypovolemic shock. In addition to prolonged elevated lactate levels correlating to mortality, lactate levels (or base deficits) can be used to evaluate for compensated shock in the setting of normal hemodynamic status.

     

    Illustration A shows the classification of hypovolemic shock. Note the percent of blood loss required for vital sign abnormalities.

     

    Incorrect Answers:

    Answer 1: While an arterial line is often helpful in the setting of

    uncompensated hemodynamic shock, it would not be as important as measures of adequate tissue perfusion for ruling out compensated shock. Answer 2: The hematocrit is expected to stay normal for a short time even in the setting of massive blood loss. The hematocrit only changes once the patient has physiologic or iatrogenic fluid shifts in response to the blood loss. Answer 3: Vital sign derangements indicate uncompensated shock, but do not directly measure tissue perfusion or end-organ damage

    Answer 5: Though uncommon, bleeding from isolated femur fractures can lead to Class II shock (blood loss 15-30%)

     

     

     

     

     

     

    OrthoCash 2020

     

  52. A 54-year-old woman who is an avid tennis player falls onto her dominant shoulder during a tennis match. Examination reveals tenderness and swelling in the shoulder region, but no neurovascular deficits. Radiographs and CT scan are shown in Figures A through E. Combined cortical thickness is 4.2mm. What is the most appropriate treatment option?

     

     

     

     

     

     

     

     

     

     

     

     

     

    1. Closed reduction and sling immobilization for 6 weeks

    2. Closed reduction and sling immobilization for 2 weeks followed by early active range of motion exercises

    3. Open reduction and internal fixation

    4. Hemiarthroplasty

    5. Reverse total shoulder arthroplasty Corrent answer: 3

    This patient has a 3-part proximal humerus fracture with good bone density (as evidenced by combined cortical thickness >4mm). Optimal treatment involves open reduction and internal fixation.

     

    Proximal humerus fractures are classified based on the relationship of 4 fracture fragments (greater tuberosity, lesser tuberosity, articular surface, shaft). Parts are considered separate part if displacement is > 1cm or angulation is >45°.

     

    Solberg et al. reviewed ORIF with locked plates (38 patients) vs hemiarthroplasty (48 patients). The former group had better outcome scores (especially in 3-part fractures), although there was a higher overall complication rate. ORIF of fractures with varus-extension deformity had worse outcomes than valgus-impacted fractures.

     

    Nho et al. reviewed the management of displaced proximal humerus fractures. They advocate using combined cortical thickness as a predictor of bone mineral density and potential for successful internal fixation. Cortical thickness is measured on the AP radiograph (adjusting for magnification). The average of medial and lateral cortical thickness at 2 levels is used.

    Hertel et al. reviewed predictors of humeral head ischemia. Good predictors of disrupted humeral head viability include minimal metaphyseal extension (<8mm) and medial hinge disruption (displacement >2mm) and the fracture type. Moderate and poor predictors include 4-fragment fractures, angular displacement of the head >45°, tuberosity displacement >1cm, glenohumeral dislocation, head-split, and 3-fragment fractures.

     

    Figures A through D show a 3-part proximal humerus fracture with the following views respectively: greater tuberosity, surgical neck; AP, external rotation, internal rotation, axillary views). Figure E is an axial CT scan showing displacement of the greater tuberosity that is > 1cm. Illustration A is a postop radiograph showing ORIF with an intramedullary structural allograft.

     

    Incorrect Answers

    Answers 1 and 2 : Three-part proximal humerus fractures with adequate cortical thickness (>4mm) are best treated with ORIF with a locking compression plate +/- bone graft. Non-operative management is a better option for unifocal extraarticular Type A surgical neck fractures with <66% displacement, or bifocal fractures with poor bone stock (cortical thickness

    <4mm).

    Answer 4: Hemiarthroplasty is indicated for type B (extraarticular) and C (articular) fractures with poor bone stock (cortical thickness <4mm).

    Answer 5: Reverse TSA is reserved for fractures with rotator cuff disruption (non-reconstructible tuberosities) in elderly patients.

     

     

     

     

     

    OrthoCash 2020

     

  53. A 35-year-old male presents with the post-traumatic deformity shown in Figures A and B. He has pain and difficulty walking, and deformity correction with a ring fixator is planned. When considering the principles of deformtiy surgery, it should be noted that angular corrections performed as opening or closing wedges NOT at the level of the apex of the deformity will create which of the following secondary deformities?

     

     

     

     

     

    1. Translational

    2. Angular

    3. Excessive shortening

    4. Rotational

    5. Excessive lengthening

     

    Corrent answer: 1

     

    Opening or closing wedge osteotomies NOT at the level of the apex of the deformity will create secondary translation deformities.

     

    The optimal level for an osteotomy is at the apex of an angular deformity. However, there may be circumstances in which osteotomies at the apex may not be possible, for example, if the deformity is at the level of the growth plate or joint, or there are soft-tissue coverage problems. When angular corrections are performed above or below the level of the apex of the deformity this will create secondary translational deformities in the bone. To avoid translational deformities, the osteotomy should be performed as close to the apex as possible or by using a translation hinge of bone at the osteotomy site.

     

    Paley et al. outlines the pre-operative considerations when planning the level of an osteotomy. For correction of diaphyseal deformities, a line can be drawn down the concave or convex cortex of the proximal and distal segments. The intersection of these two cortical lines is considered the true apex of deformity.

     

    Feldman et al. retrospectively reviewed a consecutive series of 18 patients that underwent corrective osteotomies. A Taylor Spatial Frame (TSF), with adjunctive bone graft as necessary, was used to achieve union and significant correction of their deformities in six axes (ie, coronal angulation and translation, sagittal angulation and translation, rotation, shortening) in 17/18 patients. They concluded that TSF's are effective in correct significant posttraumatic malunions in all axises.

     

    Figure A shows a clinical picture of the patients left leg deformity. Figure B shows a zoomed in AP radiograph of the distal tibia showing a significant posttraumatic varus malunion deformity. Illustration A shows the correction of the distal tibia deformity after osteotomy and application of a TSF using the ilizarov technique. Illustration B shows a translational deformity in the proximal tibia.

     

    Incorrect Answers:

    Answer 2: Angular deformities occur during the osteotomy stage with incorrect wedge resection or incorrect use of adjunctive bone graft.

    Answer 3: Shortening occurs with transverse or closing wedge osteotomies. Shortening is controlled with obliquely oriented osteotomies or adjunctive bone

    graft.

    Answer 4: Rotational deformities are more common with transverse osteotomies > closing/opening wedge osteotomy > oblique osteotomies. Answer 5: Excessive lengthening is not common at the site of the osteotomy.

     

     

     

     

     

     

     

    OrthoCash 2020

     

  54. A radiologist uses CT scans to perform research on rotational malalignment of femoral shaft fractures treated with intramedullary nailing. He determines the angle between a line drawn tangential to the femoral condyles and a line drawn through the axis of the femoral neck. He does this for both the injured and uninjured sides. In Figure A, what malalignment is present for the injured left side compared with the uninjured right side?

     

     

     

     

    1. Internal rotational malalignment of 13°

    2. External rotational malalignment of 13°

    3. Internal rotational malalignment of 3°

    4. External rotational malalignment of 3°

    5. No malalignment

     

    Corrent answer: 2

     

    There is external malrotation of 13°.

     

    Rotational malalignment arises after closed intramedullary nailing because anatomical reduction is achieved indirectly, resulting in less rotational control

    compared with plate fixation. Rotational malalignment can be expressed as a difference in femoral anteversion between the injured and uninjured legs.

     

    Jaarsma et al. reviewed rotational malalignment of femur fractures. They found that clinical assessment is inaccurate compared with CT measurement (±21°). Using CT measurement, they found the incidence of significant (=>15°) malrotation after IM nailing to be 20-30%. External malrotational leads to more symptoms than internal malrotation. Larger angles of malrotation (>15°) are more symptomatic. Complications of malrotation include degenerative arthritis of the hip and knee.

     

    Figure A shows change in internal rotation of -8°-5°= -13°, giving 13° of external malrotation. Illustration A shows muscular attachments that contribute to rotational malalignment. In proximal fractures, the proximal fragment rotates externally (action of the glutei, iliopsoas and hip external rotators), producing internal malrotation. In distal fractures, the distal fragment rotates externally (action of the plantaris and lateral gastrocnemius), producing external malrotation.

     

    Incorrect Answers:

    Answers 1, 3, 4, 5: CT-torsion measurements show external malrotation of

    13°.

     

     

     

     

     

    OrthoCash 2020

     

  55. A 25-year-old man is brought in with a Glasgow Coma Scale score of 3 and is intubated in the field following a motor vehicle collision. He is found to have Grade IV liver and splenic lacerations as well as an open book pelvic fracture, bilateral open tibia fractures, a closed left forearm fracture, and a left femoral shaft fracture. Which of the following variables is the most predictive of mortality?

    1. End tidal carbon dioxide = 47mmHg

    2. Hematocrit = 18.5

    3. Heart rate = 150 beats per minute

    4. Base excess = -12 mEq/L

    5. Blood pressure = 90/50 mmHg Corrent answer: 4

    Base excess (or deficit) is the most important value in determining overall resuscitation status of a polytrauma patient and will dictate initial fracture management when deciding between definitive fixation and damage control orthopaedics (DCO).

    Base deficit is synonymous with systemic lactate present which directly reflects the overall resuscitation status following trauma. Normal reference values are

    -2 to +2 mEq/L and 0.6 to 1.7 mmol/L, respectively. Excessive deficit or lactate should prompt DCO and temporary stabilization.

     

    Abramson et al. prospectively followed 76 ICU polytrauma patients and found that survival was 100% if lactate was corrected within the first 24 hours and 75% if corrected within the first 48 hours. They found that the time needed to normalize serum lactate levels also is an important prognostic factor for survival in severely injured patients.

     

    Manikis et al. prospectively followed 100 ICU patients, following serum lactate levels until normalization. Lactate levels were the most significant prognostic indicator of morbidity and mortality, and not only the absolute lactate level, but the duration of hyperlactatemia can be correlated with the development of organ failure.

    Illustration A shows a damage control orthopaedic treatment algorithm. Incorrect Answers:

    Answer 1: End tidal carbon dioxide does not predict mortality or morbidity and

    should not be the marker for resuscitation status.

    Answer 2: Hematocrit does not predict mortality or morbidity and should not be the marker for resuscitation status.

    Answer 3: Heart rate does not predict mortality or morbidity and should not be the marker for resuscitation status.

    Answer 5: Blood pressure does not predict mortality or morbidity and should not be the marker for resuscitation status.

     

     

     

     

     

    OrthoCash 2020

     

  56. A 35-year-old male patient sustains a twisting injury to his leg while playing soccer. Radiographs are seen in Figures A and B. You decide to treat this fracture with intramedullary nailing. In order to prevent a missed injury that should be addressed during the same surgery, you order the following test

     

     

     

     

     

    1. MRI of the ipsilateral knee

    2. MRI of the ipsilateral hip

    3. CT scan of the ipsilateral knee

    4. Radiographs of the ipsilateral ankle

    5. Axial radiograph of the ipsilateral calcaneus Corrent answer: 4

    This patient has spiral distal tibia and proximal fibula fractures. Dedicated imaging of the ankle should be performed to exclude a posterior malleolus fracture. Imaging options include either dedicated ankle radiographs or CT scans.

     

    Tibial shaft fractures arise from high- or low-energy injuries. Low-energy injuries are characterized by (1) torsional mechanism of injury, (2) spiral pattern, (3) fibula fracture at a different level. Surgical options for closed shaft fractures include IM nailing and plate fixation. Concomitant ankle fractures should be treated during the same surgery to improve outcome.

     

    Boraiah et al. examined the association of posterior malleolus fractures with spiral distal tibial fractures. They found that 39% had posterior malleolus fractures. In simple fractures (92%), none of the posterior malleolar fracture lines were contiguous with the diaphyseal fracture lines. In comminuted fractures (8%), an occult fracture line extended into the posterior tibial lip.

    Diagnosis was missed in 5% before CT scanning was initiated, and 0% after.

     

    Stuermer et al. examined tibial fractures with associated ankle injury. Of spiral tibial fractures, 37% extended into the ankle, 5% involved the medial malleolus, 26% involved distal fibula, 8% had syndesmotic disruption, and 16% had posterior malleolar fracture. They recommend ankle radiographs for rotational trauma, spiral distal third fractures, Maisonneuve fractures, and fractures associated with an intact fibula.

     

    Figures A and B are AP and lateral radiographs showing a spiral distal tibial fracture with a proximal fibula fracture. Illustration A is an axial CT scan showing the posterior malleolar fracture not seen on plain radiographs.

    Illustration B shows a missed posterior malleolar fragment in a fracture treated with IM nailing that subsequently displaced.

     

    Incorrect Answers:

    Answers 1, 2, 3, 5: There is no association between spiral distal tibial fractures and injuries of the hip, knee or calcaneus.

     

     

     

     

     

     

     

     

    OrthoCash 2020

     

  57. A 32-year-old professional skydiver lands awkwardly during a jump. He presents to the emergency room with bilateral knee injuries.

    Following successful closed reduction of both extremities, both feet are warm and pulses are present. Bedside doppler assessment is performed and the results are seen in Figure A. What would be the most appropriate next step in treatment?

     

     

     

     

    1. Long-leg splinting of bilateral lower extremities, monitoring of bilateral pedal pulses for 48 hours

    2. Intravenous dextran administration, repeat doppler evaluation at 6 hourly intervals

    3. Perform CT angiography for bilateral lower extremities

    4. Perform CT angiography for the left lower extremity, monitor right pedal pulses for 48 hours

    5. Surgical exploration of bilateral lower extremities Corrent answer: 4

    The ankle-brachial index (ABI) of left and right limbs are 0.78 and 0.96 respectively. CT angiography (or conventional angiography) of the left lower extremity is indicated because of abnormal ABI <0.9. It is sufficient to observe the right lower extremity (ABI >0.9).

     

    Injury to the popliteal artery is present in 10-40% of knee dislocations (KD). ABI <0.9 has sensitivity of 87% and specificity of 97% for the diagnosis of arterial disruption. Delayed recognition of an occlusive injury (>8 hours) is likely to result in above knee amputation.

     

    Stannard et al. examined the role of selective arteriography based on serial physical examination. They found arterial injury in 7% (9 out of 134 knees), and abnormal physical findings in 10 patients, with only 1 false positive. They recommend arteriography for patients with decreased pulses, color or temperature, expanding knee hematoma, or an abnormal physical examination prior to presentation in the emergency department.

     

    Mills et al. reviewed the value of ABI for diagnosing arterial injury after knee dislocation. They found that of the 29% with ABI <0.9, all required surgery for arterial injury. Of the remaining patients with ABI >0.9, none had vascular injury. They concluded ABI >0.9 has negative predictive value of 100%.

    Nicandri et al. reviewed an algorithm for selective angiography. They recommend the following: (1) Intact pulses and ABI >0.9, observation for 24 hours. (2) Asymmetric pulses or ABI <0.9, arteriogram. (3) Hard signs of vascular injury (absent distal pulses, distal ischemia, active hemorrhage, expanding pulsatile hematoma), surgical exploration.

     

    Figure A is a table showing systolic pressure readings at different sites. To calculate the ABI, the highest measured arterial pressure in the ankle or foot is divided by the higher brachial arterial pressure from both upper extremities.

     

    Incorrect Answers:

    Answer 1: If vascular exam is normal and ABI >0.9, neurovascular checks every 2-4 hours for 48 hours is appropriate. As ABI is abnormal on the left, CT angiogram is necessary.

    Answer 2: Anticoagulation has not been shown to stop thrombus formation for non-flow limiting intimal flap tears.

    Answer 3: CT angiography is indicated if ABI is abnormal post reduction. CT angiography for the right lower extremity is unnecessary as ABI >0.9.

    Answer 5: Surgery is indicated if CT angiography (or conventional angiography) demonstrates intimal flap tears that are flow limiting (but not if the intimal flap is NOT flow limiting), if distal pulses are absent, or if distal ischemia, active hemorrhage, or expanding pulsatile hematoma are present.

     

     

     

    OrthoCash 2020

     

  58. A previously healthy 22-year-old male presents to the hospital after a motor vehicle accident. His injuries include a closed head injury, flail chest, intra-abdominal bleed and right femoral shaft fracture. Which of the following conventional indicators would support the role for "damage control orthopaedics" as opposed to "early total care" in the clinical decision making process of his femur fracture management?

    1. Lactate level = 1.9 mmol/L

    2. Fibrinogen = 1.1 g/dL

    3. Platelet count = 20,000 per mcL

    4. Urine output = 50 cc/hr

    5. Base deficit = 2 mmol/L

     

    Corrent answer: 3

     

    Previously healthy, poly-trauma patients, presenting with platelet counts of

    <70,000 will fall into the pathophysiological category of 'in extremis'. This will

    support the role of damage control orthopaedics in the decision making process of this patients fracture management.

     

    Damage control orthopaedics (DCO) is a staged approach for the management of polytrauma patients. It is most ideal for trauma patients that are clinically unstable or in extremis. In these patients, immediate surgery is thought to cause a “second hit” phenomenon, which may lead to ARDS, multi-organ failure, or even death. A patient is classified as 'unstable' or 'in extremis', if he or she meets the criteria in at least three of the four pathophysiological parameters; blood pressure <90mmHg, platelets count <70,000, temperature

    <32°C and major soft tissue injuries.

     

    Pape et al. (2005) described four classes of patients, based on their clinical status: stable, borderline, unstable, and in extremes. The term “borderline” was coined to describe a patient who is categorized as stable before surgery, but is at significant risk of unexpected deterioration and organ dysfunction postoperatively.

     

    Pape et al. (2009) outlined that stable patient can undergo early definitive fracture fixation as necessary. In contrast, unstable patient should be resuscitated and adequately stabilized with temporary fixation before receiving definitive orthopaedic care.

     

    Illustration A shows a table outlining the classification system used by Pape to classify patients into their clinical status of stable, borderline, unstable, and in extremes.

     

    Incorrect Answers:

    Answer 1: Lactate level = 1.9 mmol/L (normal range <2.5 mmol/L) would classify this patient into a stable category.

    Answer 2: Fibrinogen = 1.1 g/dL (normal range >1 g/dL) would classify this patient into a stable category.

    Answer 4: Urine output = 50 cc/hr (normal range >150 cc/hr)would classify this patient into a borderline category.

    Answer 5: Base deficit = 2 mmol/L (normal range -2 to +2 mmol/L) would classify this patient into a stable category.

     

     

     

     

     

    OrthoCash 2020

     

  59. A 27-year-old man falls off of a bicycle at a high speed and is taken to the emergency department, where he is diagnosed with an acetabular fracture. Which of the following image sets accurately depicts a fracture that involves only one of the two columns of the acetabulum?

     

     

     

     

     

     

     

     

     

     

     

     

     

     

    1. Figure A

    2. Figure B

    3. Figure C

    4. Figure D

    5. Figure E

     

    Corrent answer: 4

     

    Figure D represents an associated posterior column and posterior wall fracture while the other four choices are consist of fractures that incorporate both the anterior and posterior columns.

     

    The transverse family of acetabulum fractures includes the elementary transverse pattern as well as associated transverse and posterior wall and T-type patterns. All transverse fractures disrupt both structural columns of the acetabulum by separating the hemipelvis from cranial to caudal. Of note, this is represented on axial CT cuts by a vertically-oriented fracture line.

    Conversely, columnar fracture patterns can include either one or both of the columns, and they separate the anterior and/or posterior column from the rest of the hemipelvis. In contrast to transverse fracture lines, this is represented on axial CT cuts by a horizontal fracture line, but is often best seen on judet views or 3d CT scans.

     

    Matta reported on his outcomes of 263 displaced acetabulum fractures treated with open reduction and internal fixation within 21 days of injury. He reported excellent or good function in 76% of patients, which correlated well to quality of reduction of the joint surface seen on follow up radiographs.

     

    Borrelli et al. reported that computer-reconstructed radiographs are equal to standard pelvic radiographs for assessing acetabulum fracture patterns.

     

    Beaulé, Dorey, and Matta analyzed the interobserver reliability of Letournel's classification system using both pelvic radiographs and CT imaging. They found that while CT scans add valuable information regarding articular impaction and intraarticular loose bodies, CT data did not significantly increase the interobserver reliability of Letournel's classification.

     

    The associated posterior column and posterior wall fracture depicted in Figure D displays an intact anterior column on both x-ray and axial CT. The axial CT also illustrates the horizontal nature of the posterior column fracture line.

     

    Illustration A shows the 5 elementary and 5 associated fracture patterns as described by Letournel.

     

    Incorrect Answers:

    Answer 1: Figure A shows reconstructed CT scans with a vertical fracture line clearly in the iliac wing which is part of the anterior column. The CT also shows involvement of the posterior column with a transverse fracture line. The X-ray demonstrates a "spur sign", which is pathonemonic for an associated both column fracture.

     

    Answer 2: Figure B X-ray sows disruption of both the Ilio-pectineal and Ilio-ischial lines which demonstrates disruption of the anterior and posterior columns respectively. The CT cut show a vertically oriented fracture line consistent with a transverse fracture. The reconstructed CT confirms the transverse fracture with addition of a posterior wall fracture.

     

    Answer 3: Figure C, the AP pelvis X-ray shows a subtle line extending into both the Ilio-pectineal and Ilio-ischial lines. Along with a CT cut showing a vertically oriented fracture line consistent with a transverse fracture as well as a

    separate posterior wall fragment.

     

    Answer 5: Figure E shows an obturator oblique Judet view, which show disruption of the ilio-pectinial line (anterior column) with the CT again showing a vertically oriented fracture line consistent with a transverse fracture as well as a separate posterior wall fragment.

     

     

     

     

     

     

    OrthoCash 2020

     

  60. A young gymnast fell awkwardly onto an outstretched hand during a competition. At the time of impact, his forearm was positioned in supination. Axial and posterolateral forces were loaded along the forearm into the elbow and the elbow underwent a significant valgus thrust. What injury pattern is most likely to result from the combination of these forces at the elbow?

    1. Extension-type supracondylar fracture

    2. Flexion-type supracondylar fracture

    3. Anterior olecranon fracture dislocation

    4. Coronoid fracture, olecranon fracture and elbow dislocation

    5. Coronoid fracture, radial head fracture and elbow dislocation Corrent answer: 5

    The combination of valgus, axial, and posterolateral rotatory forces (forearm supination) can result in a "terrible triad" injury of the elbow.

     

    The key features of a terrible triad injury include a radial head fracture, coronoid fracture, and dislocation of the elbow. Disruption of the lateral collateral ligament complex often concomitantly occurs. While restoration of the bony anatomy is important for static stability, the key primary stabilizer that needs to be addressed is the lateral collateral ligament complex. In acute

    injuries LCL repair may be possible. In chronic injury, LCL reconstruction would need to be considered.

     

    O'Driscoll et al. 1991, examined 5 patients with recurrent posterolateral rotatory instability of the elbow. They showed that by applying supination of the forearm with a valgus moment and an axial compression force to the elbow while it is flexed from full extension, this can demonstrate posterolateral rotatory instability of the elbow. The elbow is reduced in full extension and must be subluxated as it is flexed in order to obtain a positive test result (a sudden reduction of the subluxation).

     

    O'Driscoll et al. 1992 looked at a cadaveric study of the elbow. They showed that external rotation and valgus moments with axial forces resulted in posterior dislocations in 12 of the 13 specimens when the anterior medial collateral ligament (AMCL) remained intact. Clinically, it valgus stability in pronation is demonstrated, the AMCL can be assumed to be intact.

     

    Illustration A and B shows radiographs of a terrible triad injury. There is posterolateral dislocation of the elbow with associated radial head fracture, coronoid fracture.

     

    Incorrect Answers:

    Answer A: Extension-type supracondylar fractures typically occur with FOOSH injuries with posteriorly directed forces.

    Answer B: Flexion-type supracondylar fractures typically occur with the elbow in flexion with an anterior directed force.

    Answer C: Anterior olecranon fracture dislocation typically occurs with a direct high-energy blow that is applied to the posterior aspect of the forearm with the elbow in 90° flexion.

    Answer D: Coronoid fracture, olecranon fracture and lateral collateral ligament injury typically occur with the arm positioned in supination, axial and VARUS moment forces.

     

     

     

     

     

     

     

    OrthoCash 2020

     

  61. A 70-year-old female presents with right thigh ache for 6 months. Except for a history of osteoporosis, she is otherwise healthy. She has been on antiresorptive therapy for 8 years. Her radiograph is shown in Figure A. Four months later, she trips over a rug and falls, sustaining the injury shown in Figures B and C. Which of these statements is TRUE regarding surgical fixation of this fracture compared with conventional fractures?

     

     

     

     

     

     

     

     

    1. There is an increased risk of revision surgery with plate fixation.

    2. There is an increased risk of iatrogenic fracture with plate fixation.

    3. There is an increased risk of non-union with plate fixation.

    4. There is an increased risk of iatrogenic fracture with intramedullary nail fixation.

    5. There is an increased risk of hardware failure with nail fixation.

     

    Corrent answer: 4

     

    This patient has a bisphosphonate-related (BP) fracture. There is increased risk of iatrogenic fracture with IM nailing of this fracture.

     

    Subtrochanteric fractures are fractures extending from the lesser trochanter to 5cm distal to it. BP fractures are characterized by (1) focal lateral cortical thickening, (2) transverse fracture orientation, (3) medial spike and (4) lack of comminution. There is increased risk of iatrogenic fracture with IM nailing because BP fractures have thickened, brittle cortices and the mismatch between medullary diameter and increased proximal nail diameter results in iatrogenic fracture.

     

    Weil et al. reviewed the outcome of surgically treated bisphosphonate fractures. IM nailing resulted in healing in 54% of bisphosphonate fractures

    with 98-99% of conventional fractures. In their study, 46% required revision procedures.

     

    Prasarn et al. compared plate and nail treated bisphosphonate fractures with conventional fractures. They found that the bisphosphonate group had more major and minor complications (68%) than the conventional group (10%). The most common complications were intraoperative femoral shaft comminution (nail) and hardware failure (plate).

     

    Figure A shows diffuse cortical thickening with an antero-lateral cortical ridge. Figure B shows a transverse subtrochanteric fracture extending through the middle of the cortical ridge seen previously. Note also healed fractures of the left superior and inferior rami. Figure C is a post-reduction radiograph showing the transverse fracture through the beak-shaped region of the previous insufficiency fracture. Illustration A shows a typical bisphosphonate-related fracture with transverse fracture orientation, focal lateral cortical thickening (white arrows), medial beak (black arrow), and lack of comminution.

    Illustration B shows a conventional subtrochanteric fracture. Illustration C shows intraoperative iatrogenic fracture with anterolateral comminution during nailing. Illustration D shows fixation with a proximal femoral hook LCP Plate with late hardware failure at 3 months.

     

    Incorrect Answers:

    Answers 1, 3: Increased non-union after IM nailing has been observed, necessitating more revision surgery compared with conventional fractures. Revision surgery includes nail dynamization, exchange nailing, and conversion to plating. These procedures are more frequently performed for osteoporotic fractures.

    Answer 2: Compared with conventional fractures, there is no increased risk of iatrogenic fracture with plating.

    Answer 5: There is an increased risk of plate hardware failure because of because of a varus moment arm and dependence on intramembranous healing inhibited by bisphosphonates.

     

     

     

     

     

     

     

     

     

     

     

     

     

    OrthoCash 2020

     

  62. A 58-year-old African-American female who sustained an injury to her upper arm six months ago presents with persistent arm pain. She was initially treated with splinting, with conversion to fracture bracing. She is neurovascularly intact. An injury radiograph and a current radiograph are shown in Figures A and B respectively. What nutritional or metabolic disturbance is the most likely associated with this patient's diagnosis?

     

     

     

     

     

     

    1. Vitamin A deficiency

    2. Low serum testosterone

    3. Low serum thyroxine

    4. Vitamin D deficiency

    5. Hypocalcemia

     

    Corrent answer: 4

     

    This patient has sustained a humeral diaphyseal fracture that has gone on to an atrophic nonunion. Vitamin D deficiency is the most likely associated metabolic disturbance.

     

    The incidence of nonunion with non-operative management of humeral shaft injuries ranges from 2-10%. Risk factors include vitamin D deficiency (most common), open fractures, segmental injuries, smoking and obesity. The optimal treatment is compression plating with bone grafting, which has been shown to be superior to intramedullary nailing with bone grafting or compression plating alone.

     

    Ring et al. reviewed factors that contributed to humeral diaphyseal nonunion after fracture bracing. Fractures in the proximal to middle one-third of the shaft or fractures with a spiral/oblique pattern were more likely to go on to nonunion.

     

    Brinker et al. reviewed 37 low-energy fractures that went onto nonunion. These patients were evaluated by clinical endocrinologists for evaluation of metabolic abnormalities. Thirty-one of the 37 patients (84%) had a metabolic issue, with 68% (25 of 37 patients) having Vitamin D deficiency.

     

    Figure A demonstrates a humeral shaft fracture. Figure B demonstrates an atrophic nonunion of the humeral shaft fracture.

     

    Incorrect Answers:

    Answers 1, 2, 3,5: Deficiencies of these options have not been associated with nonunion

     

     

     

    OrthoCash 2020

     

  63. A 27-year-old male motorcyclist suffers a crash sustaining an isolated right distal humerus fracture. He was treated non-operatively. Ten months later, he returns complaining of limited range of motion and continued pain. Physical examination reveals range of motion of 30-90 degrees on the right and 0-130 degrees on the left. Imaging of his elbow is shown in Figure A and B. What is the most appropriate treatment to improve flexion?

     

     

     

     

     

     

    1. Continue therapy

    2. Indomethacin

    3. Radiation therapy

    4. Heterotopic ossification excision with release of the posterior band of the ulnar collateral ligament

    5. Heterotopic ossification excision with release of the anterior band of the ulnar collateral ligament

    Corrent answer: 4

     

    This patient has elbow stiffness caused by posteromedial heterotopic ossification after healing of his distal humerus fracture.

     

    Stiffness and limited range of motion is common secondary to extrinsic causes. Studies have shown operative treatment via HO excision, capsular release, and/or release of the posterior band of the UCL can improve range of motion.

     

    Park et al. studied 42 patients with extrinsic contracture of less than 100 degrees. They report posteromedial HO excision with capsular release and release of the posterior band of the UCL significantly improved ROM.

     

    Williams et al. studied 164 patients who underwent release for extrinsic

    tightness with ulnar nerve decompression for preoperative symptoms. Following decompression, low rates of postoperative ulnar nerve symptoms were noted. More post-operative symptoms were seen for more severe contractures.

     

    Figure A is an AP and lateral radiograph of the elbow with posteromedial heterotopic ossification. Figure B is a 3D CT reconstruction confirming the posteromedial location.

     

    Incorrect Answers:

    Answer 1. There is a mechanical block to motion. Continued therapy will not improve the patient's range of motion.

    Answer 2. Indomethacin is used for heterotopic ossification prophylaxis. Answer 3. Radiation therapy is used for heterotopic ossification prophylaxis. Answer 5. The anterior band of the ulnar collateral ligament should not be released because it will cause secondary valgus instability.

     

     

     

    OrthoCash 2020

     

  64. A 72-year-old woman falls down the stairs and is now unable to bear weight secondary to right groin pain. Injury radiograph and CT scans are seen in Figures A through C. What is the Young-Burgess classification of this injury and the most appropriate treatment plan?

     

     

     

     

     

     

     

     

    1. Bilateral weight bearing as tolerated for Anterior Posterior Compression Type I injury

    2. Touchdown weight bearing on the right for Lateral Compression Type I injury

    3. Bilateral weight bearing as tolerated for Lateral Compression Type I injury

    4. Posterior sacroiliac screw, followed by non-weight bearing for Lateral Compression Type II injury

    5. Posterior sacral plate, followed by non-weight bearing for Lateral Compression Type III injury

    Corrent answer: 3

     

    This patient has a Young-Burgess lateral compression (LC) I injury, with a simple buckle of the anterior sacral cortex without posterior cortex involvement (incomplete). This is a stable injury that may be treated non-operatively with weight bearing as tolerated

     

    Lateral compression injuries are internal rotation injuries. Stable type I injuries may be treated with weight bearing as tolerated, an especially important consideration in elderly patients to avoid the complications of bedrest. Rami fractures in Type II and III are usually treated non-operatively, but may often times require operative fixation.

     

    Sembler Soles et al. retrospectively reviewed 118 LC fractures with <10mm of sacral displacement. They found that only 1 patient failed nonoperative treatment and had 5mm of additional displacement, necessitating surgical

    fixation. They concluded that nonoperative immediate weightbearing is a safe and acceptable treatment for LC I injuries.

     

    Lefaivre et al. examined different types of LC I fractures. They classified sacral fractures as buckle, simple and comminuted. They found that comminuted and complete sacral fractures were more likely to be symptomatic and require posterior ring stabilization.

     

    Figure A is an AP radiograph of the pelvis. There is an nondisplaced fracture of the right superior ramus, and the sacral fracture line is hardly visible. Figures B and C are axial CT scans at different levels showing an nondisplaced simple, incomplete (not extending to posterior cortex) anterior buckle fracture of the sacrum and confirms the superior ramus fracture. Illustration A shows the Young-Burgess classification.

     

    Incorrect Answers:

    Answer 1: This is an LC I injury.

    Answer 2: For stable LC I injuries, immediate weight bearing as tolerated has low risk of additional displacement.

    Answer 4: This is an LC I injury. Posterior percutaneous SI screws are an option for LC II injuries.

    Answer 5: This is an LC I injury. Posterior sacral plate (or SI screws) are an option for LC III injuries.

     

     

     

     

     

     

    OrthoCash 2020

     

  65. A 70-year-old patient with a history of Parkinson’s disease sustains a fall onto his hip. He denies a history of antecedent hip pain

    and is otherwise healthy. A radiograph of the affected hip is shown in Figure A. What is the best treatment option and best rationale for this patient?

     

     

     

     

    1. Total hip arthroplasty; decrease his risk for dislocations

    2. Total hip arthroplasty; decrease his risk for infection

    3. Total hip arthroplasty; use a minimally invasive approach

    4. Hip hemiarthroplasty; decrease his risk for dislocations

    5. Hip hemiarthroplasty; decrease his risk for infection Corrent answer: 4

    This patient has sustained a Garden IV femoral neck fracture. The optimal treatment for this patient is a hip hemiarthroplasty to limit his risk for dislocation.

     

    Displaced femoral neck fractures are often treated with arthroplasty. Although functional outcomes are better with total hip arthroplasty (THA), the risk of dislocation is seven times higher. Patients who are at risk for falls and/or demonstrate cognitive decline (ie. Parkinson’s disease), should be treated with a hemiarthroplasty.

     

    Lee et al. reviewed 126 cases of THA used to treat acute femoral neck fractures. They noted long-term survival of the prosthesis and good clinical results despite a higher rate of complications.

     

    Ricci et al. reviewed 29 THAs performed in 26 patients who did not have Parkinson’s disease or other evidence of cognitive/ambulatory dysfunction. Factors associated with low dislocation rates include selecting the right patients

    and intraoperative technique focused on stability of the hip.

     

    Zuckerman et al. review the diagnosis and orthopaedic management of the Parkinson’s disease patient. They report for patients with femoral neck fractures, use of hip hemiarthroplasty has improved their ambulatory capacity. In contrast, complication rates following total hip arthroplasty approach 26%.

     

    Figure A shows an AP radiograph of the pelvis. There is a Garden IV fracture of the femoral neck of the right hip. On the left hip, notice thinning of the medial wall of the acetabulum, with loss of sphericity of the femoral head and significant protrusion of the hip.

     

    Incorrect Answers:

    Answer 1: THA would increase his risk for dislocations.

    Answers 2, 5: Implant placement would still put the patient at risk for infection.

    Answer 3: Although a minimally invasive choice could be taken, a THA is not appropriate for this patient.

     

     

     

    OrthoCash 2020

     

  66. A 27-year-old man presents to the emergency department with an ankle fracture. CT scans note anteromedial marginal impaction.

    Which radiograph (Figures A-E) would best correlate with this finding?

     

     

     

     

     

     

     

     

     

     

     

     

     

    1. Figure A

    2. Figure B

    3. Figure C

    4. Figure D

    5. Figure E

     

    Corrent answer: 4

     

    This patient has anteromedial marginal impaction seen on CT scans. This is characteristic of a supination-adduction ankle fracture (Figure D).

     

    In the Lauge-Hansen classification, ankle fractures are classified into supination-adduction (SA), supination-external rotation, pronation-abduction, pronation-external rotation. Certain considerations exist when treating SA fractures with marginal impaction: (1) An anteromedial approach will aid in visualizing and reducing the impaction (instead of standard medial approach).

    (2) Disimpaction of the articular fragment and possibly bone grafting of the resulting defect may be necessary rather than simple percutaneous fixation.

     

    McConnell et al. reviewed marginal plafond impaction in supination adduction injuries. Supination-adduction injuries comprised 5% of 500 fractures (44 fractures), and 42% (8 fractures) of these had marginal impaction.

     

    Illustration A is another example of a SA fracture depicting vertical fracture of the medial malleolus in association with marginal impaction of the plafond

    (arrow). Illustration B is an axial CT showing increased density at the level of the subchondral bone, characteristic of anteromedial marginal impaction.

    Illustration C is a coronal CT showing showing articular depression of the impacted segment and tibiotalar incongruity.

     

    Incorrect Answers:

    Answer 1: Figure A shows a pronation-abduction ankle fracture. Answer 2: Figure B shows a talar body fracture.

    Answer 3: Figure C shows a pronation-external rotation ankle fracture Answer 5: Figure E shows a supination-external rotation ankle fracture. Marginal impaction is not characteristic of these injuries.

     

     

     

     

     

     

     

     

     

     

    OrthoCash 2020

     

  67. A 25-year-old man sustains a twisting injury to his ankle. His radiograph is shown in Figure A. Which of the following is the most appropriate method to assess the competency of his deltoid ligament?

     

     

     

     

    1. Anterior drawer test

    2. External rotation stress radiograph

    3. Internal rotation stress radiograph

    4. Palpation of the medial ankle in the region of the deltoid

    5. Inversion stress radiograph Corrent answer: 2

    The clinical presentation is consistent with a supination external rotation (SER) injury. An external rotation stress radiograph is the most appropriate method to assess the competency of the deltoid ligament.

     

    Plain static radiographs alone can not be relied on to detect deltoid injury, which is one variable used to assess ankle stability. Various preoperative methods have been suggested, including the presence of medial tenderness, allowing the patient to weight bear with follow up radiographs, manual ER

    stress radiographs, and gravity ER stress radiographs. Evidence shows both a manual or gravity external rotation stress radiograph can indicate deltoid competency, although this isn't absolute.

     

    Park et al. performed a cadaveric study of SER ankle injuries and found that a medial clear space of > or =5 mm on radiographs taken in dorsiflexion with an external rotational stress was most predictive of deep deltoid ligament transection after distal fibular fracture. All other stress conditions and increases in medial clear space of 2 or 3 mm were less predictive.

     

    DeAngelis et al. performed a study to identify whether medial tenderness is a predictor of deep deltoid ligament incompetence in supination-external rotation ankle fractures. The identified 55 Weber B type fractures, and sought to correlate medial tenderness with positive stress radiographs. They found no statistical significance between the presence of medial tenderness and deep deltoid ligament incompetence.

     

    Figure A shows a a spiral fibula fracture at the level of the tibial plafond, consistent with a supination external rotation (SER) injury. Illustration A shows the Lauge Hansen classification of supination-external rotation ankle injuries (SER 1-4). Illustration B shows an external rotation stress with medial clear space widening, indicating incompetent deltoid ligament. Illustration C shows a ankle gravity stress test where the patient lies in the lateral position and the ankle is allowed to hang down freely over a support. Finally, Illustration D shows an intra-operative example of the Cotton test, which involves direct lateral stress on the fibula with use of a clamp.

     

    Incorrect Answers:

    Answer 1: The anterior drawer assesses the anterior talofibular ligament and is a physical exam test that can be done pre/intra/postoperatively.

    Answer 4: The lateral stress (Cotton) test is used intraoperatively to assess for syndesmotic injury (Illustration D). This test involves manual/clamp distraction of the fibula away from the tibia under fluoroscopy after stabilization of the lateral and medial malleoli.

     

     

     

     

     

     

     

     

     

     

    OrthoCash 2020

     

  68. An 80-year-old patient presents 8 months postoperatively with right groin pain. Examination reveals a leg length discrepancy of

    1.5cm. Recent radiographs are seen in Figures A and B. What is the most appropriate treatment plan?

     

     

     

     

     

    1. Nail dynamization

    2. Hardware removal, correction of alignment with a Taylor spatial frame, insertion of bone autograft

    3. Exchange unreamed nailing with a longer, larger implant

    4. Augmentative plate fixation without nail removal, insertion of bone autograft

    5. Hardware removal, correction of alignment, plate fixation, insertion of bone autograft

    Corrent answer: 5

     

    This patient has atrophic non-union (NU) and varus collapse following cephalomedullary nailing of a subtrochanteric fracture. The ideal treatment involves nail removal, correction of alignment, fracture fixation, and bone grafting. Fixation can be achieved with a nail or plate.

     

    Subtrochanteric fractures can be treated with cephalomedullary nailing or fixed angle plates. Nailing of these fractures is technically challenging because the fracture must be reduced prior to nail passage. Failure to do so leads to varus and procurvatum malreduction.

     

    Bellabarba et al. reviewed plating of femoral nonunions after intramedullary nailing. Of 23 nonunions, 21 healed at an average of 12 weeks. The remaining 2 cases required repeat plating (at 2 and 8 weeks) for hardware breakage because of noncompliance with weightbearing restrictions. They advocate plating because it allows for correction of malalignment and provides a biomechanically superior tension band construct.

     

    Incorrect Answers:

    Answer 1:Dynamization is less effective in femoral NU compared with tibial NU. It is suitable for NU of axially stable fractures without significant angular deformity, particularly if statically locked in distraction. It is not suitable in this case because of varus collapse and shortening.

    Answer 2: The use of tensioned wire fixators is poorly tolerated in the femur and has a high complication rate.

    Answer 3: Reamed exchange nailing is the procedure of choice after IM nailing in the ABSENCE of femoral deformity. It is less effective in this case of varus collapse and shortening. Exchange unreamed nailing is not useful because (1) the new nail follows the same path as the old, making deformity correction impossible, and (2) the new nail cannot be larger than the old nail. A larger implant is only possible following reaming, especially if the isthmus is to be crossed.

    Answer 4: Plating around a nail is inadvisable in this case because the existing hardware precludes deformity correction.

     

     

    OrthoCash 2020

     

  69. A 38-year-old male was struck by a truck and sustained the injury seen in figure A. Treating this injury with an intramedullary nail with a larger radius of curvature can lead to what complication?

     

     

     

     

    1. Posterior perforation of the distal femur

    2. Varus malreduction

    3. Comminution of the fracture site

    4. Iatrogenic femoral neck fracture

    5. Anterior perforation of the distal femur Corrent answer: 5

    According to the study by Egol et al, the average femoral anterior radius of curvature was 120 cm (+/- 36 cm), and currently available femoral nails have a greater radius of curvature (i.e. more straight). This mismatch has been shown to lead to an increased risk of perforation of the anterior distal femur as the nail is impacted into the canal.

     

    The referenced study by Tencer et al noted an increased risk of iatrogenic femoral fracture with anterior starting point >6mm from the anatomic axis.

    They recommend starting in line with the femoral axis, or just a few millimeters anterior in order to minimize this risk.

     

    Illustration A depicts anterior femoral cortex penetration secondary to nail/femur radius of curvature mismatch.

     

     

     

     

     

     

    OrthoCash 2020

     

  70. A 60-year-old woman is undergoing closed reduction and percutaneous pinning of a proximal humerus fracture. What structure is at greatest risk for injury from the pin marked by the red arrow in Figure A?

     

     

     

     

    1. Anterior branch of the axillary nerve

    2. Posterior humeral circumflex artery

    3. Long head of the biceps tendon

    4. Cephalic vein

    5. Musculocutaneous nerve

     

    Corrent answer: 1

    Certain anatomic structures are at risk with percutaneous pinning of proximal humerus fractures. The red arrow in Figure A marks a proximal lateral pin that would place the anterior branch of the axillary nerve at risk.

     

    Rowles and McGrory performed an anatomic study of the structures at risk with closed reduction and percuatneous pinning of the proximal humerus and found that proximal lateral pins were a mean of 3mm from the anterior branch of the axillary nerve. Pins placed through the anterior cortex and directed into the humeral head fragment were a mean of 2mm from the long head of the biceps tendon and greater tuberosity pins were found to be 8mm from the posterior humeral circumflex and 10mm from the main trunk of the axillary nerve as they penetrated the medial cortex of the humerus.

     

    Jaberg et al retrospectively reviewed the clinical and radiographic results of 48 patients at an average of 3 years after undergoing closed reduction and percutaneous pinning of a proximal humerus fracture. 70% good to excellent results with their described technique, and the authors caution that radiographic malunion did not correlate with patient function.

     

    Incorrect Answers

    Answer 2: The posterior humeral circumflex artery (PHCA) is at risk from a greater tuberosity pin as it penetrates the medial cortex. In the anatomic study by Rowles and McGrory, the distance between the pin and the PHCA/axillary nerve was decreased with internal rotation and increased with external rotation.

    Answer 3: The long head of the biceps tendon would be at risk from a pin placed through the anterior cortex and directed into the humeral head and was perforated by three pins in the study by Rowles and McGrory.

    Answer 4: The cephalic vein is at risk with anterior pins (mean of 11mm in the referenced study)

    Answer 5: The musculocutaneous nerve is at risk with anterior pins (mean of 17mm in the referenced study)

     

     

     

     

     

    OrthoCash 2020

     

  71. A 42-year-old female sustains the injury seen in the computed tomography images seen in Figures A and B. According to the Letournel classification, what is the injury pattern shown?

     

     

     

     

     

     

    1. Posterior wall

    2. Transverse

    3. Anterior wall

    4. Posterior column

    5. Both column

     

    Corrent answer: 2

    The axial CT cut and Judet radiographic view shown reveals a transverse fracture pattern according to the Letournel classification system. This can be determined by the fact that the articular surface of the acetabulum is attached to the intact portion of the ilium, which is connected to the axial skeleton posteriorly through the sacroiliac joint. This differs from a both-column fracture, in which the articular surface of the acetabulum has no attachments to the axial skeleton due to fracture line(s). The axial CT scan also shows a vertical fracture line which is typical of a transverse fracture pattern.

     

    Durkee et al review the classification schemes for these injuries, as well as comment on the importance of quality images (Judet views, CT, etc).

     

    Figures A and B show a transverse acetabular fracture with mild displacement.

     

     

     

     

     

     

    OrthoCash 2020

     

  72. Which of the following is true regarding plating of humeral shaft fractures compared to intramedullary nailing?

    1. worse functional results

    2. higher need for subsequent surgeries

    3. higher incidence of radial nerve injury

    4. lower complication rates

    5. decreased nonunion rates Corrent answer: 4

    Controversy exists regarding nailing compared with plating of humeral shaft fractures, but the most recent and highest level evidence indicates decreased complication rates with open reduction and internal fixation of these injuries.

     

    Lin et al found less blood loss with intramedullary nailing than plating, but nailing was also associated with increased shoulder surgery, likely due to disruption of the rotator cuff tendon during insertion.

    Meekers et al found a higher union rate, better functional results and a lower reoperation rate after plate and screw fixation versus nailing. They concluded that plating was superior in most cases of humeral shaft fracture, however more recent studies have challenged these findings.

     

    Heineman et al. (2012) have recently conducted an update on their meta-analysis to include more recent randomized studies. With the inclusion of newer studies the author found a statistically significant increase in total complication rate with the use of IM nailing compared with ORIF. The authors found no significant difference between the two treatment modalities for the secondary outcomes (nonunion, infection, nerve palsy, re-operation)

     

    Incorrect Answers:

    Answer 1: Although prior level 4 studies indicated better functional outcomes with ORIF, more recent studies and pooled analyses have not shown superior functional outcomes with either treatment

    Answer 2: A trend towards lower need for subsequent surgery is seen with ORIF, likely secondary to increased complications seen with IM nailing secondary to implant prominence and shoulder dysfunction

    Answer 3: No difference in radial nerve injury is seen between the two treatments

    Answer 5: Pooled analysis of the existing literature has demonstrated no difference in union rates between ORIF and IM nailing of humeral shaft fractures

     

     

     

    OrthoCash 2020

     

  73. A 35-year-old male sustains the fracture seen in Figures A and B. Which of the following substances has been shown to result in the least radiographic subsidence when combined with open reduction and internal fixation?

     

     

     

     

     

     

    1. Cancellous allograft bone chips

    2. Autograft iliac crest

    3. Femoral intramedullary reamings

    4. Calcium phosphate cement

    5. Calcium sulfate cement Corrent answer: 4

    Figures A and B show a plateau fracture with a lateral split and depression of the articular surface. In treating tibial plateau fractures, calcium phosphate has been shown to have the least amount of articular subsidence on follow-up examinations due to its high compressive strength.

     

    The study by Lobenhoffer et al noted improved radiographic outcomes and earlier weightbearing with usage of calcium phosphate cement. Welch and Zhang reproduced tibial plateau fractures in goats and compared cancellous autograft to calcium phosphate cement augmentation. At 24 hours, four of five specimens treated with autograft had subsidence of the fragment. Only two specimens from limbs treated with cement showed minimal subsidence; the remaining were congruent.

     

    Yetkinler’s study compared cement to no cement treatment in a model of depressed plateau fractures. Calcium phosphate cement of high compressive

    strength provided equivalent or better stability than conventional open reduction

    and internal fixation with either auto/allograft bone which had both a lower compressive strength and reduced mechanical stability.

     

     

     

    OrthoCash 2020

     

  74. The modified Judet approach to the posterior scapula exploits the internervous interval between what two muscles?

    1. Supraspinatus and infraspinatus

    2. Supraspinatus and subscapularis

    3. Infraspinatus and teres minor

    4. Teres minor and teres major

    5. Teres major and lattisimus Corrent answer: 3

    The posterior or modified Judet approach to the scapula is typically used for internal fixation of scapular fractures. This approach utilizes a transverse incision over the scapular spine with detachment of the posterior deltoid. The interval between the infraspinatus (suprascapular n.) and teres minor (axillary n.) is identified and used to gain access to the posterior aspect of the scapula and glenoid.

     

    The reference by Obremskey et al argues the approach "combines several important goals including: 1) exposure of all bony elements of the scapula which have adequate bone stock for internal fixation; 2) minimal trauma to the rotator cuff musculature; and 3) protection of the major neurologic structures (suprascapular nerve superiorly and axillary nerve laterally)." They believe "the main advantage of the exposure is limiting muscular dissection, which can potentially improve rehabilitation and limit morbidity of the operation."

     

     

     

    OrthoCash 2020

     

  75. An 82-year-old female sustains an intertrochanteric hip fracture and is treated with a sliding hip screw. What is the most appropriate definitive step in treating the failure seen in figure A?

     

     

     

    1. Non-weight bearing

    2. Valgus proximal femoral osteotomy

    3. Total hip arthroplasty

    4. Revision open reduction and internal fixation

    5. Proximal femoral resection Corrent answer: 3

    Figure A shows superior cutout of the lag screw from the sliding hip screw as well as the superior cannulated screw used for an "antirotation" device.

     

    In the referenced review article by Haidukewych and Berry, salvage of failed treatment of hip fractures in the elderly is limited by bone quality and comorbidities. They recommend total hip arthroplasty in this instance to restore function, decrease pain, and limit periods of immobilization. They mention that the major challenges for arthroplasty are: assessing the need for acetabular resurfacing, selecting the femoral implant, and managing the greater trochanter.

     

     

     

    OrthoCash 2020

     

  76. A 13-year-old boy falls out of a tree and sustains the injury seen in Figures A and B. He is taken to the OR for fixation of his fracture.

    The next morning, the patient’s blood pressure is 185/105 mm Hg and pulse rate is 130. He complains of pain that is not improved with opiates. On physical exam, the foot is firm. The decision is made to obtain compartment pressures to rule out compartment syndrome of the foot. Which of the following paths in Figure C marks the

    appropriate location to measure the central compartment, and what would be considered abnormal values?

     

     

     

     

     

     

     

     

     

     

    1. Path A, absolute value of 30-45 mmHg or delta p > 30mmHg

    2. Path B, absolute value of 30-45 mmHg or delta p > 30mmHg

    3. Path B, absolute value of 30-45 mmHg or delta p < 30mmHg

    4. Path C, absolute value of 30-45 mmHg or delta p > 30mmHg

    5. Path C, absolute value of 30-45 mmHg or delta p < 30mmHg Corrent answer: 3

    The correct approach to measure pressures in the central compartment of the foot is by directing the needle lateral and plantar through the abductor hallicus, just under the base of the first metatarsal. Abnormal values indicating the need for decompression are an absolute value of 30-45 mmHg or a Δp < 30mmHg (the difference between the patient's diastolic blood pressure and compartment pressures).

     

    The most common symptom of compartment syndrome in the extremities is intense pain. However, compartment syndrome can be difficult to diagnose in children and patients who are comatose, nonverbal, and/or mentally compromised because they may not be able to properly express their level of pain. Additionally, in compartment syndrome of the foot, pain on passive extension of the toes may or may not be present, and swelling and absence of the dorsalis pedis pulse may be expected findings with extensive trauma to the foot, making the clinical diagnosis even more difficult. Thus, for patients with equivocal findings on physical exam, foot compartment pressures should be measured in order to confirm the diagnosis. There are 8 compartments in the foot: lateral, medial, central, and 4 interosseous. The lateral compartment

    contains the abductor digiti minimi and flexor digiti minimi brevis, and is measured by directing the needle 1cm medial and plantar under the midshaft of the 5th metatarsal. The medial compartment contains the abductor hallicus and flexor hallicus brevis, and is measured by directing the needle lateral and plantar under the base of the first metatarsal. The central compartment contains the oblique head of the adductor hallucis, and is measured through the same approach as the medial compartment after advancing the needle more deeply. The 4 interosseous compartments entail the 2nd, 3rd, and 4th web spaces, and can be measured by directing the needle plantar into each respective dorsal webspace.

     

    Ojika et al. performed a systematic review on foot compartment syndrome. They found that the most common cause of foot compartment syndrome was crush injury to the foot, and that diagnosis was mostly made through a combination of clinical findings and compartment pressure measurements.

     

    Badhe et al. reported 4 cases where competent sensate patients developed compartment syndromes without any significant pain. They found that pain is not a reliable clinical indicator for underlying compartment syndrome, so in a competent sensate patient, the absence of pain does not exclude compartment syndrome. They concluded that a high index of clinical suspicion must prevail in association with either continuous compartment pressure monitoring or frequent repeated documented clinical examination with a low threshold for pressure measurement.

     

    Flynn et al. looked at the diagnosis and outcome of acute traumatic compartment syndrome of the leg in children. They found that a delay in diagnosis may occur because acute traumatic compartment syndrome manifests itself more slowly in children or because the diagnosis is harder to establish in this age group. They state that the results of the present study should raise awareness of late presentation and the importance of vigilance for developing compartment syndrome in the early days after injury.

     

    Figures A and B are lateral and Harris radiographs of the foot demonstrating a calcaneus fracture. Figure C is a cross-sectional image of the foot. Illustration A is an image depicting the compartments of the foot.

     

    Incorrect Answers:

    Answer 1: Path A is the incorrect approach for measuring the central compartment. Additionally, a Δp < 30mmHg (not > 30mmHg) is considered abnormal.

    Answer 2: A Δp < 30mmHg (not > 30mmHg) is considered abnormal. Answer 4: Path C is the incorrect approach for measuring the central

    compartment. Additionally, a Δp < 30mmHg (not > 30mmHg) is considered abnormal.

    Answer 5: Path C is the incorrect approach for measuring the central compartment.

     

     

     

     

     

     

    OrthoCash 2020

     

  77. A 35-year-old female presents to the emergency room after a motor vehicle collision where her leg was pinned under the car for over 30 minutes. A clinical photo and radiographs are shown. Which of the following is the most accurate way to diagnose compartment syndrome?

     

     

     

     

     

     

     

     

    1. surgeon's palpation of the leg compartments

    2. paresthesias in her foot

    3. diastolic blood pressure minus intra-compartmental pressure is less than 30 mmHg

    4. diastolic blood pressure minus intra-compartmental pressure is greater than 30 mmHg

    5. intra-compartmental pressure measurement of 25 mmHg Corrent answer: 3

    The clinical picture is consistent with compartment syndrome. The most accurate way to make the diagnosis is to measure the difference between the diastolic blood pressure and intracompartmental pressure (delta p).

     

    In a prospective study of 116 patients with tibial diaphyseal fractures, McQueen et al found that the use of a differential pressure of 30 mmHg as a threshold for fasciotomy led to no missed cases of acute compartment syndrome. They recommended that a fasciotomy should be performed if the differential pressure level drops to under 30 mmHg.

     

    The cited study by Kakar et al found the intraoperative DBP is significantly lower than the preoperative DBP in patient undergoing IM nailing for tibia shaft

    fractures. Therefore, they emphasize that the surgeon should recognize that intraoperative DeltaP may be lower than DeltaP once the patient is awakened in deciding whether to perform a fasciotomy versus awaken the patient and perform serial examinations and or compartment pressure measurements.

     

    An absolute intra-compartmental value greater than 30 to 45mmHg can also be used to make the diagnosis of compartment syndrome, but is more controversial than the delta p according to Kakar and Amendola.

     

     

     

    OrthoCash 2020

     

  78. What is the most common mode of failure of the lateral ulnar collateral ligament associated with an elbow dislocation?

    1. ligament avulsion off the humeral origin

    2. ligament avulsion off the ulnar insertion

    3. midsubstance rupture

    4. bony avulsion of the humeral origin

    5. combined proximal and distal ligament avulsions Corrent answer: 1

    The lateral ulnar collateral ligament (LUCL) is often injured with elbow dislocations, and is most commonly injured at the proximal origin.

     

    McKee noted that in 62 consecutive operative elbow dislocations and fracture/dislocations, the LUCL was ruptured in all of the patients, proximally in 32, bony avulsion proximally in 5, midsubstance rupture in 18, ulnar detachment in 3, ulnar bony avulsion in only 1, and combined patterns in 3.

     

    Pugh et al established a standard protocol to treat elbow fracture dislocations (terrible triad) which includes coronoid repair, radial head repair/replacement, LUCL repair, and MCL and/or external fixation as needed.

     

     

     

    OrthoCash 2020

     

  79. A 24-year-old male sustains the injury seen in Figure A after being thrown from a motorcycle at a high speed. Which of the following fixation methods has been shown to be the most stable fixation construct for this injury?

     

     

     

    1. Posterior bridge plating and anterior ring external fixation

    2. Percutaneous iliosacral screw and anterior ring external fixation

    3. Percutaneous iliosacral screw and anterior ring internal fixation

    4. Transiliac screw

    5. Two percutaneous iliosacral screws Corrent answer: 3

    Figure A shows an APC III injury, which is a rotationally and vertically unstable injury, with damage to the anterior ring, pelvic floor, and posterior ligamentous stabilizing structures.

     

    The referenced study by Sagi et al found that biomechanically, a percutaneous iliosacral screw and anterior ring internal fixation was the most stable construct. In addition, he found no biomechanical support for addition of a second iliosacral screw.

     

     

     

    OrthoCash 2020

     

  80. A 33-year-old male patient presents with a comminuted open tibia fracture after involvement in a motor vehicle crash. He has a history of smoking but is otherwise healthy. He is given antibiotics, and taken immediately for irrigation and debridement, followed by an un-reamed stainless steel intramedullary nail. Due to bone loss there is a non-circumferential cortical defect measuring 12 mm at the fracture site. All of the following factors in this patient's history and presentation increase his risk for adverse outcome EXCEPT:

    1. High-energy mechanism of injury

    2. Use of un-reamed nail

    3. Implant material

    4. Fracture gap

    5. History of smoking Corrent answer: 2

      Of the factors listed only the use of an un-reamed intramedullary nail for an open tibia fracture has not been shown to increase the risk of adverse outcome or need for reoperation.

       

      The treatment of open tibia fractures with intramedullary nailing can be complicated by many factors. High energy mechanism of injury, use of a stainless steel nail, residual fracture gap greater than 1 cm, and a history of smoking have all been shown to increase the risk of adverse outcome. The use of reamed and un-reamed nails for open tibia fractures have been studied, and no significant difference in outcome has been found.

       

      Schemitsch et al. present data from a prospective randomized trial of tibia fractures treated with reamed or unreamed intrameduallry nails. They found no difference in risk of adverse outcome between reamed and un-reamed nails in open tibia fractures. They did, however, find an increased risk of adverse outcomes in high-energy mechanisms, use of stainless steel (versus titanium) rods, and a residual fracture gap of greater than 1 cm. They comment that their data did not show a significant increase in risk due to history of smoking, but cite other studies that have demonstrated such a relationship.

       

      Bhandari et al. present data from a prospective randomized study of patients with tibia fractures randomized to reamed or un-reamed tibial nails. For closed fractures they found a lower rate of primary events (most commonly need for dynamization) in the reamed group. However, they found no difference in outcomes for either technique in open fractures.

       

      Incorrect answers:

      Answers 1, 3, 4, 5: Each of these factors have been shown to increase the risk of adverse outcome when treating an open tibia fracture with an intramedullary nail.

       

       

       

      OrthoCash 2020

       

  81. Following antegrade intramedullary nailing of a femoral shaft fracture, the complication shown in Figure A occurs. Which of the following errors most likely resulted in this complication?

     

     

     

    1. Applying external rotation torque on the proximal femur after placing proximal interlocking screws

    2. Excessive interfragmentary compression of the fracture site prior to placing proximal interlocking screws

    3. Using too anterior a starting point for a piriformis-entry point nail

    4. Inserting a trochanteric-entry point nail through the piriformis fossa

    5. Inserting a right femoral nail into the left femur Corrent answer: 3

    Using a piriformis nail, a starting point that is too anterior will result in iatrogenic fracture ("bursting") of the proximal femur.

     

    Antegrade nailing achieves fixation via 3-point fixation. In the sagittal plane, because of anterior sagittal bow, this is achieved at distal anterior cortex, middle posterior cortex (apex of curvature) and proximal anterior cortex. In the coronal plane, because of the lateral bow, this is achieved at the lateral distal femur, middle medial femur (apex of curvature), and proximal lateral femur (greater trochanter). Piriformis nails have a single sagittal bow.

    Trochanteric nails are bowed in 2 planes, necessitating a twisting motion during insertion to negotiate both bows.

     

    Papadakis et al. performed an experimental study on 18 cadaveric femora. Anterior bursting was found in 56% of nails placed too anteriorly. Bursting was not seen in nails placed through a more posterior entry point. They emphasize the location of the entry point when performing antegrade nailing.

     

    Johnson et al. reviewed the biomechanical factors affecting fracture stability

    and femoral bursting. They found that position of the starting hole was most important. Anterior displacement by >6mm led to high hoop stresses and bursting of the anterior cortex. This is important as an eccentrically reamed cortex may be difficult to recover from. They recommend either selecting a smaller diameter nail or overreaming by 1-2mm as a solution.

     

    Figure A shows a fracture split of the proximal femur (left, without magnification; right, close-up). Illustration A shows anterior cortex pressures exceeding 100kPa for too-anterior entry points.

     

    Incorrect Answers:

    Answer 1: External rotation of the proximal femur after proximal interlocking will lead to the entire proximal femur rotating externally. Screw cut-out is possible, but split fractures of the proximal femur will not occur.

    Answer 2: Excessive interfragmentary compression at the fracture site will lead to iatrogenic fragmentation at the fracture site, but will not produce proximal bursting at the entry site.

    Answer 4: Inserting a trochanteric nail through the piriformis fossa will lead to valgus malalignment at the fracture site. Inserting a piriformis nail through the greater trochanter will result in varus malalignment at the fracture site.

    Answer 5: Right and left nails are characteristic of trochanteric nails (double bowed). Piriformis nails can be applied to either side (single sagittal bow).

    Wrong-side placement of a trochanteric entry nail would lead to varus malalignment at the fracture site, more so than placing a piriformis nail through the greater trochanter.

     

     

     

     

     

     

    OrthoCash 2020

     

  82. A 30-year-old male sustains a brachial plexus injury as the result of a motor vehicle collision. Palsy of which of the following muscles would not be expected with this injury if the injury was postganglionic in nature?

    1. Rhomboid major

    2. Extensor carpi radialis longus

    3. Biceps brachii

    4. Deltoid

    5. Brachioradialis

     

    Corrent answer: 1

     

    A brachial plexus injury would involve all of the upper extremity muscles as well as most of the periscapular muscles. Complete plexus palsies are rare, and are often associated with scapulothoracic dissociation or other high-energy injuries.

     

    Preganglionic injuries often involve the cervical paraspinal musculature as well as a complete plexus injury. EMG evidence of intact signals in the serratus anterior (long thoracic nerve) and rhomboids (dorsal scapular nerve) are suggestive of a postganglionic lesion/injury.

     

    Tubbs et al. reported on the surgical anatomy of the dorsal scapular nerve in a cadaver study. They found that the nerve came off the C5 nerve root in 95%, ran 2.5cm medial to the spinal accessory nerve as it traveled on the anterior border of the trapezius muscle, and was intertwined with the dorsal scapular artery in all specimens.

     

    Balakrishnan et al reported on the comparison of clinical exam and EMG in predicting site of lesions in brachial plexus injuries. The combination of EMG and exam localized the nerve injury in 80%, while the paraspinal EMG was the most sensitive solitary examination method (67%).

    Illustration A shows a diagram of the brachial plexus. Incorrect Answers:

    2-5: These muscles are all innervated by nerves that come from the brachial

    plexus, and would be affected with a postganglionic injury.

     

     

     

     

     

    OrthoCash 2020

     

  83. A 40-year-old male sustains a fall from a height. He sustains the isolated injury shown in the radiograph and CT images seen in Figures A through C. Surgery is planned. Which of the following constructs is the most appropriate definitive fixation for this injury?

     

     

     

     

     

     

     

     

    1. Lateral locked plating

    2. Medial bridge plating

    3. Medial buttress plate

    4. Medial lag screw with washers

    5. External fixation

     

    Corrent answer: 3

     

    This patient has a medial tibial plateau fracture. Medial buttress plating (MBP) is indicated.

     

    Medial tibial plateau fractures (Schatzker IV, Hohl and Moore II) may represent fracture dislocations of the knee. Neurovascular injuries must be excluded. In these instances, the medial fragment represents the stable fragment, and the rest of the tibia is the fractured fragment, with the entire leg acting as the lever arm.

     

    Berkson et al. reviewed high energy tibial plateau fractures. Medial fractures may be treated with a medial plate or external fixation. Open reduction may be necessary because of fracture line obliquity and propensity of the medial plateau to shorten and rotate in the sagittal plane. Meniscal injuries should be repaired and avulsed cruciates fixed early. Collateral ligaments should be reconstructed after obtaining bone healing and range of motion.

     

    Ratcliff et al. compared the stability of lateral locked plates and medial buttress plates in a cadaver model. They found that the medial buttress plate had greater fixation strength/failure force (4136N) compared with the lateral locking plate (2895N), although maximum cyclic displacement and residual displacement results were not different. They concluded that for vertically

    orientated medial tibial plateau fractures, medial buttress plates were more stable.

     

    Figure A is a radiograph showing a medial tibial plateau fracture extending across the tibial eminence. This is also classified as a Hohl and Moore Type II fracture. Figures B and C are coronal and axial CT scan images confirming the medial tibial plateau fracture and excluding a bicondylar fracture. Illustration A shows a medial tibial plateau fracture fixed with a medial buttress plate (cadaveric model with fibula removed). Illustration B shows the Hohl and Moore Type II fracture involving the entire condyle.

     

    Incorrect Answers:

    Answer 1: Lateral locked plating is biomechanically inferior to medial buttress plating for medial tibial plateau fractures (see reference above).

    Answer 2: Medial bridge plating is appropriate for segmental or comminuted fractures

    Answer 4: Lag screw fixation alone is contraindicated because of the long lever arm of the leg may lead to implant failure.

    Answer 5: Temporary spanning external fixation may be appropriate for extensive comminution or soft tissue injury/swelling. Definitive external fixation (non-joint spanning) with olive wires, monolateral half-pins or both (hybrid) are possible options but less preferred as wires <14mm from the articular surface risk joint capsule penetration and septic arthritis.

     

     

     

     

     

     

     

     

    OrthoCash 2020

     

  84. A 39-year-old female presents with the following motor vehicle crash with the injury seen in Figure A (immobilized in a pelvic binder). The iatrogenic neurologic injury most commonly caused by placement of the anterior construct for this injury, as shown in Figure B, would cause which of the following?

     

     

     

     

     

     

    1. Weakness of hip flexion

    2. Weakness of ankle dorsiflexion

    3. Numbness of the medial thigh

    4. Numbness of the lateral thigh

    5. Numbness of the perineum Corrent answer: 4

    This patient was treated with posterior stabilization, and an anterior subcutaneous internal fixator (ASIF). The most common neurologic injury seen following placement of the ASIF construct is irritation of the lateral femoral cutaneous nerve (LFCN), causing numbness and/or pain of the lateral thigh.

     

    Unstable pelvic fractures can be treated in a multitude of ways. The ASIF construct is typically created by placing long pelvic screws or polyaxial pedicle screws in the supraacetabular region, similar to the supraacetabular pins for an

    anterior external fixator. Then a curved bar is placed subcutaneously and connected to the supraacetabular screws. They are typically removed after 3-4 months when fracture healing is complete.

     

    Vaidya et al. present a retrospective review of the use of ASIF as definitive treatment of unstable pelvic fractures. All patients in the study tolerated the construct well. LFCN irritation was seen in 30% of patients, and resolved in all but one patient.

     

    Müller et al. present a retrospective review of the use of posterior stabilization and ASIF. They report an acceptably low complication rate, and good to excellent outcomes in 64.5% of patients.

     

    Figure A is a radiograph demonstrating a right APC3 and left APC2 pelvic injury, imaged in a pelvic binder. Figure B is a postoperative radiograph following posterior stabilization and ASIF.

     

    Incorrect answers:

    Answer 1: Weakness of hip flexion would be due to injury to the femoral nerve, which is not commonly associated with this construct.

    Answer 2: Weakness of ankle dorsiflexion would be due to injury to the L5 nerve root, which does not occur due to ASIF placement.

    Answer 3: Numbness of the medial thigh is due to injury to the obturator nerve, which is not commonly associated with this construct.

    Answer 5: Numbness of the perineum is due to injury to the sacral nerve roots, which is not commonly associated with this construct.

     

     

     

    OrthoCash 2020

     

  85. What would be the most appropriate surgical indication for transferring fascicles of the ulnar nerve to the motor nerve of the biceps and fascicles of the median nerve to the motor nerve of the brachialis?

    1. C8 - T1 nerve root avulsion 3 months ago

    2. C5 - C6 nerve root avulsion 2 months ago

    3. Upper brachial plexus palsy 22 months ago

    4. Medial and posterior cord injury from gunshot wound 2 months ago

    5. C6 ASIA A spinal cord injury Corrent answer: 2

    Transfer of fascicles from (1) ulnar nerve to the nerve to the biceps and (2) median nerve to the motor nerve of the brachialis would be appropriate in the

    treatment of an acute (<3-6 months) upper brachial plexus palsy.

     

    Upper trunk injury (C5, C6) often results from the avulsion of both the C5 and C6 nerve roots. Injuries of this nature usually result from a downward force on the shoulder with lateral bending of the cervical spine in the opposite direction. This results in what is commonly called an Erb-Duchenne palsy. Patients often present with a flail shoulder and loss of elbow flexion. Other common treatments for C5 and C6 root avulsion include neurotization of the musculocutaneous (MSC) nerve by the spinal accessory (SA) or intercostal nerve, and neurotization of the supra-scapular nerve by the SA.

     

    Liverneaux et al. looked at short term results of (1) ulnar nerve fascicle transfer to the nerve to the biceps and (2) fascicle of the median nerve to the motor branch to the brachialis in 15 patients with acute C5 - C6 nerve root avulsion injuries. Grade 4 elbow flexion was restored in each of the 10 patients. There was no secondary deficit in grip strength or sensation.They concluded that this double nerve transfer technique will likely reduce the need for secondary procedures to augment elbow flexion.

     

    Teboul et al. reviewed thirty-two patients with an upper nerve-root brachial plexus injury that underwent ulnar nerve fascicle transfer to the nerve of biceps to restore elbow flexion. After the nerve transfer, twenty-four patients achieved grade 3 elbow flexion strength or better. They note that this procedure will spare the C5 nerve root and other nerves for grafting or transfer elsewhere.

     

    Illustration A shows harvesting of an ulnar nerve fascicle for transfer. Illustration B shows transfer of the fascicle of the ulnar nerve to the motor nerve of the biceps.

     

    Incorrect Answers:

    Answer 1: C8 - T1 nerve root avulsion would result in ulnar nerve dysfunction. Transfer of the non-functional ulnar nerve to the motor nerve of the biceps would be a redundant procedure.

    Answer 3: Nerve transfers in upper brachial plexus palsy more than 20-24 moths post-injury is a relative contraindication due to the eventual loss of neuromuscular end plates at 20 to 24 months after denervation. Free functioning muscle transfers are more commonly indicated in late presenting injuries.

    Answer 4: Medial and posterior cord injury from gunshot wound likely leave the musculocutaneous nerve intact (lateral cord). Therefore, this transfer technique would not be indicated.

    Answer 5: C6 ASIA A spinal cord injury would likely result in motor and

    sensory quadriplegia. Nerve transfers using the ulnar nerve (C8-T1) would also be redundant as this nerve would be non-functional in this patient.

     

     

     

     

     

     

     

     

     

    OrthoCash 2020

     

  86. A 31-year-old female presents to the trauma bay following a motorcycle crash. Her blood pressure is 95/70 mmHg, heart rate is 115 bpm. Lactate measured in the trauma bay is 10 mmol/L. She has multiple rib fractures, pulmonary contusions, and a positive FAST exam requiring immediate exploratory laparotomy. After laparotomy her lacate remains unchanged. She has a closed right femur fracture and an open right tibia fracture as seen in Figures A and B. Besides antibiotics and thorough irrigation and debridement, which of the following would be an appropriate step in the immediate management of her fractures?

     

     

     

     

     

     

    1. Reamed intramedullary nailing of the tibia and femur

    2. Un-reamed intramedullary nailing of the tibia, and reamed intramedullary nailing of the femur

    3. Reamed intramedullary nail of the tibia, and un-reamed intramedullary nail

      of the femur

    4. Posterior slab splint of the tibia, and 10 lbs skeletal traction of the femur

    5. External fixation of the tibia and femur Corrent answer: 5

    This patient is suffering from multiple injuries and has evidence of chest injury and incomplete resuscitation. The immediate treatment of her fractures should be external fixation for both the tibia and the femur.

     

    For polytraumatized patients with multiple injuries including extremity fractures, damage control orthopaedics dictates that long bone fractures should be temporarily stabilized. Either inadequate stabilization, or early total care, such as a reamed or unreamed nails, can exacerbate the patient's condition and increase the risk of a second-hit phenomenon. For this patient with pulmonary contusions and continued elevation of lactate indicating end-organ hypoperfusion her extremities should have staged treatment according to damage control principles.

     

    Morshed et al. present a retrospective review of polytraumatized patients with femur fractures and compared outcomes based on the time frame in which their fractures were definitively treated. They found delaying treatment at least 12 hours to allow appropriate resuscitation and treatment of other traumatic injuries led to a decrease in mortality of 50%. Patients with intra-abdominal injuries benefited most from staged treatment of the extremities.

     

    Figure A is a radiograph showing a closed right femur fracture. Figure B is a radiograph of an open right tibia fracture.

     

    Incorrect answers:

    Answers 1-3: Immediate intramedullary nailing, either reamed or unreamed, in this patient would increase her risk of fat emboli syndrome, and should not be done acutely.

    Answer 4: A posterior slab splint would not adequately stabilize the tibia fracture seen, and should not be used instead of external fixation. Splints would also make it more difficult to monitor this patient's skin, compartments, and her traumatic wound. Similarly, skeletal traction alone of the femur fracture would be insufficient.

     

     

     

    OrthoCash 2020

     

  87. A 68-year-old woman undergoes a hemiarthroplasty for a proximal humerus fracture through a deltopectoral approach. What

    range of motion exercise should not be utilized in the immediate postoperative period due to concerns about lesser tuberosity fixation?

    1. Pendulums

    2. Passive internal rotation of the shoulder to the plane of the body

    3. Active forearm supination

    4. Passive external rotation of the shoulder past 30 degrees

    5. Passive forward flexin of the shoulder to 90 degrees Corrent answer: 4

    Frankle et al found that passive external rotation of the shoulder placed the most stress on the lesser tuberosity fixation. The subscapularis tendon inserts on the lesser tuberosity and is the deforming force when placed under tension during external rotation. They also found that non-anatomic tuberosity reduction of 4-part proximal humerus fractures treated with hemiarthroplasty increased torque and impaired external rotation kinematics.

     

     

     

    OrthoCash 2020

     

  88. A 72-year-old female sustains a displaced intracapsular femoral neck fracture. Which of the following is TRUE regarding the long term differences between possible treatment options for this injury?

    1. Patients undergoing total hip arthroplasty are more likely to experience persistent pain than those undergoing internal fixation

    2. Patients undergoing total hip arthroplasty are less likely to require reoperation than those undergoing internal fixation

    3. There is no difference in functional outcome scores between internal fixation and total hip arthroplasty

    4. Patients undergoing internal fixation perform activities of daily living better than those undergoing total hip arthroplasty

    5. Mortality rates are higher following total hip arthroplasty than internal fixation

    Corrent answer: 2

     

    Elderly patients with femoral neck fractures (FNF) undergoing total hip arthroplasty (THA) are less likely to require reoperation than those undergoing internal fixation.

     

    Intracapsular FNF are common in elderly patients after a fall from standing height. Treatment depends on physiological age and displacement (Garden's classification). For displaced fractures, physiologically young patients are treated with internal fixation while physiologically old patients are treated with

    either hemiarthroplasty (debilitated, less active patients) or THA (more active patients, those with acetabular disease or preexisting inflammatory arthritis).

     

    Chammout et al. retrospectively compared the long term (17 years) results of THA (cemented both component) and ORIF (2 cannulated screws) in elderly patients (>65 years). They found no difference in mortality. But hip scores were higher and pain was better in the THA group, while reoperation rates were higher in the ORIF group. Walking speed was initially faster in the THA group, but later did not differ between groups. They recommend THA for elderly patients with displaced FNF.

     

    Rogmark et al. prospectively compared closed reduction and internal fixation (CRIF) with arthroplasty (combining hemiarthroplasty and THA) at 2 years in elderly patients (>70 years). Failure rates were higher, pain was worse, and walking was more impaired after CRIF. They recommend arthroplasty for patients >70 with FNF.

     

    Incorrect Answers:

    Answer 1: Patients undergoing THA are more likely to have less pain than internal fixation.

    Answer 3: Patients undergoing THA have superior functional outcome scores. Answer 4: Patients undergoing THA perform ADL better in the short term. In the long term, there is no difference in ADL between the groups.

    Answer 5: Mortality rates are similar after the two procedures.

     

     

     

    OrthoCash 2020

     

  89. A polytrauma patient underwent the following procedures: (1) statically locked intramedullary nailing for a right femoral shaft fracture; (2) open reduction with plate-and-screw fixation [ORIF] for a right simple distal fibula fracture; (3) ORIF right middle third radius and ulna fracture; and (4) ORIF left humeral shaft fracture. What is the appropriate weightbearing status for this patient?

    1. Weight bearing as tolerated in all extremities

    2. Early protected weight bearing right lower extremity in walking cast, weight bearing as tolerated left upper extremity, non-weight bearing right forearm

    3. Weight bearing as tolerated in bilateral lower extremities and right upper extremity, non-weight bearing left upper extremity

    4. Non-weight bearing bilateral upper extremities and right lower extremity

    5. Non-weight bearing right upper and lower extremities, weight bearing as tolerated left upper and lower extremities without walking cast

    Corrent answer: 2

     

    The standard postoperative weightbearing for locked medullary nailing for femoral shaft fractures and humeral shaft fractures is weight bearing as tolerated (WBAT). Simple ORIF ankle fractures may be managed with early protected weight bearing. ORIF right middle third radius and ulna fracture should be managed with a period of non-weight bearing due to risk of secondary displacement of the fracture.

     

    Tingstad et al. examined the effect of immediate weightbearing on plated fractures of the humeral shaft. They reported that immediate weightbearing on humeral shaft fractures, treated with plating and full weightbearing, did not have any negative effect on the union or malunion rates.

     

    Brumback et al. evaluated the feasibility, safety and efficacy of immediate weightbearing after treatment of femoral shaft fractures with statically locked IM nail. Using biomechanical and clinical data, they showed that all fractures united with no loss of fixation or hardware failure.

     

    Starkweather et al. retrospectively assessed the complications and loss of reduction in patients who bore weight in a short leg cast within 15 days after surgical repair of acute unilateral closed ankle fractures. Of the 81 ankle fracture radiographs, 80 (98.8%) showed no displacement in fracture reduction on the final follow-up examination. These results suggest early protected weightbearing may be safe.

     

    Incorrect Answers:

    Answers 1, 3-5: Protected weight bearing has shown to shorten hospital stay and return of function with lower extremity fractures, without a significant increase in hardware failure. Both bone forearm fractures should not be treated with early weight bearing.

     

     

     

    OrthoCash 2020

     

  90. A 22-year-old female falls off the back of a motorcycle and sustains the injury in Figure A. She is hemodynamically unstable and massive transfusion protocol is activated. What is the correct ratio of transfusion of packed red blood cells, platelets and plasma?

     

     

     

    1. 2:1:1

    2. 3:1:1

    3. 1:1:1

    4. 1:1:2

    5. 1:1:3

     

    Corrent answer: 3

     

    When massive transfusion protocols are activated, the correct ratio of packed red blood cells, platelets, and plasma is 1:1:1. Implementation of these ratios allows for homeostasis of factors that avoid tipping the balance of the coagulation cascade.

     

    Incorrect ratios of transfusion can lead to physiologically unfavorable states, including coagulopathy and life threatening metabolic imbalances. The lethal triad includes acidosis, hypothermia, and coagulopathy. Also possible but rare is associated citrate toxicity, and transfusion-associated lung injury.

     

    Langford et al. reviewed the typically presentation, initial evaluation, management and treatment of pelvic fractures. APC-III fractures have the highest association with bleeding and the authors emphasize the importance of close hemodynamic status monitoring. Initial clot can be achieved with the placement of a pelvic binder around the greater trochanters, and the initiation of massive transfusion if necessary. Frequent, close monitoring of associated electrolytes are mandatory.

     

    Elmer et al. explain the preferred ratio of 1:1:1 as a goal, not to achieve a physiologic hemodynamic state, but to offer a favorable environment to facilitate resuscitation while avoiding excessive fluctuations in the coagulation cascade and metabolic state. In this paper, they work through a case presentation of a patient requiring massive transfusion, in which electrolyte

    balance was maintained with supplemental calcium gluconate, potassium, and crystalloid fluids as needed with frequent, regular lab analysis.

    Figure A exhibits a widely displaced APC pelvic injury. Incorrect answers:

    Answers 1, 2, 4, and 5 are incorrect ratios of transfusion.

     

     

     

    OrthoCash 2020

     

  91. A 25-year-old male presents with the injury seen in Figures A and B following a motorcycle collision. He has an ipsilateral open tibia fracture. No other injuries are noted. He is hemodynamically stable and cleared for operative intervention. What would be the most appropriate definitive treatment for this injury?

     

     

     

     

     

    1. Reamed unlocked antegrade nailing

    2. Unreamed antegrade nailing with dynamic interlocking

    3. Reamed unlocked retrograde nailing

    4. Reamed retrograde nailing with static interlocking

    5. Reamed retrograde nailing with dynamic interlocking Corrent answer: 4

    Reamed, statically locked nailing is the standard treatment for diaphyseal femur fractures. In this patient with an ipsilateral lower extremity fracture, retrograde nailing allows for supine positioning to address both injuries.

     

    Antegrade, reamed, statically locked intramedullary nailing is the treatment of choice for the majority of diaphyseal femur fractures. Relative indications for retrograde nailing include multi-injured patients, ipsilateral lower extremity trauma, morbid obesity, and infra-isthmal fracture patterns.

     

    Brumback and Virkus review the current concepts and controversies regarding the management of femoral shaft fractures with reamed and unreamed nailing. The authors highlight the benefits of reaming including deposition of local autograft, stimulation of periosteal blood supply and increased nail diameter.

    They note that there is a subset of trauma patients, specifically those with pulmonary injury, that may be adversely affected by nailing.

     

    Figures A and B demonstrate AP and Lateral views of a comminuted mid-diaphyseal femoral shaft fracture. Illustrations A and B demonstrate the same fracture that progressed to uneventful union after treatment with a reamed, statically locked retrograde nail supplemented with blocking screws to correct angular alignment.

     

    Incorrect answers:

    Answers 1 and 3: Unlocked nailing is rarely indicated and is mainly of historical significance

    Answer 2: Antegrade nailing would not necessarily allow for the tibia fracture to be addressed simultaneously. Dynamic interlocking is contra-indicated in this length unstable fracture pattern

    Answer 5: Dyamic interlocking is not typically used for femoral shaft fractures. It would be contra-indicated in this length unstable femur fracture.

     

     

     

     

     

     

     

    OrthoCash 2020

     

  92. Figure A shows intra-operative radiographs of a 44-year-old male patient that has undergone fracture fixation of an acute elbow dislocation. Examination under anesthesia in the operating room reveals there is residual posterolateral instability when the elbow is extended < 35°. What would be the next most appropriate step in the management of his injury?

     

     

     

     

    1. Application of a long arm cast, with the elbow flexed >40 degrees

    2. Application of a hinged elbow brace and a progressive range of motion protocol

    3. Repair of the lateral collateral ligament complex

    4. Repair of the medial collateral ligament complex

    5. Application of an elbow external fixator, with the elbow flexed >40 degrees.

     

    Corrent answer: 3

     

    This patient has sustained a terrible triad injury. He has undergone successful fixation of the radial head and coronoid fractures, but has residual posterolateral instability. The next most appropriate step in the management would be repairing the LCL in the operating room.

     

    The step-wise principles for the operative management of terrible triad injuries include, (1) restore coronoid stability through fracture fixation or capsular repair, (2) restore radial head stability through fracture fixation or replacement with a metal prosthesis, (3) restore lateral elbow stability through repair of the lateral collateral ligament (LCL) complex (4) repair the medial collateral ligament (MCL) in patients with residual posterior instability, and (5) apply a hinged external fixator when conventional repair did not establish sufficient joint stability to allow early motion.

    Pugh et al. provided a surgical protocol for elbow dislocations with associated radial head and coronoid fractures. They showed that early intervention, stable fixation, and repair would provide sufficient stability to allow motion in patients postoperatively and enhance functional outcomes. Prolonged immobilization following an acute episode of elbow instability is associated with poor results.

     

    Ring et al. retrospectively reviewed eleven patients that underwent operative fixation of terrible triad injuries. At two year follow-up, three patients were considered to have a failure of the initial treatment. There was significant loss of elbow range of motion with an average of 92 degrees (range, 40 degrees to 130 degrees) of ulnohumeral motion.

     

    Schneeberger et al. examined the role of the radial head and coronoid process as posterolateral rotatory stabilizers of the elbow. Excision of the radial head in an elbow with intact collateral ligaments caused a mean posterolateral rotatory laxity of 18.6 degrees (p < 0.0001) compared to 5.4 degrees in the intact elbows. Additional removal of 30% of the height of the coronoid fully destabilized the elbows, always resulting in ulnohumeral dislocation despite intact ligaments.

     

    Figure A shows intraoperative radiographs following open reduction internal fixation of the coronoid fracture and radial head fracture.

     

    Incorrect Answer:

    Answer 1: Prolonged immobilization following an acute episode of elbow instability is associated with poor results.

    Answer 2: A hinged brace would not address the residual posterolateral instability in this patient. Surgical repair of LCL +/- MCL +/- application of external fixator would be required, sequentially, until elbow stability is restored.

    Answer 4: Medial collateral ligament repair is indicated in patients with residual instability after fixing the lateral collateral ligament complex.

    Answer 5: Application of an elbow external fixator would be the final step in the surgical algorithm to maintain elbow stability after the medial collateral ligament is repaired.

     

     

     

    OrthoCash 2020

     

  93. Figures A and B are post-operative radiographs of a 54-year-old female. In the first 6 months after this procedure, what is the most likely factor for functional impairment in this patient?

     

     

     

     

     

    1. Osteonecrosis

    2. Anterior knee pain

    3. Re-fracture

    4. Hardware failure

    5. Non-union

     

    Corrent answer: 2

     

    A residual deficit in muscle performance and anterior knee pain are expected in the majority of patients at 6 months after surgical fixation of their patella fractures.

     

    Anterior knee pain is reported to be a common symptom following treatment of patellar fractures. A likely contributing factor to the anterior knee pain is scarring and tightness of the structures surrounding the knee, as well as patella maltracking due to quadricep/hamstring weakness and/or poor muscle synchrony. Other factors for anterior knee pain may include symptomatic hardware, which may be treated with removal of fixation after union has been achieved.

     

    Lazaro et al. looked at the outcome data on thirty patients with isolated unilateral patellar fractures. Anterior knee pain during activities of daily living was experienced by twenty-four (80%) of the patients. The knee extensor mechanism on the injured side had deficits in strength (-41%), power (-47%), and endurance (-34%) as compared with the uninjured side.

     

    Lebrun et al. reviewed a series of 40 operatively treated patella fractures and found that at over 6 years postoperatively, significant symptomatic complaints and functional deficits persisted based on validated outcome measures as well as objective physical evaluations. Removal of symptomatic fixation was required in 52% of the patients treated with osteosynthesis, whereas 38% of those with retained fixation self-reported implant-related pain at least some of the time.

     

    Figure A and B show AP and lateral radiographs of a comminuted patella fracture treated with a tension band repair construct. The articular surface looks well reduced.

     

    Incorrect Answers:

    Answer 1: Osteonecrosis of the patella is rare, occurring in less than 5% of patella fractures. It most commonly occurs in the inferior pole of the patella. Answer 3: Re-fracture is rare after osteosynthesis with retained hardware.

    Answer 4: Hardware failure is more common with poor surgical technique. Short term reported data shows prominent and symptomatic implants as a

    result of breakage to occur in <50% of patients.

    Answer 5: Non-union is rare in minimally displaced patella fractures.

     

     

     

     

    OrthoCash 2020

     

  94. A 40-year-old male presents with the injury seen in Figure A after a fall from height. He undergoes closed reduction with sedation in the trauma bay. The house staff who initially evaluated the patient does not recall if the patient had palpable pulses prior to reduction but does state that the foot felt "cool". Following closed reduction he has palpable dorsalis pedis pulses and brisk capillary refill. His knee is able to be flexed to approximately 60 degrees prior to any clinical instability. AP and Lateral X-rays of his knee demonstrate a congruent reduction in a knee immobilizer. His ankle brachial index (ABI) is 0.95. What is the appropriate next step in the treatment of this patient?

     

     

     

     

    1. CT Angiogram

    2. Serial clinical examination

    3. Knee spanning external fixation and vascular exploration

    4. Arteriography

    5. Immediate ligament reconstruction Corrent answer: 2

    Serial examination is an appropriate next step in the management of this patient with a knee dislocation with associated tibial plateau fracture. After limb alignment was restored, the patient has adequate distal flow clinically and an ABI >0.9.

     

    It is critical to evaluate for the presence of vascular injury in patients presenting with knee dislocations. The first priority should be reduction of the dislocation to restore limb alignment, prior to proceeding with advanced imaging or invasive procedures. Once alignment has been restored, patients without hard signs of vascular injury (expanding hematoma, cool and pulseless limb) should be evaluated using the ankle-brachial index.

     

    Mills et al prospectively evaluated 38 patients presenting with knee dislocation utilizing the ABI as a screening tool for vascular injury. The authors found that an ABI less than 0.9 predicted clinically relevant vascular injury with 100% sensitivity (11 of 11 patients). Those patients with ABI >0.9 (n=27) had no evidence of vascular injury on serial examination.

     

    Figure A demonstrates an anterior knee dislocation with an associated posteromedial plateau fracture.

     

    Incorrect Answers:

    Answer 1: CT angiogram is not indicated in this patient with no clinical evidence of vascular injury and an ABI >0.9

    Answer 3: The patient may require knee spanning external fixation if his knee is grossly or unstable reduction cannot be maintained in a brace, however vascular exploration is not indicated

    Answer 4: Formal arteriography is not indicated as there is no evidence of vascular injury

    Answer 5: This patient will likely require ligamentous reconstruction however the next appropriate step in treatment is monitoring for changes in vascular status and compartment syndrome

     

     

     

    OrthoCash 2020

     

  95. A 34-year-old otherwise healthy male is involved in a motor vehicle collision and sustains the injuries shown in the images below. His initial lactate is 8 and blood pressure is 90/50. He receives 2 liters

    of normal saline followed by 2 units of crossmatched packed red blood cells, plasma and platelets. His repeat lactate just prior to being taken to the operating room is 7. What is the most appropriate treatment at this time?

     

     

     

     

     

     

    1. Unreamed femoral intramedullary nailing; open reduction and internal fixation of the pelvis

    2. External fixation of the femur and pelvis

    3. Reamed intramedullary nailing of the femur; external fixation of the pelvis

    4. External fixation of the femur; open reduction and internal fixation of the pelvis

    5. Splinting of the femur and external fixation of the pelvis.

     

    Corrent answer: 2

     

    The patient has multiple injuries including pelvic trauma resistant to initial resuscitative measures as evidenced by the persistently elevated lactate. External fixation of both the femur and pelvis should be employed at this time to avoid additional insult to a patient with evidence of end-organ hypoperfusion.

     

    Early total care versus stabilization and eventual definitive fixation remains controversial. Indications to employ DCO include an injury severity score (ISS)

    >40 without thoracic trauma, ISS>20 with thoracic trauma, severe pelvic/abdominal injuries and hemorrhagic shock, bilateral femur fractures, pulmonary contusions, and a base deficit >2. Early definitive fixation can cause a "second hit" and increase the risk for acute respiratory distress syndrome (ARDS) and multi-organ failure.

     

    D'Alleyrand et al. review the current evidence and practical applications of early appropriate care. They conclude that controversy continues regarding which "borderline" patients benefit from DCO and the ideal timing of fracture fixation surgery. They state that patients with closed head injuries, poor response to resuscitation, and poor ventilator parameters are good candidates for DCO.

     

    Pape et al. review the timing of fracture fixation in polytrauma patients. They conclude that DCO, which uses external fixation as a primary tool, is most appropriate for patients in severe hemorrhagic shock or any other life-threatening condition.

     

    Figure A demonstrates a pelvic ring injury with widening of pubic symphysis, associated anterior column acetabular fracture, and widening of the left SI joint. Figure B demonstrates a comminuted femoral shaft fracture.

     

    Incorrect Answers:

    Answers 1, 3, 4: All have definitive fixation of either the pelvis or femur that may lead to a "second hit".

    Answer 5: Splinting of the femur would limit the ability to mobilize the patient.

    Additionally, continued motion at the fracture site may potentiate local and systemic inflammation.

     

     

     

    OrthoCash 2020

     

  96. A 33-year-old woman sustained a right elbow dislocation 18 months ago and complains of persistent elbow stiffness. She has been refractory to nonsurgical treatment and is scheduled to undergo surgical elbow release. The range of motion of her elbow measures 30 to 95 degrees. Figure A shows a picture from the pre-operative clinic visit, while a radiograph of her elbow can be seen in Figure B. What operative steps would best restore range of motion in her elbow?

     

     

     

     

     

     

     

    1. Supracondylar corrective osteotomy and release of anterior bundle of the medial collateral ligament (MCL)

    2. Excision of heterotopic ossification, release of anterior capsule, lateral

      collateral ligament (LCL) and ulnar nerve decompression

    3. Excision of heterotopic ossification and release of anterior capsule

    4. Release of posterior capsule, anterior capsule, LCL and ulnar nerve decompression

    5. Excision of heterotopic ossification, release of posterior bundle of the MCL and ulnar nerve decompression

    Corrent answer: 5

     

    This patient has significant right elbow stiffness. The steps to best restore range of motion in her elbow would include, removing heterotopic ossification and release of posterior bundle of the MCL and ulnar nerve decompression.

     

    Joint stiffness is a well-established complication following elbow trauma. The arc of elbow motion from 30° to 130° is required to perform most activities of daily living. The causes of stiffness can be broken down into intrinsic and extrinsic causes. Intrinsic stiffness can be caused by intra-articular injuries such as, articular cartilage injury, articular incongruency, or impingement.

    Contracture of the extra-articular soft-tissue structures is known to be a cause of extrinsic stiffness. Heterotopic bone is another factor that is known to cause a mechanical limitation of motion.

     

    Park et al. reviewed forty-two patients with <100° of elbow flexion due to an extrinsic contracture after trauma. They showed that posttraumatic heterotopic ossification, most commonly in the posteromedial aspect of the capsule, is closely associated with loss of elbow flexion. Surgical release should include releasing the posterior band of the medial collateral ligament and excision of heterotopic bone.

     

    Williams et al. looked at the outcomes of ulnar nerve decompression in patients with preoperative ulnar nerve symptoms or a positive Tinel test and elbow stiffness. They found the rate of developing postoperative ulnar nerve symptoms was significantly higher in patients with preoperative flexion < 100° (15.2%) compared with those with preoperative flexion > 100° (3.7%).

     

    Figure A shows a claw hand deformity relating to underlying ulnar nerve compression at the elbow. Figure B shows a lateral radiograph of the right elbow with significant posterior heterotopic ossification.

     

    Incorrect Answers:

    Answer 1: The MCL is composed of anterior, posterior, and transverse bundles. After decompression of the ulnar nerve, release of the medial collateral ligament should be done sequentially starting with the posterior bundle, transverse component of the ligament, and occasionally the posterior half of

    the anterior bundle of the medial collateral ligament. There is no indication for supracondylar osteotomy.

    Answer 2 and 3: Release of the anterior capsule would improve elbow extension, which would not be a critical step in the management of this patient as she has funcational elbow extension. Release to the lateral collateral ligament would render the elbow unstable.

    Answer 4: Ulnar nerve decompression would be indicated as this patient has ulnar nerve symptoms pre-operatively, as well as preoperative flexion < 100°. Releasing posterior capsule can help to improve elbow flexion. However, release to the anterior capsule and lateral collateral ligament would not be indicated in this scenario.

     

     

     

    OrthoCash 2020

     

  97. Which of the following types of nonunions is most likely to achieve union following a reamed exchange intramedullary nailing only?

    1. Distal femoral nonunion with less than 10% bone loss

    2. Infected nonunion of the femoral shaft

    3. Mid-diaphyseal humeral nonunion with less than 10% bone width loss

    4. Proximal humeral shaft nonunion with less than 10% bone width loss

    5. Diaphyseal tibial shaft nonunion with less than 30% cortical width bone loss Corrent answer: 5

    Reamed exchange intramedullary nailing of diaphyseal tibial shaft fractures in which there is less than 30% of cortical bone loss can achieve union rates ranging between 76%-96%.

     

    In a review article, Brinker et al discusses the indications and limitations of exchange nailing of ununited fractures. Biological as well as mechanical factors contribute to the healing of nonunions. Anatomically, multiple studies cited in this review article demonstrate that distal femoral nonunions do not readily achieve union following exchange nailing. Humerus nonunions, both diaphyseal and proximal locations, more readily achieve union with plate fixation and bone grafting according to articles cited in this review as well.

     

    Banaszkiewicz et al also discusses the difficulties with exchange nailing of femoral nonunions with a large percentage of patients requiring additional surgeries to achieve union.

     

    Templeman et al discusses the successful results of reamed exchange intramedullary nailing of delayed union and nonunion of the tibial shaft. The

    authors recommend the use of bone graft only when there is substantial bone loss, usually exceeding 30% of the cortical diameter.

     

     

     

    OrthoCash 2020

     

  98. A 28-year-old male sustains a fall on icy ground and fractures his ankle. An intraoperative fluoroscopy image is shown in Figure A. This fixation suggests that the mechanism of injury was one of

     

     

     

     

    1. Pronation-abduction

    2. Supination-adduction

    3. Pronation-external rotation

    4. Supination-external rotation

    5. Dorsiflexion-axial loading

     

    Corrent answer: 2

     

    The fixation of the medial malleolus in the figure consists of a contoured reconstruction plate placed in buttress (antiglide) mode with screws running parallel to the joint surface and perpendicular to the fracture line. This fixation is indicated for vertical fractures of the medial malleolus from a supination-

    adduction (SA) shearing force.

     

    A vertical fracture of the medial malleolus is characteristic of a Lauge-Hansen SA fracture. Bimalleolar SA fractures will also demonstrate a distal fibular avulsion. Another characteristic is impaction of anteromedial distal tibia by the talar dome. An anteromedial surgical approach allows visualization of this region, elevation of impaction, and bone grafting of the defect if necessary.

     

    Dumigan et al. analyzed 4 different methods of treating vertical shear fractures with polyurethane models. They found that fixation with a 4 hole 1/3 tubular plate with 2 screws in the distal fragment was stiffer than fixation with the same plate with 1 screw in the distal fragment (3 in the proximal), fixation with two 3.5mm cortical screws with washers, and fixation with two 4mm cortical screws with washers. They recommend plating over screw-alone constructs.

     

    McConnell et al. reported 8 cases of anteromedial marginal impaction in SA fractures. They note that 44 fractures were SA type. Following reduction and fixation, all 8 patients had excellent outcome and no arthritis. They recommend an anteromedial approach to the medial malleolus for visualization.

     

    Figure A is a lateral radiograph showing fixation of both malleoli with reconstruction plates. Illustration A is an postoperative AP fluoroscopy image. Illustration B is an injury film demonstrating the same SA ankle fracture.

    Illustration C shows articular impaction at the medial shoulder of the ankle joint typical of SA injuries. Illustration D shows marginal impaction in SA fractures.

     

    Incorrect Answers

    Answer 1: Bimalleolar pronation-abduction fractures comprise an avulsion fracture of the medial malleolus and comminuted fibula fracture proximal to the syndesmosis (with a butterfly fragment).

    Answer 3: Bimalleolar pronation-external rotation fractures comprise a transverse medial malleolar fractures and a spiral fibula fracture (anterosuperior to posteroinferior) above the syndesmosis.

    Answer 4: Bimalleolar supination-external rotation fractures comprise a transverse medial malleolar fracture and short oblique fibula fracture (anteroinferior to posterosuperior).

    Answer 5: Axial loading leads to pilon fractures. A dorsiflexed position leads to anterior plafond injury.

     

     

     

     

     

     

     

     

     

     

     

     

    OrthoCash 2020

     

  99. A 21-year-old woman is struck by a car and sustains a Gustillo IIIB fracture of the tibia. The wound was debrided and immobilized with an external fixator. Radiographs are shown in Figure A. The soft tissue defect was covered with a free flap. Her recovery was complicated by wound infection with Klebsiella pneumoniae and Escherichia coli. One month after her injury, she underwent intramedullary nailing and placement of an antibiotic spacer measuring 15cm in length. Radiographs are shown in Figure B. At the next stage of surgery 6 weeks later, the surgeon should plan to do all of the following:

     

     

     

     

     

    1. Excise the spacer

    2. Excise the spacer, debride all membranous tissue, perform exchange nailing

    3. Excise the spacer, debride all membranous tissue, bone graft the cavity

    4. Excise the spacer, preserve all membranous tissue, bone graft the cavity

    5. Excise the spacer, preserve all membranous tissue, bone graft the cavity, remove the nail

    Corrent answer: 4

     

    The second stage of the Masquelet technique requires removal of the cement bolus, incision into the induced membranes and bone grafting. The existing hardware is preserved where possible as the fracture is still not stable. Bone graft is inserted INTO the membranous cavity, AROUND the nail.

     

    The Masquelet staged technique of induced membranes is an option for filling large bone defects up to 25cm in length. This technique protects against autograft resorption, stimulates mesenchymal cell-to-osteoblast differentiation, maintains graft position, and prevents soft tissue interposition. Cement impregnation achieves high local antibiotic concentration without risk of systemic toxicity.

     

    Ashman et al. discussed the techniques of addressing bone defects. Options include: (1) acute limb shortening (up to 4cm in the tibia and humerus, and 7cm in the femur); (2) distraction osteogenesis for defects up to 10cm long (at 1mm/day with consolidation period of 5days per mm, or total treatment time of up to 60days/cm), (3) autograft (up to 25cm of vascularized fibula, or 3cm of nonvascularized iliac crest), and (4) Masquelet technique.

     

    Taylor et al. reviewed the induced membranes technique. They found that the membrane is well vascularized and composed of type I collagen with fibroblasts with an inner epithelial cell layer. There is a high concentration of VEGF, RUNX2 (CBFA1), TGFß1, and BMP2. The membrane is sutured over bone graft to create a closed pouch. When a nail is present, they note a second internal membrane around the nail, potentially increasing local vascularity and osteoinductive factor concentration.

     

    Figure A shows a Gustillo IIIB tibia fracture with a large bone defect held in a temporizing external fixator. Figure B shows the same defect following intramedullary nailing and with a cement spacer placed circumferentially around the nail in the defect.

     

    Incorrect Answers

    Answer 1: Excising the spacer alone will leave a cavity around the nail. Small amounts of bone form on the inside of the membrane, but without bone

    grafting, bone will not bridge the gap. Bone graft should be inserted into the cavity at the same sitting as cement removal.

    Answers 2 and 3: The membrane prevents bone graft resorption and contributes growth factors. Debriding the membrane will negate these effects. Exchange nailing will result in a period of instability and may compromise the membrane.

    Answer 5: The nail cannot be removed as yet because there is no fracture site stability.

     

     

     

    OrthoCash 2020

     

  100. A 38-year-old man is involved in a moderate speed motor vehicle collision. He is hemodynamically stable in the emergency room. He is noted to have a single right-sided rib fracture, left clavicle fracture, and the injury depicted in figures A-D. The injury is closed and he is neurovascularly intact. He is placed in a knee immobilizer. The next morning there is moderate swelling and fracture blisters on the lateral aspect of the knee. What is the next best step in management?

     

     

     

     

     

     

     

     

     

     

     

    1. Long leg splint

    2. Bivalved long leg cast

    3. Knee spanning external fixation

    4. ORIF with lateral plate only

    5. ORIF with lateral and medial plate Corrent answer: 3

    The patient has a bicondylar tibial plateau fracture extending to the diaphysis (Schatzker VI). Given his swelling and fracture blisters, the most appropriate next step would be knee-spanning external fixation.

     

    High energy injuries of the tibia are often accompanied by significant soft tissue damage and swelling. Tibial plateau fractures, and more often pilon fractures, with significant swelling or fracture blisters are best managed acutely with external fixation to allow swelling to improve before definitive fixation. This strategy, which typically applies more often to pilon fractures, helps to limit infection and wound healing complications.

     

    Reahl et al. retrospectively reviewed 419 patients who underwent surgical management of tibial plateau fractures to determine risk factors for subsequent surgery for knee stiffness. They found that amount of time spent in external fixation and bilateral tibial plateau fractures were independent risk factors for need for later surgery for knee stiffness.

     

    Egol et al. investigated a staged treatment protocol for high energy tibial plateau fractures. Initial treatment was with knee-spanning external fixation followed by definitive fixation at an average of 15 days later. While wound complications were low (5%), they do cite a potential downside of increased knee stiffness.

     

    Figures A-D are knee x-rays and CT scan showing a bicondylar tibial plateau fracture with significant comminution.

     

    Incorrect Answers:

    Answer 1,2: A long leg splint or bivalve cast will not help maintain length and is not the most appropriate next step. Acute casting of a high energy fracture is not recommended

    Answer 4,5: The presence of fracture blisters and soft tissue swelling indicate that immediate ORIF is not the most appropriate next step.

     

     

     

    OrthoCash 2020

     

  101. The dominant arterial blood supply to the patella enters at which anatomical location?

    1. Proximal pole

    2. Mid-lateral

    3. Mid-medial

    4. Directly anterior

    5. Distal pole

     

    Corrent answer: 5

     

    The largest arterial contribution to the patella will enter at the distal (inferior) pole of the patella, with the dominant artery entering inferomedially.

     

    The arterial blood supply to the patella is made up of branches of six main arteries: the descending genicular, the superior medial and lateral genicular, the inferior medial and lateral genicular, and the anterior genicular. Several of these branches contribute to the anastomotic network that surround the patella. From the ring, there are two main interosseous blood supply systems to enter the patella, known as the midpatellar and polar vessel systems. The distal pole of the patella is considered to be the largest arterial contribution to the peripatellar ring and the polar vessel system.

     

    Lazaro et al. used twenty matched pairs of fresh-frozen cadaveric knees to isolate the dominant blood supply to the patella. After cannulating the superficial femoral artery, anterior tibialis artery, and posterior tibialis artery and performing magnetic resonance imaging, they found that the largest arterial contribution to the patella entered at the inferior pole in 100% of the specimens. In sixteen specimens (80%), the dominant artery entered the medial aspect of the distal pole. In three specimens (15%), it entered the lateral aspect of the distal pole.

     

    Illustation A shows the arterial supply system to the patella. The dominant arterial supply enters at the distal (inferior) pole of the patella, with the dominant geniculate arteries entering inferomedially (labelled with a green star). Illustration B shows an anatomical illustration of the patellar blood supply. Note the dominant distal pole blood supply (arrow).

     

    Incorrect Answers:

    Answers 1-4: Two main arterial systems (midpatellar and polar vessel systems) form the intraosseous blood supply of the patella, which are supplied by the medial/lateral and superior/inferior genicular branches, respectively.

    The dominant arterial supply comes from the inferior branches.

     

     

     

     

     

     

     

     

    OrthoCash 2020

     

  102. Which of the following treatment options is the most rigid fixation for the fibula?

     

     

     

     

     

     

     

     

     

     

    1. Figure A

    2. Figure B

    3. Figure C

    4. Figure D

    5. Figure E

     

    Corrent answer: 4

     

    Figure D shows lag screw fixation neutralized with a locking plate. Plates that allow locking screw fixation create a fixed-angle construct with inherently high stiffness.

     

    Fixation options for fibular fractures vary widely with many possible implant types and many different ways to utilize each of those implants. Fibula fixation with locking plates can be advantageous in the setting of poor bone quality or extremely distal fracture fragments with small amounts of available bone.

    Locking plates are generally stiffer implants however techniques of widely spacing screws can decrease the construct stiffness. The disadvantages of locking plates for the fibula are hardware prominence and excessive stiffness leading to nonunion.

     

    Siegel et al. reviewed 31 patients with pronation-abduction ankle injuries who were treated strictly with extraperiosteal plating of the fibula. They found a 100% healing rate with no deep infections and average AOFAS of 82. They concluded extraperiosteal plating of the fibula in pronation-abduction ankle fractures leads to predictable fibular union.

     

    Bottlang et al. reviewed how locked plate stiffness can affect bone healing and if that stiffness can be decreased by various techniques. They found that interfragmentary motion is attenuated at the near cortex and that far cortical locking screws allow for more motion across the fracture site and greater amounts of callus formation. They concluded their research supports locked plates can be too stiff and that far cortical locking screws reduce construct stiffness and improves bone healing.

     

    Figure A shows a fibular nail without syndesmotic fixation. Figure B shows a 1/3 tubular plate acting as an anti-glide plate. Figure C shows a 1/3 tubular plate with non-locking screws acting as a bridging construct. Figure D shows an example of an a lag screw with a locking plate used as a neutralization plate. Figure E is an example of an isolated lag screw fixation of a distal fibula fracture without a neutralization plate.

     

    Incorrect Answers:

    Answer 1: A fibular nail which is an intramedullary device and not as stiff as a

    locking plate.

    Answer 2: An anti-glide plate is not as stiff a construct compared to a locking plate.

    Answer 3: A bridge plate using non-locking screws is not as stiff of a construct as a bridge plate with using locking screws.

    Answer 5: Isolated lag screws without a neutralization plate would be less rotationally stable.

     

     

     

    OrthoCash 2020

     

  103. Which of the following is TRUE regarding intimate partner violence (IPV)?

    1. Physicians always have a duty to report intimate partner violence

    2. Elderly women are more likely to be victims than elderly men

    3. Intimate partner violence among patients is 4 times more prevalent than orthopaedic surgeons perceive

    4. Victims do not typically seek medical attention for injuries related to abuse

    5. Risk of abuse is unrelated to socioeconomic status Corrent answer: 3

    The actual prevalence of intimate partner violence (IPV) among patients presenting to an orthopaedic clinic has been described as high as 40%, while surgeon-perceived prevalence of IPV is only about 10%.

     

    Domestic abuse in the form of IPV, elder abuse, or child abuse must be considered and evaluated appropriately for any patients presenting with an injury. Red flags that an injury may be secondary to abuse include inconsistent history, repeated presentation to the ER or clinic, multiple injuries of different ages, delay in seeking care, changes in behavior or affect, and many others.

    Victims of abuse often present multiple times before the proper diagnosis is made.

     

    Bhandari et al. present a survey done of Canadian and American orthopaedic surgeons on their attitudes and perceptions of domestic abuse. Surgeons believed that knowledge regarding domestic abuse was important to their practice, but felt less than 10% of the patients they see are victims of abuse, and that mandatory abuse screenings are unnecessary.

     

    The PRAISE investigators did a survey of women presenting to orthopaedic clinics regarding IPV. They found a lifetime prevalence of IPV among these women of 33%, and that only 14% of these women were ever asked about IPV or the safety of their home environment in a health care setting.

    Della Rocca et al. present a survey of orthopaedic surgeons about domestic abuse. Misconceptions they found common among their respondents included that victims may be getting something out of the abusive relationship, that some women have personalities that foster abuse, and that abuse would stop if the abuser quit using alcohol. Only 4% of surgeons who completed the survey reported currently screening women for IPV.

     

    Incorrect answers:

    Answer 1: Physicians do not always have a duty to report IPV. In fact, in many states IPV can not be reported without the patients consent. Doctors do typically, however, have a duty to report child abuse and elder abuse. If IPV is suspected the patient should be appropriately screened, offered resources, and encouraged to seek help from proper authorities. Exact reporting laws and responsibilities vary by state.

    Answer 2: Gender is not a risk factor for elder abuse.

    Answer 4: Victims of abuse are commonly seen multiple times for treatment of their injuries. Multiple presentations to the ER or clinic is a red flag for abuse. Answer 5: Socioeconomic status is a known risk factor for abuse.

     

     

     

    OrthoCash 2020

     

  104. Prescribing touch (10 to 15 kg) weight-bearing would be most appropriate in the following scenario?

    1. Acute grade II anterior talofibular ligament ankle sprain

    2. Partial lateral menisectomy for incomplete radial tear

    3. Open reduction internal fixation for comminuted calcaneus fracture

    4. Cemented hemiarthroplasty for displaced femoral neck fracture

    5. Open reduction internal fixation for transtectal transverse posterior wall fracture

    Corrent answer: 5

     

    Touch weight bearing (10 to 15 kg) regimens have shown to minimize joint reaction forces across the hip. This weight bearing restriction should be considered in patients who have undergone open reduction internal fixation of transtectal transverse posterior wall fractures.

     

    The definition of touch weight bearing (also known as touch-down weight bearing) is ill-defined in the literature. Published data suggest touch weight bearing to be 10 to 15 kg of load applied to ground by the affected limb or less than 20% of body weight. In contrast, partial weight bearing is reported as 20 to 25 kg or 30% to 50% of body weight. Joint reaction forces across the hip

    have been shown to be lowest with touch weight-bearing. In this scenario, the foot should be flat against the ground so the flexor and extensor musculature that cross the hip are relaxed. With non-weight bearing restrictions, the musculature across the hip will be contracted, which increases contact pressures and joint reaction forces.

     

    Rubin et al. looked at the validity of touch weight-bearing and partial weight bearing regimens. They found that most patients overload the limb up to 50% more than the target weight prescribed.

     

    Lewis et al. showed that maintaining non-weight-bearing position of the involved leg produces increased compressive forces across the hip joint due to activation of the hip flexors compared to restricted weight-bearing.

     

    Incorrect Answers:

    Answer 1: Acute grade II lower ankle sprain can be managed with immediate full weight bearing as tolerated.

    Answer 2: Partial lateral meniscectomy for an incomplete radial tear can be managed with immediate full weight bearing as tolerated.

    Answer 3: Multifragmentary calcaneus fracture open reduction internal fixation should be managed initially with non-weight bearing.

    Answer 4: Uncomplicated cemented hemiarthroplasty for hip fracture can be managed with immediate full weight bearing as tolerated.

     

     

     

    OrthoCash 2020

     

  105. A 24-year-old male presents following a motorcycle crash with an isolated injury to his right lower extremity. He has a 3x2cm wound over the fracture site, and he immediately receives Gram positive and Gram negative coverage along with a tetanus booster. The patient is splinted, optimized, and brought to the operating room where the wound is debrided and classified as a Type IIIB fracture. Deemed stable, the plastic surgery team arrives and acutely performs a free flap for coverage, following definitive fixation with an intramedullary nail. All of the following are factors that have been shown to increase infection risk EXCEPT:

     

     

     

    1. Time to antibiotic administration

    2. Thoroughness of debridement

    3. Time to initial debridement

    4. Ability to close/cover an open wound

    5. Time to definitive fixation Corrent answer: 5

    Time to definitive fixation is not a modifiable risk factor concerning open fractures. The other factors are risk factors that have been studied in regards to infection, and all are more important than definitive fixation. Definitive fixation can wait until complete closure and/or coverage.

     

    When concerning management of open fractures, the most important factor is a thorough debridement. However, the quality of debridement is often not able to be quantified and thus, often not mentioned in studies. While early clinical and animal studies have shown that initial debridement should occur within 6 hours of injury, more recent clinical trials have not found a significant correlation within that urgent time frame, but rather recommend initial debridement as soon as possible within 24 hours. Time to antibiotic administration has been found to have a significant impact in lowering infection risk. Immediate administration in the emergency room is recommended. The ability to cover and/or close an open wound also has a significant impact on infection. Recent studies have recommended placing hardware after fasciotomy closure and have also demonstrated lower infection rates when flaps are placed

    within 72 hours of injury.

     

    Pape and Webb concisely review the evolution of open fractures and wound management. The authors describe the early days where amputation was favored, to wet-to-dry dressings, to the advent of negative pressure wound therapy. Throughout, however, the authors emphasize the importance of soft tissue coverage. They also stress the importance of a technically thorough debridement, the most important factor of any wound management.

     

    Scheneker et al. performed a systematic review and meta-analysis of 16 studies to determine if time to the operating room for debridement was an independent, modifiable risk factor in regards to subsequent infection following open tibia fracture. At the time of the study, the gold standard (based on a previous rat model), had recommended initial debridement within 6 hours of injury. The results of this meta-analysis, however, could not find conclusive evidence to suggest that late debridement alone placed the patient at a significantly higher risk for infection. The authors provided a moderate recommendation that initial debridement should occur as soon as possible within 24 hours, although more data is required in order to find a definitive time.

     

    The SPRINT investigators report a landmark study that randomized over 1200 patients to either reamed or unreamed tibial IMN with the primary outcome analyzed as return to the operating room for either non-union treatment or deep infection. A notable difference between the two cohorts was a significantly higher primary event rate in the unreamed group.

    Figure A exhibits a distal third open tibia fracture. Incorrect answers:

    Answer 1: Antibiotic administration as soon as an open fracture has been

    diagnosed is a significant risk factor in minimizing infection risk.

    Answer 2: Although a non-quantifiable measure, a thorough debridement is the most important component of treating an open fracture.

    Answer 3: Initial animal models cite a 6 hour window to initial debridement, however, clinical trials have not found a significant window that can affect increased or lowered infection risk.

    Answer 4: Coverage and/or closure of any open wounds or soft tissue defects is a significant factor in lowering infection risk; when flap coverage is needed, coverage within 72 hours is optimal.

     

     

     

    OrthoCash 2020

  106. Which of the following activities produces greater hip joint contact pressures than full weight bearing during normal gait?

    1. Performing isometric hip exercises

    2. Getting on a bed pan

    3. Ambulating with a cane

    4. Rising from a chair using the affected leg

    5. Toe-touch weight bearing with passive hip abduction Corrent answer: 4

    Rising from a seated position on a chair on the affected leg has been shown to create the highest contact pressure within the hip, even higher than full weight-bearing during walking.

     

    Limiting activities that create high contact pressures is important in situations such as after internal fixation of an acetabular fracture. Full weight bearing during a normal gait cycle is often considered too much contact pressure and considered a risk for early failure of fixation. During the postoperative period weight bearing and activities are limited to prevent this. It has been shown that the highest contact pressures, even higher than normal walking, are seen when rising from a chair on the affected leg.

     

    Brand et al. analyzed joint reactive forces in patients walking with and without a cane. Compared to age matched controls they estimate that using a cane decreases the contact pressure in the hip to about 60% of normal.

     

    Hodge et al. looked at data from an implanted hip prosthesis with pressure sensors. They found that some activities common to the early rehabilitative period, such as using a bed pan and performing isometric exercises about the hip, can create pressure approaching those of normal walking. The highest pressures recorded were when rising from a chair.

     

    Incorrect answers:

    Answers 1 and 2: Isometric hip exercises and using a bed pan cause contact pressure approaching that of normal walking, but not exceeding it.

    Answer 3: Ambulating with a cane reduces the contact force of the hip to about 60% of normal walking.

    Answer 5: Toe touch weight bearing with passive hip abduction significantly decreases the contact pressures in the hip. Toe-touch weight bearing with passive abduction creates even lower joint reactive force than complete non-weight bearing status.

     

     

    OrthoCash 2020

     

  107. A 25 year-old-male sustains a closed injury shown in Figure A. If a tibial intramedullary nail is placed with the starting points shown (arrows), what subsequent alignment will occur?

     

     

     

     

    1. Neutral

    2. Varus, apex anterior

    3. Varus, apex posterior

    4. Valgus, apex anterior

    5. Valgus, apex posterior Corrent answer: 4

    In proximal third tibial shaft fractures, due to the deforming forces of the pes anserine and the extensor mechanism, utilizing standard starting points during intramedullary nailing (IMN) will result in a valgus and apex anterior deformity.

     

    There are several tips and tricks to avoid subsequent deformity following tibial IMN of a proximal third fracture. One way to avoid deformity is to use a more lateral starting point than normal to ensure nail placement in the true center of the canal, which is more lateral when compared to the tibial plateau.

     

    Walker et al. studied 12 cadaveric tibias and inserted a Kirschner wire depending on rotated views of the knee. In order obtain a perfect starting point, a perfect anteroposterior as well as lateral of the knee must be obtained; otherwise, the authors noted that malrotation is bound to occur. With a perfect view, a more lateral starting point correlated with the center of the tibial canal.

    McConnell et al. studied cadaveric and subsequent radiographic correlation on a lateral knee x-ray to determine the ideal 'safe zone' for the starting point of a tibial nail. This safe zone is more lateral and posterior, when looking at the axial cut of the plateau.

     

    Figure A exhibits a proximal third tibia fracture with starting points that are not lateral enough, and too distal (on the lateral view), which will result in apex anterior and valgus deformity.

     

    Incorrect answers:

    Answer 1 (neutral alignment) will occur if a more lateral and more proximal start point is used.

    Answers 2,3, and 5 are deformities that will typically be seen following tibial IMN if desired start points are utilized.

     

     

     

    OrthoCash 2020

     

  108. A 30-year-old female involved in a severe motor-vehicle collision that requires prolonged extrication. She arrives at a referral trauma center almost 10 hours after her initial injury. She receives tetanus and intravenous antibiotics upon arrival. The patient has an open tibial fracture with significant periosteal stripping and a closed head injury that requires intracranial pressure monitoring. She is cleared for operative intervention by the neurosurgery and trauma surgery services the following morning. She undergoes a thorough debridement, placement of an antibiotic bead pouch, and external fixator placement approximately 18 hours after her injury. She is definitively treated 4 days after her injury with a repeat debridement, gracilis flap and intramedullary nail. Which of the following factors places the patient at increased risk of infection?

    1. Free tissue transfer instead of rotational flap

    2. Flap coverage at four days after injury

    3. Use of an intramedullary nail instead of minimally invasive plating

    4. Delay in administration of IV antibiotics

    5. Debridement at 15 hours after injury Corrent answer: 4

    Intravenous antibiotics are critical to prevent infection in open fractures. Delay in administration of intravenous antibiotics has been linked with increased risk of infection.

     

    Open tibia fractures are associated with high rates of infection. Historically,

    early debridement (within 6-8 hours) and early flap coverage (typically defined as less than 72 hours) were thought to minimize the risk of infection. Recent evidence has challenged these findings, with multiple studies demonstrating no significant link between the timing of debridement and rates of infection.

    Multiple studies from the Lower Extremity Assessment Project (LEAP) found no significant difference in infection or complication with flap coverage more than 72 hours after injury.

     

    Bhattacharyya et al retrospectively evaluated patients with type IIIB tibial fractures treated with extended use of negative pressure wound therapy. The authors found increased rates of infection beyond 7 days despite the use of negative pressure wound therapy.

     

    Lack et al evaluated the timing of antibiotic administration on infection rates for type III tibial fractures. The authors found increased rates of infection with administration of antibiotics beyond 66 minutes. The authors discuss the possibility of pre-hospital intervention as many severely injured patients have delayed arrival at treatment centers.

     

    Pollak et al prospectively analyzed rates of complication with flap coverage as part of the LEAP study. The authors found no increase in complications with flap coverage beyond 72 hours. The only significant risk for complication was the use of rotational flaps in comminuted or segmental (AO/OTA type C) tibial fractures.

     

    Incorrect answers:

    Answer 1: Lower rates of complication, including infection, were seen with free flaps in AO/OTA type C fractures in the study by Pollak et al.

    Answer 2: Timing of flap coverage is controversial. Early studies demonstrated increased infection with delay beyond 72 hours, however recent studies using more rigorous statistical analysis do not support these findings.

    Answer 3: No study has demonstrated lower infection rates with the use of plating versus nailing in open tibial fractures.

    Answer 5: The timing of debridement with open fractures has been shown not to effect the rate of infection in multiple recent studies.

     

     

     

    OrthoCash 2020

     

  109. Recombinant human bone morphogenetic protein-2 (rhBMP-2) is US Food and Drug Administration (FDA) approved for which of the following indications?

     

     

     

     

     

     

     

     

     

     

    1. Acute open injury shown in Figure A (left), undergoing procedure shown in Figure A (right)

    2. Acute open injury shown in Figure B (left), undergoing procedure shown in Figure B (right)

    3. 8-month-old condition shown in Figure C (left), undergoing procedure shown in Figure C (right)

    4. 8-month-old condition shown in Figure D (left), undergoing procedure shown in Figure D (right)

    5. Acute open injury shown in Figure E (left), undergoing procedure shown in Figure E (right)

    Corrent answer: 5

     

    For long bones, rhBMP-2 is FDA approved for acute open diaphyseal fractures of the tibia treated with intramedullary (IM) nail fixation.

     

    rhBMP-2 is approved for repair of posterolateral lumbar pseudarthrosis, anterior lumbar interbody fusions (ALIFs) with a lumbar fusion device, open tibia shaft fractures with intramedullary nail fixation. rhBMP-7 is approved under humanitarian device exemption as an alternative to autograft in long bone nonunions and as an alternative to autograft for posterolateral fusion.

     

    Govender et al. compared 3 groups of patients: tibia fractures receiving IM nail (standard of care), IM nail with an implant containing 0.75mg/ml of rhBMP-2, and IM nail with an implant containing 1.5mg/ml of rhBMP-2. The implants were collagen sponges. Patients receiving IM nail with 1.5mg/ml of rhBMP-2 had reduced risk of delayed union, fewer interventions (bone-grafting and nail exchange), faster fracture healing, fewer hardware failures and infections.

    They concluded that rhBMP-2 was superior to the standard of care.

     

    Garrison et al. reviewed 11 trials. Four trials showed that BMP treated acute fractures required fewer subsequent procedures. Six trials showed that BMP was neither better nor worse than bone grafts for nonunion. One trial showed no difference between BMP and bone grafts for osteotomies.

     

    Figure A shows a mid-diaphyseal femur fracture treated with an intramedullary nail. Figure B shows a mid-diaphyseal femur fracture treated with a plate.

    Figure C shows a mid-diaphyseal femur fracture nonunion treated with a second supplementary anterior plate. Figure D shows a mid-diaphyseal tibia fracture treated with exchange nailing. Figure E shows a mid-diaphyseal tibia fracture treated with an intramedullary nail.

     

    Incorrect Answers:

    Answers 1 to 4: rhBMP2 is not approved for long bone fractures except for

    acute open diaphyseal fractures of the tibia treated with intramedullary (IM) nailing.

     

     

     

    OrthoCash 2020

     

  110. A 20-year-old woman is involved in a high-speed motor vehicle collision and sustains bilateral tibial plateau fractures as well as the clavicle fracture shown in Figure A. What is the most appropriate management of the clavicular injury?

     

     

     

     

    1. Closed reduction and figure of 8 splinting

    2. Open reduction and plate fixation

    3. Open reduction and percutaneous pinning

    4. Simple sling to involved side

    5. Sling with abduction pillow to involved side Corrent answer: 2

    The radiograph shows a comminuted clavicle fracture with significant displacement. Indications for surgical fixation of clavicle fractures include: open fractures, underlying neurovascular injury, or impending open fracture from internal bony pressure causing skin compromise. Relative indications for fixation include: greater than 15 mm of shortening, greater than 100% displacement (no bony contact), highly comminuted fractures, and polytrauma patients.

     

    Jeray et al review the evaluation and treatment of midshaft clavicle fractures. They state "when midshaft clavicular fractures are completely displaced or comminuted, and when they occur in elderly patients or females, the risk of nonunion, cosmetic deformity, and poor outcome may be markedly higher.

    Thus, some surgeons propose surgical stabilization of a complex midshaft

    clavicular fracture with either plate-and-screw fixation or intramedullary devices. Further randomized, prospective trials are needed to provide better data on which to base treatment decisions."

     

     

     

    OrthoCash 2020

     

  111. A 32-year-old male sustains the injury seen in Figure A. His blood pressure preoperatively was 132/84. After closed reduction and placement of an intramedullary nail, his intraoperative leg compartment pressures are measured, with the highest being 28 mmHg. His blood pressure at this time is 84/57. What is the next appropriate step?

     

     

     

     

    1. Immediate four compartment fasciotomy

    2. Fasciotomy of the highest compartment(s)

    3. Removal of the nail and placement of an external fixator

    4. Repeat evaluation in recovery room

    5. Addition of pressors to anesthesia Corrent answer: 4

    Figure A shows a mildly comminuted tibia fracture, which is a fracture highly associated with compartment syndrome. However, in this scenario, the delta p (difference between compartmental pressures and diastolic pressure) is greater than 30 preoperatively, with a decrease to less than 30 intraoperatively, due to the hypotension associated with anesthesia.

     

    The referenced article by Kakar et al notes that the delta p may be spuriously

    low intraoperatively, and with tibial nailing, it is safe to assume the delta p will return to a higher level postoperatively. They recommended continued monitoring in the postoperative period with clinical examination and measurements as needed.

     

    The McQueen referenced article showed that the delta p is more important than absolute pressures, as an absolute threshold of 30mmHg would have led to unnecessary fasciotomies in 43% of their cohort.

     

     

     

    OrthoCash 2020

     

  112. Figures A and B demonstrate a proximal tibial metaphyseal fracture which will be treated with an intramedullary nail. Placing blocking screws at the sites marked with x's in the figures would help to prevent what type of malreduction deformity?

     

     

     

     

    1. Valgus and procurvatum

    2. Valgus and recurvatum

    3. Varus and procurvatum

    4. Varus and recurvatum

    5. Varus and external rotation

    Corrent answer: 1

     

    Placing blocking screws at the sites shown would help to prevent a valgus and procurvatum deformity. This represents the most common pattern of malreduction for this fracture pattern.

     

    Proximal tibial metaphyseal fractures being treated with an intramedullary nail are known to commonly fall into a valgus and procurvatum deformity due to lack of cortical fit in the metaphyseal segment. To help prevent this deformity blocking screws may be used. As a general rule, a blocking screw should be placed in the concavity of the deformity you intend to prevent. Thus, to prevent a valgus deformity you would place a screw just lateral to the nail.

    Likewise, to prevent a procurvatum deformity you would place a blocking screw just posterior to the nail. These screws would provide a buttress to keep the metaphyseal segment in line with the tibial shaft.

     

    Ricci et al. present a prospective cohort of 12 patients who presented with a proximal tibial metaphyseal fracture treated with an intramedullary nail and blocking screws. They found that all 12 patients went on to heal with less than 5° of angulation in all planes in which a blocking screw was used. They concluded that blocking screws are effective at maintaining proper alignment for this type of fracture being treated with an intramedullary nail.

     

    Hiesterman et al. present a review on treatment techniques for extraarticular proximal tibia fractures treated with an intramedullary nail. They note that valgus and apex anterior deformities are most common. They cite several techniques for preventing this deformity including alternate patient positioning or approach, a universal distractor, unicortical plates, and blocking screws.

     

    Figure A is an AP radiograph of a proximal third tibia fracture with a marking to indicate placement of a blocking screw just lateral to the central axis of the tibia. Figure B is a lateral radiograph of a proximal third tibia fracture with a marking to indicate placement of a blocking screw just posterior to the central axis of the tibia. Illustration A and B are AP and lateral radiographs of the tibia fracture seen above treated with an intramedullary nail and no blocking screws. Note the resulting valgus and procurvatum deformity about the fracture even with the nail in place. Illustrations C and D are diagrams illustrating the use of lateral and posterior blocking screws to properly align the fracture.

     

    Incorrect Answers:

    Answers 2, 3, 4, and 5: These do not correctly identify the deformity that would be prevented with locking screws placed lateral and posterior to the

    intramedullary nail.

     

     

     

     

     

     

     

     

     

     

     

    OrthoCash 2020

     

  113. A 78-year-old community ambulator without assistive devices suffers a displaced femoral neck fracture and is treated acutely with a hemiarthroplasty. He is discharged to a skilled nursing facility, and follows up in your clinic 4 weeks after his surgical date. He brings a report from the physical therapist that states the patient's "Timed up and Go Test" is 11 seconds. This score is predictive of which of the following:

    1. Recurrent geriatric fragility fracture within 5 years

    2. Return to ambulation without assistive device at 2 years

    3. Life expectancy > 20 years

    4. Persistent use of front wheel walker after surgical recovery

    5. Cardiopulmonary compromise without supplemental oxygen Corrent answer: 2

      The Timed Up and Go test can be used as a quantitative assessment of a patients functional status. A score of < 12 seconds is normal in a population of adults of all ages.

       

      The timed up and go test requires a patient to rise from a chair, walk around a cone 10 feet away, and return to the seated position in the same chair (see video below). It is a commonly used assessment of functional status across clinical settings (pulmonary, cardiac, musculoskeletal, etc.). It has been used in the hip fracture literature and found to be most predictive of use of ambulatory aids.

       

      Laflamme et al. found that the timed up and go test at 3 wks following

      hemiarthroplasty for hip fractures is predictive of ambulatory aid use two years after the fracture. Specifically, a test score of greater than 26 seconds predicted a 90x higher chance of using an ambulatory device.

       

      Kristensen et al. established high interrater reliability with an ICC of 0.95 in a group of fifty hip fracture patients evaluated with the timed up and go test.

       

      The video link below shows a patient performing the test without an ambulatory aid. It can be performed with one as well.

       

      Incorrect Answers:

       

      Answers 1, 5: The patient has a normal timed up and go test within normal limits. The FRAX score calculation would be a better tool for future fragility fracture risk. Abnormal scores are indicative of cardiac and pulmonary system compromises, but this question is not referring to these clinical settings.

      Answer 3: While gait speed is predictive of overall longevity, this has not been evaluated beyond 20 years in the geriatric hip fracture population.

      Answer 4: Persistent use of ambulatory aids would be predicted if his score had been > 26 seconds.

       

       

       

      OrthoCash 2020

       

  114. Figures A-C are images of a 37-year-old man who presents with isolated muscle atrophy due to a compressed nerve. Which of the following sequences correctly describes the pathway of this nerve through the brachial plexus, before it innervates the affected muscles?

     

     

     

     

     

     

     

     

    1. C5-C7 nerve roots; upper/middle trunks; anterior division; lateral cord

    2. C5-C6 nerve roots; upper trunk; posterior division; posterior cord

    3. C5-C6 nerve roots; upper trunk

    4. C5-C7 nerve roots

    5. C8-T1 nerve root; lower trunk; anterior division; medial cord Corrent answer: 2

    Figures A-C shows atrophy of teres minor and deltoid due to compression of the axillary nerve. The correct pathway of the axillary nerve within the brachial plexus is, C5-C6 nerve roots; upper trunk, posterior division, posterior cord.

     

    Quadrilateral space syndrome is a condition defined by axillary nerve, +/-posterior humeral circumflex artery compression in the quadrilateral space. It most commonly affects the dominant shoulder in overhead movement athletes (e.g. basketball players) or other throwing athletes. Physical examination may reveal weakness with the arm positioned in abduction and external rotation. In situations of long-standing compression, there may also be atrophy of the teres minor and deltoid muscle.

     

    Chafik et al. dissected thirty-one cadaveric human shoulders to describe the neuromuscular anatomy of teres minor. They showed that the primary nerve branch to teres minor travelled in a fascial sling 44 mm medial to the muscular insertion. This area may be the potential site of greatest compression and tethering of this nerve in patients with isolated teres minor atrophy.

    Friend et al. performed a cadaveric dissection of nine shoulder specimens to look at the anatomical variability in course, length and branching pattern of both the teres minor nerve and the axillary nerve. These were compared to a case-based study of these two male patients with isolated atrophy of teres minor. They concluded that there is no good anatomical predictor of nerve compression outside the quadrilateral space as there is considerable anatomical variation in its origin and course, as well as potential site of compression.

     

    Figure A-C are MRI images that show atrophy of the teres minor muscle and possibly deltoid muscle. The rotator cuff muscles are labeled in Illustration A. The teres minor muscle is labeled in Illustration B. Illustration C shows a diagram of the brachial plexus.

     

    Incorrect Answers:

    Answer 1: This describes the musculocutaneous nerve. Answer 3: This describes the suprascapular nerve.

    Answer 4: This describes the long thoracic nerve. Answer 5: This describes the ulnar nerve.

     

     

     

     

     

     

     

     

     

     

    OrthoCash 2020

     

  115. A 27-year-old combat veteran undergoes a late limb amputation following a blast injury. Discussing the risks and complications of the procedure, the treating surgeon discusses the risk of nonunion and bone-bridge dislocation. What type of amputation is the surgeon referring to?

    1. Fibula-pro-tibia

    2. Modified Ertl amputation

    3. Modified Burgess amputation

    4. Skew flap transtibial amputation

    5. Long posterior flap transtibial amputation Corrent answer: 2

    These complications are unique to the modified Ertl transtibial amputation.

     

    The Ertl operation was intended to enhance prosthetic end-bearing. The original Ertl amputation required a corticoperiosteal flap bridge while the modified Ertl uses a fibular strut graft. This procedure requires longer operative and touniquet times than the standard Burgess transtibial amputation. The fibula is fixed in place with cortical screws, fiberwire suture with end buttons,

    or heavy nonabsorbable sutures.

     

    Tintle et al. compared the complications of 100 modified Burgess and 37 modified Ertl amputations. There were more complications in the Ertl group, and the noninfectious complication rate was not lower. These include bone-bridge complications (32%), delayed union/nonunion (11%), and implant related complications (27%). They advise careful patient selection when choosing to perform the modified Ertl amputation.

     

    Keeling compared the functional outcomes following 38 modified Burgess and 27 modified Ertl amputations. They found no differences between treatment cohorts with regard to functional and health-related quality-of-life variables. They do not recommend routinely performing the Ertl amputation.

     

    Taylor et al. retrospectively reviewed 26 patients with Ertl and 10 with traditional transtibial amputation. The Ertl group had greater rates of return to work, decreased rates of revision, and improved Sickness Impact Profile questionnaire results.

     

    Illustration A shows a modified Ertl amputation with symptomatic fibular-sided bone-bridge nonunion. The fibula strut is fixed with TightRope (fiberwire suture with end buttons). Illustration B shows a hypertrophied fibula-pro-tibia.

    Illustration C shows the modified Burgess long posterior flap technique. Illustration D shows the skew flap technique.

     

    Incorrect Answers:

    Answer 1: Fibula-pro-tibia or fibula centralization is not an amputation but a procedure used for reconstruction of tibia defects (e.g. tumor, infection). The fibula is osteotomized proximally and distally and transposed medially keeping the entire soft tissue attachments intact and fixed proximally to the residual tibia and distally to the residual tibia. The tibia then functions as a vascularized graft and hypertrophies with loading. Microvascular skills are not required.

    Answers 3 and 5: The modified Burgess long posterior flap incision is the traditional non-bone bridging transtibial amputation. It is the most commonly performed method of below knee amputation.

    Answer 4: The skew flap incision is a type of below knee amputation.

     

     

     

     

     

     

     

     

     

     

    OrthoCash 2020

     

  116. A 29-year-old female has sustained the acute injury shown in Figure A. Which of the following is an indication for open reduction internal fixation in this patient?

     

     

     

    1. Medial sided tenderness

    2. Medial sided swelling

    3. Positive cotton test

    4. Medial clear space widening with gravity stress radiographs

    5. Positive squeeze test Corrent answer: 4

    Figure A shows a minimally displaced Weber B ankle fracture. The need for operative treatment would be dependent on fracture stability. A gravity stress test would best demonstrate fracture displacement, syndesmotic injury and medial sided ligamentous integrity.

     

    In patients who present with no medial widening on standard ankle radiographs and no clinical symptoms of deltoid ligament injury, the integrity of the deltoid ligament remains unknown. The gravity stress radiograph may be used to help identify a deltoid ligament injury in association with an isolated distal fibular fracture. Stage-IV supination-external rotation fractures, which involve the deltoid ligament, are more likely to be treated operatively as they are often considered unstable ankle fractures.

     

    Egol et al. reviewed 101 patients with isolated fibular fracture and an intact mortise. They found that medial tenderness, swelling, and ecchymosis were

    not sensitive with regard to predicting widening of the medial clear space on stress radiographs. Interestingly, they report that good functional results can be obtained in patients with widening of the medial clear space on a stress radiograph in the absence of medial signs.

     

    Gill et al. compared the effectiveness of gravity stress radiograph as compared to manual stress radiograph for the detection of deltoid ligament injury in isolated fibular fracture. A total of twenty-five patients with SER type-II fracture and SER Type IV-equivalent fractures were enrolled. They found the gravity stress radiograph was equivalent to the manual stress radiograph for determining deltoid ligament injury.

     

    Figure A shows a mortise radiograph displaying a minimally displaced Weber B ankle fracture. Illustration A shows the positioning for a gravity stress radiograph. The patient is in the lateral decubitus position with the injured leg dependent and off the end of the table, a mortise view is taken in 10° of internal rotation of the tibia.

     

    Incorrect Answers:

    Answers 1-2: Differentiation between stage-II and stage-IV supination-external rotation fractures (Lauge-Hansen Classification) is clinically relevant as an intact deltoid ligament stabilizes the ankle mortise and open reduction is thought to be unnecessary. The best way to determine deltoid integrity would be to perform a gravity stress radiograph.

    Asnwer 3: The cotton test is an intra-operative test to assess for syndesmotic injury. It would not be used in an awake patient for this purpose.

    Answer 5: A positive squeeze test is suggestive of a syndesmotic injury. However, this test has a lower sensitivity for detecting syndesmotic injury in comparison to stress radiographs.

     

     

     

     

     

    OrthoCash 2020

     

  117. Figures A through D are the radiographs and 3D reconstructed computed tomography images of the left elbow of a 23-year-old man who fell off of a ladder while working construction. He is right hand dominant. All of the following could be appropriate elements of his treatment, EXCEPT:

     

     

     

     

     

     

     

     

     

     

     

     

    1. Elbow-spanning external fixation

    2. Medial collateral ligament (MCL) repair

    3. Lateral collateral ligament (LCL) repair

    4. Radial head excision

    5. Radial head replacement Corrent answer: 4

    This patient has an elbow fracture dislocation with a comminuted radial head fracture as well as a coronoid fracture (terrible triad). Operative treatment is indicated for this unstable injury and must include radial head fixation or replacement. Radial head excision alone is rarely appropriate.

     

    Restoration of elbow stability begins by restoring the congruity of bony

    articulation. In terrible triad injuries, coronoid and radial head fractures must be addressed. Radial head excision will not adequately restore articular congruity. Internal fixation and radial head replacement will both restore the articulation but radial head fixation should be reserved for simple, non-comminuted fracture patterns. Associated elbow dislocation makes fixation of radial head fractures less likely to be successful.

     

    Moro et al. found that in 24 patients (25 elbows) with unreconstructable radial head fractures, patients had mild-moderate impairment when fixed with a radial head replacement. Subjective patient satisfaction was high with radial head replacement.

     

    Ring et al. followed 11 patients with terrible triad injuries. The combination of radial head and coronoid injuries was very unstable. Seven patients redislocated in a splint after reduction and five redislocated after operative treatment, including all four patients treated with radial head resection. The authors concluded that restoration of radiocapitellar contact was essential for elbow stability.

     

    Figures A and B show an elbow fracture dislocation. Computed tomography confirmed coronoid fracture as well as multifragmentary articular comminution of the radial head.

     

    Incorrect answers:

    Answer 1: External fixation may be appropriate if the elbow remains unstable after bony and soft tissue repairs.

    Answer 2: Medial collateral ligament repair may be appropriate if fracture fixation or radial head replacement plus LCL repair does not restore stability. Answer 3: LCL repair is required to restore stability and would be appropriate Answer 5: Radial head replacement is appropriate for this comminuted fracture.

     

     

     

    OrthoCash 2020

     

  118. An 18-year-old patient sustains a comminuted left femoral fracture starting 6.5cm distal to the lesser trochanter. He undergoes antegrade femoral nailing in the supine position on a radiolucent table. Upon completion of proximal and distal interlocking, both patellae are positioned facing the ceiling and a lateral radiographs confirms that the posterior condyles of both limbs are aligned. On AP imaging of both femora, it is noted that the lesser trochanter of the left (injured) side is larger than the right (uninjured) side. Assuming symmetrical anteversion, the left femur has been nailed

    1. with varus malalignment

    2. with valgus malalignment

    3. with external rotation malalignment

    4. with internal rotation malalignment

    5. with no malalignment Corrent answer: 4

    When the lesser trochanter (LT) profile is larger than the uninjured side, the proximal fragment is externally rotated. This leads to an overall internal rotation (IR) malalignment of the distal fragment. Malalignment is described based on the distal fragment relative to the proximal fragment. For more proximal femoral fractures, the proximal fragment tends to be flexed and externally rotated due to the iliopsoas. Matching rotation requires external rotation of the distal fragment when the patient is supine on a fracture table.

     

    Rotational malalignment is the most common complication of intramedullary nailing of a comminuted diaphyseal femoral fracture. The rotational profile of the lesser trochanter can be used to evaluate rotational alignment. The proximal femur is rotated until a neutral position is obtained as judged by the radiographic profile of the lesser trochanter. If the AP image shows a smaller lesser trochanter, there is IR of the LT. A larger LT indicates external rotation (ER) of the LT.

     

    Jaarsma et al. describe CT imaging in determining rotational alignment. They note that the incidence of post-nailing malalignment > 10 ° is 40%, > 15 ° is 20-30%, and > 20 ° is 16%. They also note that patients with ER deformities have more symptoms than those with IR deformities, and that small deformities <15 ° give rise to less complaints. This is because ER deformities lead to compensation with hip retroversion, which causes more symptoms than hip anteversion when compensating for IR deformities.

     

    Incorrect Answers:

    Answers 1 and 2: Difference in LT size between sides is not indicative of coronal plane malalignment.

    Answer 3: ER malalignment of the distal fragment implies relative IR of the proximal fragment. IR of the LT results in a smaller-appearing LT compared with the other side.

     

     

     

    OrthoCash 2020

  119. A 21-year-old male is brought to the emergency department after being involved in gang-related violence. A radiograph of his pelvis is shown in Figure A. The patient is hemodynamically stable. Which of the following imaging modalities is the next best step in evaluating this patient for the most common associated injury?

     

     

     

     

    1. Ultrasound bladder to exclude bladder perforation

    2. CT abdomen to exclude bowel perforation

    3. MRI pelvis to exclude genital injuries

    4. CT angiogram exclude laceration of major vessels

    5. CT acetabulum to exclude intra-articular foreign body Corrent answer: 2

    Low velocity gunshot wounds (GSW) to the hip are most commonly associated with bowel perforation. Consultation with general surgery (or in some facilities, trauma surgery) is necessary to exclude this.

     

    The incidence of GSW is increasing and it is the 2nd leading cause of death in young males in the US after motor vehicle accidents. The incidence of a GSW to the buttock is approximately 8% of all GSW to the extremities. Potential complications of pelvic and acetabular GSW include septic arthritis, enterocutaneous, enteroacetabular, and vesicoacetabular fistulas, infected nonunion, malunion, and injuries to the iliac vessels. The presence or absence of intra-abdominal injuries affects treatment and outcome.

     

    Bartkiw et al. reviewed 2808 GSW and found 1235 associated fractures including 42 fractures of the hip and pelvis. Ten orthopaedic operative

    procedures were performed in 7 patients. Associated nonorthopaedic injuries included 15 small/large bowel perforations (36%), 7 vessel lacerations (17%), and 2 urogenital injuries (5%) that required surgery.

     

    Najibi et al. reviewed 39 GSW to acetabulum. They found 32 simple and 7 associated fracture patterns. The most common simple and associated patterns were anterior column and both column, respectively. Bowel injuries were the most common associated injures (42%). Predictors of poor outcome include high-velocity missile, involvement of acetabular dome, abdominal injury, nerve injury, vascular injury, and male gender. Deep infection was associated with primary anastomosis of bowel injury and an associated fracture pattern.

     

    Figure A shows a GSW to the right hip with acetabular fracture and visible bullet fragment.

     

    Incorrect Answers:

    Answers 1, 3, 4: Urogenital and vascular injuries are not as common as bowel injuries.

    Answer 5: Intraarticular bullet fragments should be removed to prevent lead toxicity (plumbism), which arises from fragments dissolving in synovial fluid. Other reasons for removal include prevention of third body wear, joint sepsis and lead arthropathy. It is not the priority in this case and exclusion of bowel injury must take precedence.

     

     

     

    OrthoCash 2020

     

  120. When treating a proximal tibia fracture, the surgeon decides to

    (1) use blocking screws in the proximal fragment, and (2) pick the intramedullary nail based on the location of the Herzog curve. Which of the following combinations will best prevent the classic deformity associated with this fracture?

    1. Place blocking screws medial and posterior to the nail. Use a nail with a Herzog curve proximal to the fracture site.

    2. Place blocking screws medial and posterior to the nail. Use a nail with a Herzog curve distal to the fracture site.

    3. Place blocking screws lateral and posterior to the nail. Use a nail with a Herzog curve proximal to the fracture site.

    4. Place blocking screws lateral and posterior to the nail. Use a nail with a Herzog curve distal to the fracture site.

    5. Place blocking screws lateral and anterior to the nail. Use a nail with a Herzog curve proximal to the fracture site.

    Corrent answer: 3

     

    Proximal tibial fractures develop an apex anterior (procurvatum) and valgus malalignment. Blocking (poller) screws should be placed in the concavity of the deformity, thus posterior and lateral to the nail. The Herzog curve should be proximal to the fracture site.

     

    Up to 58% of proximal tibial fractures are malaligned. Malalignment arises because the nail fits loosely in the wide metaphyses and cannot control alignment. Without close fit of the nail at the fracture site, the nail will not align the fracture independent of a stable reduction and careful nail path. Blocking screws serve to reduce the size of the proximal metaphyseal canal and guide final nail passage.

     

    Stinner et al. discuss strategies in proximal tibial fracture nailing. They describe an accurate starting point (using the twin peaks AP view or fibular bisector AP view, and flat plateau lateral view). They emphasize fracture reduction prior to reaming and implant placement.

     

    Hiesterman reviewed nailing of extra-articular proximal tibial fractures. Techniques described include blocking screws, unicortical plating, using a universal distractor, nailing in flexion/locking in extension, semiextended nailing (including percutaneous suprapatellar quads-splitting approaches), multiple proximal interlocking screws (>=3).

     

    Illustration A shows placement of a coronal blocking screw. Illustration B shows placement of a sagittal blocking screw. Illustration C shows the effect of the Herzog curve. A more distal Herzog curve leads to a "wedge" effect and fracture displacement whereas a proximal Herzog curve contains the fracture.

    The "wedge" effect occurs as the nail is seated and impinges on the posterior cortex of the distal segment accentuating an apex anterior deformity because of the effective widening of the nail above the bend and posterior force on the distal segment to match the nail shape.

     

    Incorrect Answers:

    Answers 1, 2, 4, 5: Blocking screws in the proximal fragment should be lateral and posterior to the nail. The Herzog curve should be proximal to the fracture site.

     

     

     

     

     

     

     

     

     

     

    OrthoCash 2020

     

  121. What is the most common fracture associated with a lateral subtalar dislocation?

    1. Distal fibular fracture

    2. Cuboid fracture

    3. Calcaneus fracture

    4. Talus fracture

    5. Navicular fracture

     

    Corrent answer: 2

     

    The most common tarsal fracture associated with lateral dislocations is the cuboid, although the anterior process of the calcaneus and the lateral process of the talus can also be affected. In medial dislocations, the dorsomedial talar head, the posterior tubercles of the talus, and the lateral navicular are most often fractured.

     

    Post-reduction x-rays need to be scrutinized carefully for fractures of the tarsal bones as this is the most common injury associated with subtalar dislocations.

    Occult fractures of the lateral process of the talus are also associated with these injuries.

     

    The referenced study by Wagner et al noted radiographic subtalar degenerative changes in 2/3 of their patients, but noted no correlation between radiographic and clinical outcomes.

     

    The referenced study by Bibbo et al noted an 88% incidence of other concurrent injuries and an 89% rate of radiographic degenerative changes at 5 year follow-up. This study reported the cuboid as the most frequently fractured bone with these dislocations.

     

    The referenced article by Saltzman et al is a review of hindfoot dislocations. They recommend conservative treatment if an anatomic reduction is achieved and no osteochondral fragments are seen in the joint post-reduction (via a CT scan).

     

    Illustration A shows a lateral subtalar dislocation.

     

     

     

     

     

     

    OrthoCash 2020

     

  122. A 72-year-old female presents to the office 5 weeks after distal radius fracture surgery with the findings seen in Figure A. She performed daily cleansing with soap and water and dry dressings.

    Which of the following has been shown to decrease the risk of developing this complication?

     

     

     

    1. A solution of 0.45% sodium chloride and hydrogen peroxide

    2. A solution of 0.9% sodium chloride and hydrogen peroxide

    3. A solution of 0.45% sodium chloride and chlorhexidine

    4. A solution of 0.9% sodium chloride and chlorhexidine

    5. None of the above, as specialized cleansing solutions do not decrease the risk this complication

    Corrent answer: 5

     

    There is no difference between daily showers with soap, water and dry dressings, and solutions comprising (1) saline and chlorhexidine, or (2) saline and hydrogen peroxide.

     

    The risks of external fixation include cellulitis, pin-track drainage, osteomyelitis and pin loosening. The rate of complications is about 20%. Chlorhexidine gluconate has broad spectrum activity against gram-positive and negative bacteria by disrupting cell membranes. It is not affected by blood and has low skin irritancy. Studies show that simple pin-site care (showers, dry dressings) maintains a low infection rate.

     

    Egol et al. performed a randomized trial examining external fixation pin tracts about the wrist. They compared (1) weekly dry dressings, (2) daily pin care with half normal saline (NS) and hydrogen peroxide (H2O2) and (3) chlorhexidine discs with weekly changes. They found pin-site complications in 19%, with no difference between the 3 groups. They do not recommend additional wound care beyond sterile dressings.

     

    Stinner et al. performed a survey on the use of half-pins by the limb lengthening and reconstruction society. They found that most respondents preferred hydroxyapatite coating (81%) because of improved fixation and decreased loosening (less radiographic pin-tract rarefaction and greater extraction torque, which may lead to lower infection rates). Most encouraged

    shower (60%) and washing solution (67%)(soap and water, peroxide or saline) for pin site care.

     

    Figure A shows an external fixator around the wrist with cellulitis around the proximal half-pins.

     

    Incorrect Answers

    Answers 1-4: Hydrogen peroxide and chlorhexidine based solutions have not been conclusively shown to decrease the rate of pin site complications.

     

     

     

    OrthoCash 2020

     

  123. A 33-year-old female sustains the injury shown in Figure A as the result of a fall off a chair, and subsequently undergoes operative stabilization of her injury. Which of the following is most correlated with positive outcomes when treating this injury?

     

     

     

     

    1. Immediate weightbearing

    2. Subchondral debridement of any osteochondral defect

    3. Repair of medial ligamentous structures

    4. Casting or splinting in a neutral position postoperatively

    5. Anatomic reduction of the syndesmosis Corrent answer: 5

    Long-term outcomes after an ankle syndesmotic injury are most correlated with an anatomic reduction of the ankle syndesmosis. Formal open reduction of the syndesmosis has been shown to improve outcomes by improving the reduction quality.

     

    While the importance of anatomic reduction of a syndesmotic injury is clear, controversy exists regarding the ideal method of fixation. No significant differences are reproducibly reported in regards to number of syndesmotic screws, size, or number of cortices. There is emerging data supporting the use of suture button fixation.

     

    Wikeroy et al. reviewed 48 patients at a mean of 8.4 years, and they found that patients with a difference in the syndesmotic width between the operated and the nonoperated ankle of 1.5 mm or more showed inferior results.

    Posterior malleolar fragments and obese patients also had worse outcomes.

     

    Schepers et al. published a review on the suture button device comparisons to traditional screw fixation, reviewing 6 biomechanical studies and 34 clinical studies. They found that the suture button systems have similar outcomes to screw fixation, but insufficient long-term and high-quality evidence prevented a strong conclusion. Implant removal in the suture button groups averaged 10%, while screw removal averaged 52%.

     

    Sagi et al. reviewed 107 patients with ankle fractures and associated syndesmotic injuries requiring surgery. They found that 39% were malreduced, but open reduction of the syndesmotic injury cut the malreduction rate by 2/3. They also reported that at a minimum of 2 years follow-up, patients with malreduced syndesmotic injuries demonstrated worse functional outcome scores.

    Figure A shows an ankle fracture with obvious syndesmotic injury/widening. Incorrect answers:

    1-4: These choices are not correlated with excellent outcomes with treatment

    of a syndesmotic injury.

     

     

     

    OrthoCash 2020

  124. A 40-year-old man is thrown off his motorcycle and sustains an open Type IIIA fracture shown in Figure A. He is taken to the operating room for debridement and reamed intramedullary nailing with a 10mm diameter nail. He returns at 10 months with persistent pain at the fracture site with ambulation. Examination reveals healed wounds with no erythema, warmth or tenderness. Erythrocyte sedimentation rate and C-reactive protein levels are within normal limits. Radiographs taken at that time are shown in Figure B. What is the next best treatment step?

     

     

     

     

    1. Adjunctive plate fixation without nail removal

    2. Nail removal and plating

    3. Partial fibulectomy at the same level as the tibia fracture and weightbearing cast application

    4. Exchange nailing

    5. Local administration of rhBMP-2 Corrent answer: 4

    This man had a mid-diaphyseal tibial fracture that has gone into nonunion. Reamed exchange nailing is indicated.

    Tibial delayed union can be defined as lack of union from 20-26 weeks postinjury, while nonunion is defined as lack of healing at >9mths post-injury, or absence of progressive signs of healing on radiographs for 3 consecutive months. Persistent pain is a symptom of nonunion. ESR and CRP are performed to rule out infection.

     

    Bhandari et al. performed a blinded, multicenter trial on 622 reamed tibial nails and 604 unreamed tibial nails. In closed fractures, patients in the unreamed nail group were at greater risk of primary events than the reamed nail group.

    There was no difference in groups for open fractures. Primary events were defined as bone-grafting, implant exchange/removal, dynamization, and debridement.

     

    Hak reviewed aseptic tibial nonunion. They discuss exchanged reamed nailing for diaphyseal nonunion, adjunctive plate fixation for metaphyseal nonunion, and nail removal and plating for metadiaphyseal nonunion, external fixation for infected nonunion and distraction osteogenesis of defects.

     

    Figure A shows a mid-diaphyseal tibial fracture Figure B shows nonunion following IM nailing of the fracture. Illustration A shows union following exchange nailing with a larger 12mm diameter nail.

     

    Incorrect Answers:

    Answer 1: Adjunctive plate fixation improves mechanical stability and is indicated for hypertrophic nonunions. It functions best for metaphyseal nonunion.

    Answer 2: Nail removal and plating is an option for metadiaphyseal nonunion. In this location, exchange nailing is unlikely to improve mechanical stability.

    Further, the plate can be used to correct deformity, and other techniques can be used concurrently (interfragmentary lag screws, dynamic compression plates, push-pull compression devices, articulated tension device).

    Answer 3: Partial fibulectomy at the same level will destabilize the construct. This is compounded by removal of internal fixation and placement of an external cast.

    Answer 5: rhBMP-2 has been approved for open tibial fractures managed with IM nailing, but not for tibial nonunion. In contrast, rhBMP-7 is not FDA approved, but has been approved under Humanitarian Device Exemption for use for recalcitrant long bone nonunions where autograft is unfeasible and other treatments have failed.

     

     

     

     

     

    OrthoCash 2020

     

  125. A 22-year-man is shot once with a handgun on the way to the library by an unknown assailant. Examination reveals an entry wound in the left buttock, but no exit wound. There is blood on digital rectal examination. A sigmoidoscopy is planned. A radiograph of his right hip and CT scan image are shown in Figures A and B respectively. What is the next best step?

     

     

     

     

     

     

    1. 24 hours of empiric antibiotics that cover gram-negative and enteric organisms

    2. single dose of empiric antibiotics that cover gram-positive, gram-negative and enteric organisms

    3. surgical debridement of the hip, bullet removal, 24 hours of empiric antibiotics that cover gram-negative and enteric organisms

    4. surgical debridement of the hip, bullet removal, single dose of empiric antibiotics that cover gram-positive, gram-negative and enteric organisms

    5. surgical debridement of the hip, 24 hours of empiric antibiotics that cover gram-positive, gram-negative and enteric organisms

    Corrent answer: 5

     

    This patient has a transabdominal gunshot wound (GSW) to the right hip. Urgent irrigation and debridement of the hip joint is indicated. Antibiotic coverage should be for at least 24 hours and should cover gram-positive, gram-negative and enteric organisms.

    Transabdominal GSW of the hip and pelvis are those that traverse the gastrointestinal system before entering the pelvis and hip and may be contaminated by bowel contents. Blood on rectal examination, and bullet paths crossing the midline at or below the pelvic brim raise concern for lower GI injuries and should be evaluated with sigmoidoscopy.

     

    Miller et al. reviewed transabdominal GSW to the hip and pelvis. They state that hip injury is evidenced by retained bullets in the joint, intraarticular air on CT, or acetabular or femoral head fracture without retained missile components. If there is hip injury, debridement is necessary. If the hip joint is not involved, the patient should be assessed for other pelvic injuries. If there is no need for surgical stabilization, at least 24 hours of antibiotics alone will suffice.

     

    Bartkiw et al. reviewed 2808 GSW and found 1235 associated fractures including 42 fractures of the hip and pelvis. They performed 10 orthopaedic procedures on 7 patients. All fractures healed with no pelvic ring instability or chronic osteomyelitis. They recommend orthopedic intervention for intraarticular projectiles or bone fragments, and reconstruction of the hip and acetabulum.

     

    Incorrect Answers

    Answer 1: As the hip joint is involved, surgical debridement is necessary. Antibiotic coverage should include gram-positive coverage as contaminants can be introduced by bullet passage through clothes and skin.

    Answer 2: At least 24 hours of antibiotic coverage is indicated for prophylaxis against osteomyelitis and localized abscess formation. A longer period may be necessary depending on injury severity and overall patient condition.

    Answers 3 and 4: Bullet removal is only indicated for intra-articular bullets to prevent late septic arthritis, lead toxicity, and articular damage with motion of the joint. Bullets retained in soft tissues can be observed or removed on an elective basis if they prove problematic.

     

     

     

    OrthoCash 2020

     

  126. Figures A and B are radiographic images of an 85-year-old woman with isolated left hip pain. She describes a non-syncopal fall from standing 4 hours ago. Physical examination reveals pain with log-rolling the left thigh and the inability to bear weight on the affected leg. The radiologist reports no fracture in the left hip. What would be the next best step?

     

     

     

    1. Stress view radiographs of the left hip

    2. Non-weight bearing and pelvic bone scan in 7 days

    3. Non-weight bearing and repeat the CT scan after 48 hours from injury

    4. MRI hip and pelvis

    5. Weight bearing activity as tolerated with close follow-up Corrent answer: 4

    The next best step would be an MRI hip and pelvis to investigate for an occult fracture of the left hip.

     

    Moderate evidence supports MRI as the advanced imaging of choice for diagnosis of presumed hip fracture not apparent on initial radiographs. MRI has been shown to be able to detect occult fractures earlier than bone scan, with better spatial resolution. Usually the MRI should be obtained in less than 24 hours from the time of injury. For situations in which MRI is not immediately available, bone scan can be considered after 72 hours form the time of injury. However, this may compromise patient care and put the patient at risk of fracture displacement.

     

    Cannon et al. reviewed the imaging of choice in occult hip fracture. They showed that physical examination yields a poor sensitivity identifying occult hip fractures, with log-rolling and straight-leg raise as 50% and 70%, respectively. The most sensitive modality for occult fracture identification was MRI.

     

    Iwata et al. retrospectively reviewed a cohort of 35 patients with clinically suspected fractures of the hip that underwent MRI. All radiographs were

    negative. In 26 of these patients, a T1-weighted coronal MRI showed a hip fracture with 100% sensitivity.

     

    Roberts et al. reviewed the 2015 AAOS Clinical Practice Guideline: Management of Hip Fractures in the Elderly. They report moderate evidence that supports MRI as the advanced imaging of choice for diagnosis of presumed hip fractures not apparent on initial radiographs.

     

    Figure A is a AP radiograph of the left hip and pelvis. Apart from a healed fracture of the ischiopubic rami and generalized osteopenia, there is no obvious hip fracture. Figure B is a coronal CT image that does not demonstrate evidence of an acute hip fracture. Illustration A is a T1 weighted MR image that shows a non displaced fracture (white arrow) through the intertrochanteric region of the left proximal femur.

     

    Incorrect Answers:

    Answer 1: Stress view radiographs would not be recommended.

    Answer 2: If MRI is contraindicated, for example if a pacemaker is present, then a bone scan at 72 hours is the next test of choice. Waiting 7 days would not be appropriate for this patient.

    Answer 3: Repeating the CT scan will not help to identify fracture, unless the fracture becomes displaced. A delay in identifying a fracture pattern by 48 hours, while the patient is non-mobile, significantly increases their risk of complications.

    Answer 5: Weight bearing activity as tolerated with close follow-up may be suggested if the patient is clinically able to walk and advanced imaging is negative for fracture. The best modality to rule-out occult fracture is MRI.

     

     

     

     

     

     

    OrthoCash 2020

     

  127. All of the following are considered contraindications to the use of functional bracing of a humeral shaft fracture EXCEPT:

    1. Mid-diaphyseal segmental fracture with ipsilateral pilon fracture

    2. Mid-diaphyseal fracture with radial nerve palsy from nonballistic penetrating injury

    3. Proximal one-third oblique fracture

    4. Mid-diaphyseal closed fracture with a radial nerve palsy on presentation

    5. Mid-diaphyseal fracture with a L1 burst fracture and paraplegia on presentation

    Corrent answer: 4

     

    A closed mid-diaphyseal humerus fracture with a radial nerve palsy on presentation is not a contraindication to functional brace management.

     

    Commonly accepted parameters for closed treatment include less than 30 degrees of varus angulation, 20 degrees of anterior/posterior angulation, and 3 cm of shortening. Operative indications are: associated vascular injuries, bilateral humeral shaft fractures, polytrauma patient (including paraplegia), segmental fractures, injury to the brachial plexus, pathological fractures, floating elbow, and floating shoulder.

     

    The article by Rutgers and Ring found that proximal one-third oblique humeral shaft fractures had an unacceptably high 29% rate of nonunion treated with a functional brace.

     

    The article by Sarmiento et al found a 97% rate of union, a radial nerve palsy incidence of 11%, and no contraindication to the use of functional braces in humeral shaft fractures associated with radial nerve palsy.

     

    The review article by Defranco and Lawton states that 70% of these radial nerve injuries recover spontaneously. They note that it "seems reasonable, however, to consider surgical intervention (radial nerve exploration) between 4 and 6 months based on the patient’s clinical course."

     

     

     

    OrthoCash 2020

     

  128. A 27-year-old woman falls off a speedboat and sustains a femoral neck fracture. Following fracture fixation, she develops the condition shown in Figure A. It is determined that the fracture line lies 40° to the horizontal plane as shown in Figure B. The surgeon decides to use a dynamic hip screw for revision. The screw is first placed across the fracture site and lies at 130° to the vertical plane as shown in Figure C. The surgical correction necessary is

     

     

     

     

     

     

    1. 20° wedge osteotomy and 110° side plate

    2. 20° wedge osteotomy and 140° side plate

    3. 20° wedge osteotomy and 150° side plate

    4. 35° wedge osteotomy and 165° side plate

    5. 35° wedge osteotomy and 95° side plate

     

    Corrent answer: 3

     

    This patient had a femoral neck fracture fixed with cannulated screws that subsequently went into nonunion. The valgization (valgus producing intertrochanteric) osteotomies that will correct this deformity is a 20° wedge osteotomy held with a 150° side plate.

     

    The aim of a Pauwel's osteotomy is to reduce the obliquity of the fracture line. Using the DHS method, the side plate implant angle (150°) = guide pin insertion angle (130°) + osteotomy angle (20°). Similarly, if one made a 10° osteotomy, the implant required would be 130° + 10° = 140°.

     

    Hartford et al. describe fixing intertrochanteric osteotomies with dynamic hip screws in 8 patients with femoral neck fracture (Pauwels type III) nonunion. The fracture plane decreased from 68° to 41°, and hip scores increased. Seven fractures healed (1 patient died). They recommend this technique for reliable fixation of intertrochanteric valgus osteotomies for femoral neck nonunion.

     

    Figure A shows nonunion of a femoral neck fracture after cannulated screws. Figure B shows a fracture line at 40° to the horizontal. Figure C shows DHS screw trajectory 130° to the vertical. Illustration A shows the technique of fixation of a fracture with a 40° angulation to the horizontal with a 20° lateral closing wedge osteotomy and a 150° DHS sideplate (parts A-F). Illustration B shows the same fracture fixed with a fixed angle device (left, blade plate) that went on to union (right, radiograph 7 years later). Illustration C shows correction of the fracture angle from 55° to 20° with a traditional Pauwels 35° valgization osteotomy.

     

    Incorrect Answers:

    Answers 1 and 5: As the screw passage has already been made, a side plate

    <130° (such as 110° or 95° would essentially be a varus-producing ostoetomy).

    Answer 2: A 20° osteotomy paired with a 140° side plate will result in medial cortical contact but lateral cortical gapping. Diminished contact area at the osteotomy site is a risk for nonunion and implant failure.

    Answer 4: A 35° osteotomy would correct the fracture line to 5° from the horizontal, but would necessitate a 165° side plate. This is less desirable as (1) a larger closing-wedge osteotomy would result in increased femoral

    shortening, and (2) a 165° side-plate does not exist in standard inventory, and if absolutely necessary, would require customized manufacturing. Plates with 130-150° barrel angle are available as standard inventory.

     

     

     

     

     

     

     

     

     

     

     

    OrthoCash 2020

     

  129. A healthy 39-year-old male presents to clinic with posttraumatic elbow stiffness after a minimally displaced radial head fracture. His injury occurred 4 months ago with no improvement in range of motion despite 10 weeks of supervised physiotherapy. Follow-up radiographs reveal normal osseous anatomy. What is the next best step in treatment?

    1. Intra-articular and extra-capsular cortisone injection

    2. Closed manipulation under anesthesia

    3. Aggressive home exercise program

    4. Continuous passive motion device

    5. Static or dynamic progressive elbow splinting Corrent answer: 5

    Supervised exercise therapy with static or dynamic progressive elbow splinting over a 6 month period has shown to have the greatest improvement on DASH scores and functional range of motion (ROM) in patients with post-traumatic elbow stiffness.

     

    The goal of treatment in post-traumatic stiffness is to restore a functional range of elbow motion (30° to 130°). Non-operative modalities are considered the first-line of treatment. Aggressive physical therapy has traditionally been advocated. However, the use of static or dynamic progressive elbow splinting with a turnbuckle has shown to provide better functional outcomes. Treatment is usually maintained over a period of 6-12 months. Surgery is considered when nonoperative therapy fails.

     

    Gelinas et al. treated 22 patients with an elbow contracture using a static progressive turnbuckle splint for a mean of 4.5 +/- 1.8 months. The mean range of motion improved from 32 - 108, to 26 - 127 degrees (p = 0.0001). Their results suggest that static progressive splinting is an effective modality for postoperative elbow stiffness.

     

    Lindenhovius et al. randomized sixty-six patients with post-traumatic elbow stiffness into static progressive elbow splint therapy or dynamic elbow splinting over a 12 month period. There was no significant difference in outcomes between treatment modalities. ROM increased by 40° vs. 39° at six months, respectively. DASH scores improved from 50 vs 45 at enrollment to 32 vs. 25 at six months, respectively.

     

    Illustration A shows an image of a static progressive elbow splint.

    Incorrect Answers:

    Answer 1: Intra-articular and extra-capsular cortisone injection have not shown to improve ROM in this scenario.

    Answer 2: Closed manipulation under anesthesia may worsen elbow stiffness and cause intra-articular damage. Manipulation causes significant swelling and inflammation with tearing of soft tissues, causing hemarthrosis and additional fibrosis in the joint.

    Answer 3: Aggressive home exercise program are not effective when formal physiotherapy has failed.

    Answer 4: Continuous passive motion machines have a limited role in treating established contractures. They do not seem to improve end-range mobility in these patients.

     

     

     

     

     

     

    OrthoCash 2020

     

  130. A 28-year-old construction worker sustains the closed injury shown in Figures A and B after a fall from a height. He is taken to the operating room. What is the next best step?

     

     

     

     

     

    1. Locked anterior tibial plating and fibular plating

    2. Locked medial tibial plating and fibular plating

    3. Reamed intramedullary nailing without fibular plating

    4. Unreamed intramedullary nailing and fibular plating

    5. Reamed intramedullary nailing and fibular plating Corrent answer: 5

    This patient has an extraarticular distal tibia fracture and distal fibula fracture. Reamed intramedullary nailing and fibular plating is indicated in this case.

     

    In the distal tibial metaphysis, there is no snug endosteal fit for an IM nail. Center-center nail placement in both proximal and distal fragments is necessary to maintain alignment. There is also increased stress on distal locking bolts to maintain fracture alignment. Assuming static medial-lateral distal locking screws, accurate coronal plane and rotational alignment is achieved by fibular plating as a first step. This also prevents late loss of alignment because of distal locking screw toggle. Reamed nailing allows a stiffer, larger nail to be placed, and allows redistribution of endosteal osteogenic material to the fracture site. Although there is endosteal vascular compromise, this does not affect fracture healing because of intact periosteal supply.

     

    Bhandari et al. conducted a prospective, randomized, blinded comparison of 622 patients who had reamed nailing, and 604 who had unreamed nailing. For closed fractures, a significantly greater number in the unreamed group required bone grafting, implant exchange and dynamization. There was no difference in groups for open fracture nailing.

     

    Egol et al. retrospectively reviewed distal metaphyseal tibia-fibula fractures treated with IM nailing with (25 cases) and without (47 cases) adjunctive plating. They found that plating was associated with maintenance of reduction (significant) as was the use of 2 medial-lateral distal locking bolts (not significant). They recommend fibular plating when IM nailing for distal tibia fractures.

     

    Figures A and B show an extraarticular distal tibia fracture with distal fibula fracture.

     

    Incorrect Answers

    Answers 1 and 2: Locked tibia and fibular plating plating has more soft tissue dissection compared with fibular plating and tibia IM nailing. This increases the risk of wound complications.

    Answer 3: Fibular plating reduces postoperative valgus malalignment and late

    loss of alignment.

    Answer 4: In closed tibia fractures, unreamed nailing has a significantly greater need for bone-grafting, secondary dynamization, and exchange nailing compared with reamed nailing. In open tibia fractures, this difference is not significant.

     

     

     

    OrthoCash 2020

     

  131. Total hip arthroplasty is most appropriate for the injury shown in Figure A for which of the following patients?

     

     

     

     

    1. 66-year-old female golf instructor

    2. 66-year-old female with Parkinsons dementia

    3. 87-year-old male household ambulator

    4. 50-year-old male alcoholic with hepatic encephalopathy

    5. 24-year-old male laborer Corrent answer: 1

    Figure A is an AP radiograph demonstrating a displaced femoral neck fracture. Active older patients who present with a displaced femoral neck fracture should

    be treated with total hip arthroplasty (THA).

     

    Displaced femoral neck fractures can present a challenge to treat. In younger patients with good bone stock a closed vs. open reduction and internal fixation should be attempted. For active older patients a total hip arthroplasty is the best option, especially if there is pre-existing arthritis in the injured hip. THA provides the best function with the least pain and less need for repeat surgery (compared to hemiarthroplasty). For low-demand or debilitated patients, for patients older than age 80, or for those who can not reliably follow hip precautions a hemiarthroplasty provides the lowest risk of dislocation, and thus would be the treatment of choice.

     

    Macaulay et al. present a prospective randomized trial of patients with femoral neck fractures treated with THA vs hemiarthroplasty. They found that functional outcomes and patient satisfaction were higher in the THA group without significant increased risk of complications. Inclusion criteria required patients to be over age 50, be a community ambulator, and were excluded for presence of dementia.

     

    Abboud et al. retrospectively reviewed patients treated with THA for osteoarthritis and compared them to patients treated with THA for a femoral neck fracture. They found no significant difference between the two groups for outcomes or complications.

    Figure A is an AP radiograph demonstrating a displaced femoral neck fracture. Incorrect Answers:

    Answer 2: Parkinsons dementia presents an increased risk for dislocation and

    would make a hemiarthroplasty a more suitable choice.

    Answer 3: According to AAOS guidelines, age greater than 80 and low demand status is an indication for hemiarthroplasty

    Answer 4: Hepatic encephalopathy would raise concern for the ability to follow hip precautions and presents an increased risk for dislocation. Thus a hemiarthroplasty would be a better choice.

    Answer 5: For a young male laborer a closed vs open reduction and internal fixation should be attempted rather than an arthroplasty.

     

     

     

    OrthoCash 2020

     

  132. A 50-year-old woman loses her balance and falls off 3 stairs, landing awkwardly on her right knee. Examination reveals loss of dorsalis pedis and posterior tibialis pulses and the foot is cool to the

touch. Her body mass index is 52. Reduction is performed successfully in the emergency room under sedation. Pulses are still not palpable and the foot remains cool to touch. Ankle brachial index is 0.72 (right) and 1.01 (left). What is the most appropriate next step?

 

 

 

 

 

 

 

  1. Splint the knee and admit the patient for hourly ankle brachial index monitoring

  2. Emergent vascular surgical exploration

  3. Emergent external fixation and reassessment of perfusion